Você está na página 1de 201

Instituto de Fı́sica - UFRJ

Primeira Prova de Fı́sica IA - 2011/1

Obs: em todas as questões em que for necessário, considere que g é o módulo da aceleração da gravidade

Questão 1) Uma bola desliza sem atrito sobre o telhado de um prédio que possui uma inclinação
para baixo de θ (como pode ser visto na figura). A extremidade do telhado está situada a uma
altura H acima do solo e a bola possui uma velocidade inicial de módulo v0 no instante em que
abandona o telhado. Sem considerar a resistência do ar e segundo o sistema de eixos orientados
indicado na figura determine:
a) a equação que representa vetor posição da bola, ~r(t), para um instante de tempo t qualquer;
b) a equação que representa o vetor velocidade da bola, ~v (t), para um instante de tempo t
qualquer;
c) o alcance da bola, isto é, a coordenada x = R quando a bola toca o chão.

Questão 2) Três caixas de massas m1 , m2 e m3 estão empilhadas dentro de um elevador, que


sobe com aceleração constante de módulo igual a a, como mostra a figura. Calcule:
a) a força resultante que atua sobre a caixa de massa m2 ;
b) a força que a caixa de massa m2 exerce na caixa de massa m3 ;
c) a força do piso do elevador sobre a caixa de massa m1 .
Questão 3) Um cubo pequeno de massa m é colocado no interior de um funil. O sitema cubo-
funil gira solidariamente em torno do eixo vertical de simetria do funil e a parede do funil forma
um ângulo θ com a horizontal. O coeficiente de atrito estático entre o cubo e o funil vale µ e o
centro do cubo está a uma distância r do eixo de rotação; vide figura abaixo.
a) Faça um diagrama indicando todas as forças que atuam sobre o cubo, com a sua posição
ocupada como mostra a figura, na condição da iminência de deslizar para baixo.
b) Determine o menor valor da velocidade do cubo para que ele não escorregue para dentro
do funil. Justifique a sua resposta.
c) Determine a frequência de rotação f de acordo com o item anterior.

r θ

Questão 4) Um bloco de massa m está comprimindo uma mola de constante elástica k por meio
de um fio como mostra a figura abaixo. A compressão da mola(posição A) é igual a d e na
posição O ela se encontra relaxada. Há atrito entre o bloco e a superfı́cie sobre a qual ele se
movimenta. Ao ser liberado o bloco oscila algumas vezes em torno de O e pára definitivamente
na posição B localizada a d/2 da posição O. Determine:
a) a energia mecânica do sistema massa-mola em A, imediatamente após o fio ser cortado(bloco
ainda parado);
b) o trabalho da força de atrito fat sobre o bloco desde a sua liberação até parar;
c) dado que o módulo da força de atrito é constante e igual a fat , determine o módulo da
velocidade do bloco, v1 , quando ele passa por O pela primeira vez. Justifique a sua resposta.
Questão 1

a) valor = 0,7 pontos


Usando a equação vetorial para um movimento com aceleração constante e segundo o
sistema de coordenadas indicado na figura temos:

~r(t) = (x0 + v0xt + 1/2ax t2)ı̂ + (y0 + v0y t + 1/2ay t2 )̂


Substituindo na equação acima os dados da questão,

~r(t) = (v0 cos θ t)ı̂ + (H − v0 sin θ t − 1/2gt2 )̂ i)

b) valor = 0,8 pontos


A velocidade ~v (t) é dada pela derivada da posição ~r(t) em relação ao tempo, do item
anterior da relação i),

d ~r(t)
= ~v(t) = (v0 cos θ)ı̂ − (v0 sin θ + gt)̂
dt

c) valor = 1,0 pontos


Para calcular o alcance R podemos utilizar a equação vetorial em ~r(t) impondo a
condição de que ao tocar o solo ~r(t∗) = Rı̂; t∗ corresponde ao tempo de queda da bola. A
outra possibilidade é escrever ry = f(rx ) e fazer ry = 0 quando a bola toca o solo.
De acordo com esta última opção e decompondo o vetor posição ~r(t):
 rx
 rx = v0cosθ t
 → t=
v0cosθ

ry = H − v0senθ t − 21 g(t)2

1 r2
Substituindo t na equação em ry , obtemos ry = H − tanθ.rx − g 2 x 2 . Quando a bola
2 v0 cos θ
toca o solo ry = 0 e rx = R,
r 1 g 
tanθ ± tan 2 θ − 4. − H
1 R2 2 v02cos2 θ
0 = H − tanθ.R − g 2 2 → R=  1
2 v0 cos θ g 
2. − 2 2
2 v0 cos θ

Para o valor positivo de R,


s
v 2sen2θ  v 2sen2θ 2 v02cos2 θH
⇒ R=− 0 + 0
+2
2g 2g g

1
Questão 2

a) valor = (0,7 pontos)


A caixa de massa m2 sobe com aceleração de módulo igual a a. Pela segunda lei de
Newton, a força resultante sobre a caixa é igual a

F~2 = m2~a.

b) valor = (1,0 pontos)


No diagrama de forças para a caixa de massa m3, F~23 representa a força da caixa de
massa m2 sobre a caixa de massa m3 , e P~3 o peso associado a m3. Pela segunda lei de
Newton,
F~23 + P~3 = m3~a

F23 − P3 = m3 a ⇒ F23 = m3 (a + g)

c) valor = (0,8 pontos)


O conjunto m1 + m2 + m3 sobe com aceleração a constante, sob a ação do peso total
P~123 = P~1 + P~2 + P~3 e da força F~piso exercida pelo piso do elevador. Utilizando mais uma
vez a segunda lei de Newton,

F~piso + P~123 = m~a

Fpiso − (m1 + m2 + m3) g = (m1 + m2 + m3 ) a

⇒ Fpiso = (m1 + m2 + m3) (a + g)

2
Questão 3
a) valor = (0,5 pontos)
Veja diagrama abaixo.

θ ~
f~at N
θ
x

P~

b) valor = (1,3 pontos)


No caso limite em que o cubo está prestes a escorregar, fat tem seu valor máximo,
fat = µN(módulos).
Ncosθ + µNsenθ = mg ⇒ N(cosθ + µsenθ) = mg i)

−Nsenθ + µNcosθ = −mv 2/r ⇒ −N(senθ − µcosθ) = −mv 2/r ii)


Dividindo a equação ii) pela i):

(mv 2/r)/mg = (senθ − µcosθ)/(cosθ + µsenθ)

ou s
(senθ − µcosθ)
v= gr
(cosθ + µsenθ)

c) valor = (0,7 pontos)


A frequência está relacionada com a velocidade por f = 1/T = ω/2 π = v/(2πr). De
acordo com o resultado do item anterior,
s
1 (senθ − µcosθ)
f = gr
2πr (cosθ + µsenθ)

s
1 g(senθ − µcosθ)
f=
2π r(cosθ + µsenθ)

3
Questão 4

a) valor = (0,5 pontos)


Na posição A temos apenas a energia mecânica da mola.
1 1
UA = k(−d)2 = kd2
2 2

b) valor = (1,0 pontos)

As energias potencial e cinética nas posições inicial A e final B são:


 UA = 12 k(−d)2 e KA = 0

UB = 21 (d/2)2 e KB = 0

Pelo teorema trabalho-energia ∆K = Wtotal = W C + W N C , onde W C e W N C corres-


pondem ao trabalho das forças conservativas e não conservativas respectivamente. Como
as energias cinéticas inicial e final são nulas:

0 = W C + W NC e W C = Wmola = −∆Umola temos:


1 1 2
2
0 = −(UB − UA ) + Wfat ⇒ 0 = − k(d/2) − kd + Wfat
2 2

Wfat = −3/8kd2

c) valor = (1,0 pontos)


Analogamente ao item b) as energias potencial e cinética nas posições inicial A e final
O são dadas por:

 UA = 21 kd2 e KA = 0

UO = 0 e KO = 12 mv12.

Aplicando novamente o teorema trabalho-energia, W C = 21 kd2 e W N C = f~a ◦ d~ = −fa.d.


r
1 2 1 2 2 1
∆K = mv1 − 0 = kd − fat d ⇒ v1 = ( )[ kd2 − fatd]
2 2 m 2

Justificativa: como o bloco ultrapassa a posição O, [ 12 kd2 − fat d] > 0, ou seja, a


compressão inicial da mola d, deve ser
2
d> fat
k

4
Instituto de Fı́sica - UFRJ

Primeira Prova de Fı́sica IA - 2011/2

Obs: em todas as questões em que for necessário, considere que g é o módulo da aceleração da gravidade

Questão 1) Um ı́on penetra na região entre duas placas planas e paralelas, uma em x = 0 e
a outra em x = D. No instante t = 0, ao passar pela origem com velocidade ~v0 = v0 ı̂, como
representado na figura, o ı́on sofre a ação de forças elétricas, que imprimem ao mesmo uma
aceleração da forma ~a = a1 ı̂ + a2 ̂, onde a1 e a2 são constantes positivas. Considerando o
movimento do ı́on até que este atinja a placa em x = D, e desprezando o peso do ı́on:
a) obtenha os vetores posição ~r(t) e velocidade ~v (t) do ı́on em função do tempo;
b) determine a coordenada yc do ponto de impacto do ı́on com a placa.
c) Em algum instante o vetor velocidade do ı́on é (i) perpendicular ou (ii) paralelo ao vetor
aceleração? Justifique a sua resposta.

Questão 2) Os dois blocos mostrados na figura estão ligados por uma


corda uniforme, inextensı́vel e pesada de massa M. Um agente ex-
terno aplica uma força vertical F ~ contrária ao campo de gravitação
constante ~g, sobre o bloco superior, conforme indicado na figura ao
lado. O bloco superior tem massa m1 e o inferior m2 . Consideramos
como dados: as massas dos três corpos, a força externa F e a acele-
ração da gravidade terrestre g.
a) Para cada bloco e para a corda, desenhe o diagrama de corpo
livre identificando cada força que atua no respectivo corpo.
b) Escreva as equações de Newton para cada bloco e para a corda.
c) Determine a aceleração com que cada bloco e a corda se movi-
mentam.
d) Determine as tensões no topo e no fundo da corda.
e) Qual é o valor da tensão no meio da corda?
Questão 3) Um bloco de massa m é solto a partir do repouso do alto de um plano cuja inclinação
é θ em relação ao plano horizontal. Este plano é feito de um material tal que o coeficiente de
atrito cinético não é constante. Depois de percorrer uma distância d ao longo do plano, o bloco
colide com uma mola de constante elástica k, e de massa desprezı́vel, que se encontra relaxada.
Após o impacto a mola sofre uma compressão s e a massa para momentaneamente. Determine:
a) energia dissipada pela força de atrito, em função dos dados (m, g, k, s, d e θ);
b) o menor valor do coeficiente de atrito estático µe , para que o bloco permaneça em repouso
na situação de compressão máxima da mola.

Questão 4) Uma criança de massa m encontra-se no topo de um domo esférico de raio R


(representado pelo ponto A na figura). Inicialmente em repouso, ela começa a escorregar com
velocidade inicial desprezı́vel, devido a uma pequena perturbação, sem atrito, pelo domo esférico
passando pelo ponto B, onde ainda mantém contato com a superfı́cie, conforme indicado na
figura.
a) A energia mecânica conserva-se? Por quê? Qual é o módulo da velocidade da criança no
ponto B? Considere que o zero da energia potencial gravitacional como sendo o solo.
b) No ponto B, cuja direção OB faz o ângulo θ com a vertical, faça o diagrama de forças que
atua na criança e escreva a 2a Lei de Newton correspondente a esta direção.
c) Calcule a altura, h, na qual a criança perde o contato com a superfı́cie do domo.

A
B

θ
R

O
UNIVERSIDADE FEDERAL DO RIO DE JANEIRO

INSTITUTO DE FÍSICA

FÍSICA I – 2012/1

PRIMEIRA PROVA (P1) – 27/04/2012

VERSÃO: A

INSTRUÇÕES: LEIA COM CUIDADO!

1. Preencha CORRETA, LEGÍVEL E TOTALMENTE os campos em branco do cabeçalho do caderno


de resolução, fornecido em separado.

2. A prova constitui-se de duas partes:

• uma parte objetiva, perfazendo um total de 5,0 pontos, constituı́da por dez (10) questões objetivas (de
múltipla escolha), cada uma das quais valendo 0,5 ponto, sem penalização por questão errada.
• uma parte discursiva, perfazendo um total de 5,0 pontos, constituı́da por duas (2) questões discursivas
(ou argumentativas ou dissertativas), cada uma das quais valendo 2,5 pontos.

3. Acima da tabela de respostas das questões objetivas, na primeira página do caderno de resolução, INDIQUE
CLARAMENTE A VERSÃO DA PROVA (A, B,. . . ).

4. O item considerado correto, em cada uma das questões objetivas, deve ser assinalado, A CANETA (de
tinta azul ou preta), na tabela de respostas correspondente do caderno de resolução

5. É vedado o uso de qualquer instrumento eletro-eletrônico (calculadora, celular, iPod, etc)

6. Seja organizado e claro.

Formulário

sen2 θ + cos2 θ = 1, sen2θ = 2senθcosθ


sen(α ± θ) = senαcosθ ± cosαsenθ, cos(α ± θ) = cosαcosθ ∓ senαsenθ
d n xn+1
Z
x = nxn−1 xn dx = (n 6= −1)
dx n+1
d d
senax = acosax, cosax = −asenax
dx dx

a b c
Lei dos senos: = =
senα senβ senγ
Lei dos cossenos: a2 = b2 + c2 − 2bc cosα

1
Seção 1. Múltipla escolha (10×0,5 = 5,0 pontos)

1. Um rio tem margens, num determinado trecho, pa- 4. Um carro percorre um trilho em forma de laço ver-
ralelas e uma correnteza cuja velocidade horizontal tical, como mostra a figura. No ponto mais alto,
constante é vc . Um nadador no rio sem a corren- A, ele passa sem perder contato com o trilho. Se
teza consegue nadar com velocidade vN > vc . Ele g é o módulo da aceleração local da gravidade e a
resolve atravessar o rio nadando com o módulo da aceleração no ponto A é a, o módulo da força que
velocidade constante vN , de tal maneira que per- o trilho exerce sobre o carro, Fn em A é igual a:
manece sempre perpendicular às margens, para
chegar em linha reta à margem oposta. Se d é
largura do rio neste trecho, o tempo de travessia
será:
q
(a) d/ vN2 − v2
c

(b) d/vN
(c) d/(vN − vc )
q
(d) d/ vN2 + v2
c
(a) zero

(e) d/(vN + vc ) (b) m(g − a)


(c) m(g + a)
(d) mg
(e) Nenhuma das respostas anteriores.
2. Uma partı́cula descreve um movimento circular
uniforme, cujo raio do cı́rculo é R, com velocidade
angular ω. Os vetores ~a, ~aT , e ~ar são os vetores
aceleração do movimento; resultante, tangencial e
radial respectivamente. A resposta correta é:
(a) ~a = constante
(b) ~aT 6= 0
(c) ~ar = constante
(d) |~ar | = w2 R
(e) ~aT = ~ar

3. Dois blocos de massas m1 e m2 estão em um dispo-


sitivo em equilı́brio estático, como mostra a figura.
Não há atrito entre as superfı́cies dos blocos e do
dispositivo. Se α 6= β, qual das opções abaixo é
correta: 5. Um carro de massa M faz uma curva de raio R em
uma estrada plana. O coeficiente de atrito estático
entre os pneus e a estrada é µe . A velocidade
máxima que o carro pode atingir sem derrapar na
curva é; considere que g é valor local da aceleração
da gravidade:

(a) m1 cos α = m2 cos β (a) gR

(b) m1 sen α = m2 cos β (b) πgR
p
(c) m1 sen α = m2 sen β (c) π/2gR

(d) m1 = m2 sen (α + β) (d) 2µe gR

(e) m1 tan α = m2 tan β (e) µe gR

2
6. Um dispositivo é constituido de duas molas de 8. Um canhão dispara dois projéteis A e B com a
constantes elásticas diferentes k1 e k2 que estão mesma velocidade v mas com ângulos de disparo
fixas em paredes opostas. Entre elas há um bloco diferentes em relação à horizontal. Se α = π/4 + δ
de massa m conectando-as. A superfı́cie onde o e β = π/4 − δ a relação entre os alcances RA e RB
bloco se encontra tem atrito desprezı́vel; a figura é igual a:
mostra o dispositivo em equilı́brio estático. Ao ser
(a) RA senα = RB senβ
deslocado para a direita de x e liberado imedia-
tamente, neste instante, atuam sobre o bloco as (b) RA /RB = 1
forças F~1 (mola 1) e F~2 (mola 2). Portanto a opção (c) RA = tan (α − β)RB
abaixo correta é; (ı̂ é o vetor unitário na direção
(d) RA > RB pois α > β
horizontal OX):
(e) RA senβ = RB senα
9. Um fio de massa desprezı́vel de comprimento L, é
preso ao teto num ponto fixo e na sua extremidade
um corpo de massa m, gira em torno da vertical
com velocidade angular ω constante descrevendo
um cone; vide a figura. Se g é o módulo da ace-
(a) F~1 + F~2 = ~0
leração local da gravidade, o ângulo que o fio faz
(b) F~1 − F~2 = ~0 com a vertical é dado por:
(c) força resultante F~R = (k1 k2 )/(k1 + k2 )xı̂
(d) as forças F~1 e F~2 constituem o par ação e
reação
(e) força resultante F~R = (k1 + k2 )xı̂

g
(a) cosα =
ω2 L
g
(b) senα = 2
ω 2πL
ω2L
(c) tan α =
g
2πg
(d) senα = 2
ω L
7. Dois blocos A e B de massas mA e mB respecti- 2πLω 2
(e) cosα =
vamente estão sobre uma superfı́cie horizontal de g
atrito desprezı́vel. Uma força constante horizon- 10. Duas pedras são lançadas verticalmente simulta-
tal é aplicada sobre o bloco A que se encontra à neamente. A primeira com velocidade VA = v
direita de B. Se ~a é o vetor a aceleração com que para cima e a segunda com velocidade V2 = v para
os blocos se movem afirma-se que: baixo do topo de um prédio. Considerando que a
1
I) ~a = F~ . Terra é um referencial inercial e que a intensidade
ma + mb
da aceleração local da gravidade é g. Na situação
II) B exerce sobre A a força F~AB = mB~a.
em que as pedras ainda estão em movimento é
III) A exerce sobre B a força F~BA = mA~a.
mB possı́vel afirmar para o movimento relativo de A
IV) Em B atua a força F~B = F~ em relação B que:
mA + mB
Qual opção abaixo é correta?
(a) |~vAB | = v
(a) I e II estão corretas (b) ~vAB = ~0
(b) I e III estão corretas (c) ~aAB = ~0
(c) II e III estão corretas (d) ~aAB = ~g
(d) I e IV estão corretas (e) Nada se pode afirmar pois não se conhece
(e) nenhuma delas esta correta a altura do prédio

3
Seção 2. Questões discursivas (2×2,5 = 5,0 pontos)

1. Dois blocos de massas M e m estão em um dispositivo mostrado na figura abaixo. Na rampa existe atrito
entre, cujo coeficiente atrito cinético entre m e a sua superfı́cie é µc . A roldana assim como o fio que liga
os blocos podem ser considerados como ideais. Abandonados a partir do repouso verifica-se que o bloco M
desce. Considere conhecidos as massas M e m, o coeficiente de atrito cinético µc , o módulo da aceleração
da gravidade g e θ o ângulo de inclinação da rampa onde m se encontra.

a) Isole os blocos e represente por meio de um diagrama de corpo livre todas as forças que atuam
em cada um deles.
b) Determine o valor da aceleração com que os blocos se movem.
c) Qual o valor do coeficiente de atrito cinético µ∗c para que os blocos se movam com velocidade constante?
d) Determine o valor da tração que atua no fio, para o caso b).

2. Um bloco de massa m encontra-se sobre um plano horizontal. Ele é arremessado horizontalmente com
velocidade v de encontro com uma mola de constante elástica k que tem um anteparo de massa desprezı́vel
preso na sua extremidade. O outro extremo da mola esta fixo em uma parede. Ao colidir com a mola o bloco
prende-se ao anteparo e comprime a mola até parar momentaneamente de uma distância d desconhecida.

a) Determine a distância d.
b) Coloca-se agora um piso de coeficiente de atrito cinético(entre as superfı́cies do bloco e do piso)
desconhecido, µc , na região a − b onde a mola se encontra e arremessa-se o bloco com a mesma velocidade
v. Nesta situação o bloco comprime a mola de d′ , até parar momentaneamente. Determine o coeficiente de
atrito cinético µc em função de d, d′ , k, m e g(módulo da aceleração local da gravidade), supondo que a for
c ca de atrito é constante.

a região da mola b

4
Universidade Federal do Rio de Janeiro
Centro de Ciências Matemáticas e da Natureza
Instituto de Fı́sica
Primeira Prova de Fı́sica IA - 03/05/2012
Turma EQN - Respostas para provas hı́bridas

Gabarito das Questões objetivas

Versão A Versão B
Questão (a) (b) (c) (d) (e) Questão (a) (b) (c) (d) (e)
1 1
2 2
3 3
4 4
5 5
6 6
7 7
8 8
9 9
10 10

Versão C Versão D
Questão (a) (b) (c) (d) (e) Questão (a) (b) (c) (d) (e)
1 1
2 2
3 3
4 4
5 5
6 6
7 7
8 8
9 9
10 10

Observação: onde aparece uma tarja preta significa que esta questão foi anulada.
A anulação deveu-se a um erro de datilografia, cuja resposta correta é: F~R = −(k1 + k2 )xı̂.

1
Gabarito da Prova P1 da Disciplina Fı́sica IA - Turma EQN

I. QUESTÃO DISCURSIVA 1

A. Item (a): 0.25 pontos

Os diagramas de forças são mostrados na Fig.1. Para o corpo de massa m, escolhemos um sistema de coordenadas
tal que:

1. O eixo Ox seja paralelo ao plano inclinado e as coordenadas x aumentem da esquerda para a direita

2. O eixo Oy seja perpendicular ao plano inclinado e as coordenadas y aumentem de baixo para cima

E para o bloco de mass M , o sistema de coordenadas:

1. O eixo Ox seja horizontal e as coordenadas x aumentem da esquerda para a direita

2. O eixo Oy seja vertical e as coordenadas y aumentem de baixo para cima

B. Item (b) 1.25 pontos

Inicialmente, focamos no bloco localizado sobre o plano, com massa m. De forma a analisarmos as forças agindo
sobre ele, escolhemos um sistema de coordenadas tal que (veja o item anterior):

1. O eixo Ox seja paralelo ao plano inclinado e as coordenadas x aumentem da esquerda para a direita

2. O eixo Oy seja perpendicular ao plano inclinado e as coordenadas y aumentem de baixo para cima

Desta forma, as forças são especificadas por: a força normal (exercida pelo plano sobre o bloco)

~n = n̂ ; (1)

 

       

 

 

 

 

 

Figura 1: Diagramas de corpo livre para os blocos de massa m e M , conforme indicado em cada caso.
2

a tensão (ou tração, exercida pelo fio)

T~m = Tm ı̂ ; (2)

a força de atrito cinético (exercida pela superfı́cie do plano inclinado)

f~c = −µc |~n|ı̂ ; (3)

e finalmente, o seu peso (atração gravitacional da Terra)

P~m = −mg sin θı̂ − mg cos θ̂ . (4)

Como não há aceleração na direção perpendicular ao plano, a componente y da resultante das forças agindo sobre
P
o bloco de massa m é nula, Fy = 0. Desta forma, podemos obter o módulo da força normal
X
Fy = −mg cos θ + n = 0
(5)
n = mg cos θ .

Assim, a força de atrito cinético é dada por

f~c = −µc mg cos θı̂ . (6)

Tendo analisado as forças agindo sobre o bloco de massa m, fazemos o mesmo para o bloco de massa M . O sistema
de coordenadas neste caso é tal que (veja o item anterior)

1. O eixo Ox seja horizontal e as coordenadas x aumentem da esquerda para a direita

2. O eixo Oy seja vertical e as coordenadas y aumentem de baixo para cima

Assim, as forças agindo sobre o segundo bloco são: o seu peso (a força gravitacional exercida pela Terra)

P~M = −M g̂ (7)

e a tensão exercida pelo fio

T~M = TM ı̂ . (8)

O sistema constituı́do pelos dois blocos move-se com uma aceleração constante de módulo a, que queremos deter-
minar. Para o bloco de massa m, a aceleração pode ser escrita (levando-se em conta a escolha de eixos definidos em
relação ao plano inclinado)

~am = aı̂ , (9)

e para o bloco de massa M (no sistema de coordenadas definido em termos da superfı́cie da Terra)

~aM = −a̂ , (10)

Aplicando a segunda lei de Newton temos para o bloco m (note que as componentes y cancelam-se neste caso)

F~m = m~am
X

Tm,x + Pm,x + fc,x = mam,x (11)

Tm − mg sin θ − µc mg cos θ = ma .
3

De forma similar, para o bloco de massa M (neste caso são as componentes x que se cancelam)

F~m = m~am
X

TM,y + PM,y = M aM,y (12)

TM − M g = −M a .

Notamos que as Eqs. (11) e (12) envolvem as tensões desconhecidas, Tm e TM . Entretanto, como a roldana e o fio são
considerados ideais, a tensão não varia ao longo do fio, de modo que Tm = TM = T . As Eqs. (11) e (12) então podem
ser reescritas

T − mg sin θ − µc mg cos θ = ma
(13)
T − M g = −M a .

Explicitando T na segunda equação acima, obtemos T = M (g − a). Inserindo este resultado na primeira das equações
acima

M (g − a) − mg sin θ − µc mg cos θ = ma
(14)
(M + m)a = M g − mg (sin θ + µc cos θ) ,

e finalmente

[M − m (sin θ + µc cos θ)]


a=g
M +m

1. Solução Alternativa

Alternativamente, podemos também solucionar o problema se encararmos os dois blocos como um único com massa
total M +m. Ao fazermos isto, ignoramos a presença de forças internas no sistema: ou seja, ignoramos as tensões T~m e
T~M . Adicionalmente, devemos lidar com o fato de que as acelerações de cada um dos blocos possuem direções distintas.
Podemos contornar esta dificuldade se imaginarmos que o bloco composto de massa M + m situa-se inteiramente
sobre o plano inclinado, sob ação das forças agindo sobre o bloco de massa m (exceto T~m ) e, adicionalmente, sob
ação do peso p
~M do bloco de massa M , que imaginamos atuar na direção da aceleração do sistema sobre o plano
inclinado: ou seja, p
~M = +M gı̂.
Desta forma, aplicando a segunda lei de Newton

F~ 0 = (M + m)~a0
X

Pm,x + pM,x + fc,x = (M + m)a0 x (15)

− mg sin θ + M g − µc mg cos θ = (M + m)a .

Desta forma, podemos simplesmente isolar a, obtendo novamente

[M − m (sin θ + µc cos θ)]


a=g
M +m
4

C. Item (c): 0.5 pontos

Quando o coeficiente de atrito cinético assume o valor µ∗c , o sistema passa a se movimentar com velocidade constante,
estando portanto em equilı́brio. As seguintes relações devem ser obedecidas neste caso: para o bloco m

F~m = ~0
X

Tm,x + Pm,x + fc,x = 0 (16)

Tm − mg sin θ − µ∗c mg cos θ = 0 ,

e para o bloco M

F~m = ~0
X

TM,y + PM,y = 0 (17)

TM − M g = 0 .

Lembrando que o fio e a roldana são tomados como ideais, de forma que Tm = TM = T , temos as relações

T − mg sin θ − µ∗c mg cos θ = 0


(18)
T − Mg = 0 .

Da segunda temos T = M g, que inserimos na primeira obtendo

M g − mg sin θ − µ∗c mg cos θ = 0 . (19)

Isolando µ∗c , obtemos

M − m sin θ M
µ∗c = = − tan θ
m cos θ m cos θ

D. Item (d): 0.5 pontos

Para obtermos a tensão T basta que façamos uso das Eqs. (13), que reescrevemos aqui

T − mg sin θ − µc mg cos θ = ma
(20)
T − M g = −M a .

Já calculamos a aceleração do sistema no item (b), tendo obtido

M − m (sin θ + µc cos θ)
a=g (21)
M +m

Inserindo este último resultado na segunda das Eqs. (20) obtemos o resultado
 
M − m (sin θ + µc cos θ)
T = Mg 1 −
M +m
5

II. QUESTÃO DISCURSIVA 2

A. Item (a): 1.25 pontos

Escolhemos um sistema de coordenadas tal que:

1. O eixo Ox seja paralelo ao plano sobre o qual o bloco se movimenta e as coordenadas x aumentem da esquerda
para a direita

2. O eixo Oy seja perpendicular ao plano sobre o qual o bloco se movimenta e as coordenadas y aumentem de
baixo para cima

Empregando-se tal sistema de coordenadas, podemos expressar as grandezas vetoriais envolvidas da seguinte forma:
a velocidade do bloco (ele se movimenta da direita para a esquerda)

~v = −vı̂ , (22)

a força elástica variável

F~el = kxı̂ (23)

e o deslocamento infinitesimal

d~r = −dxı̂ . (24)

A resultante das forças atuando sobre o bloco, após este entrar em contato com a mola, é igual à força elástica
P~
exercida sobre ele pela mola, F = F~el = kxı̂, de forma que o trabalho total realizado sobre o bloco é dado por
Z 2 Z 2 Z x=−d  2 x=−d
~ x kd2
Wtot = Fel · d~r = (kxı̂) · (−dxı̂) = −k xdx = −k =− . (25)
1 1 x=0 2 x=0 2
Os limites de integração 1 e 2 referem-se, respectivamente, ao ponto em que o bloco entra em contato com a mola
(x = 0, a origem do nosso sistema de coordenadas) e ao ponto de compressão máxima da mola em x = −d.
O teorema trabalho-energia relaciona o trabalho total Wtot à variação da energia cinética do bloco:
mv22 mv12
Wtot = ∆K = K2 − K1 = − . (26)
2 2
Como o módulo da velocidade inicial v1 = v e da velocidade final v2 = 0, temos
mv2
Wtot = ∆K = − . (27)
2
Igualando-se as Eqs. (25) e (27) obtemos
kd2 mv2
− =−
2 2 (28)
2
mv
d2 = ,
k
e finalmente
r
m
d=v
k
6

B. Item (b): 1.25 pontos

A inclusão de uma força de atrito cinético f~c implica que o trabalho total não mais é dado pela Eq. (25), pois a
P~
resultante sobre o bloco agora é dada por F = F~el + f~c . Primeiramente, devemos calcular a força de atrito, dada
em termos gerais por (note que, com a nossa escolha de eixos coordenados e devido ao fato de que o bloco desloca-se
da direita para a esquerda, a força de atrito aponta da esquerda para a direita)

f~c = +µc |~n|ı̂ , (29)

onde µc é o coeficiente de atrito cinético, que queremos obter, e ~n é a força normal exercida sobre o bloco pela
superfı́cie horizontal. Para obtermos o módulo desta última, observamos que o sistema encontra-se em equilı́brio ao
longo da direção perpendicular ao plano, de forma que a soma das componentes y das forças atuando sobre o bloco
Fy = 0. As forças com componentes verticais atuando sobre o bloco são: o seu peso, P~ = −mg̂, e a
P
se cancelam:
P
força normal ~n = n̂. Desta forma, Fy = 0 implica que n = mg e portanto

f~c = +µc mgı̂ , (30)

O trabalho total realizado sobre o bloco enquanto este comprime a mola até uma distância d0 é então dado por
2 x=−d0 x=−d0
x2

kd02
Z  Z
Wtot = ~ ~
Fel + fc · d~r = − (kx + µc mg)dx = −k − µc mgd0 = − − µc mgd0 . (31)
1 x=0 2 x=0 2

Neste cálculo, notamos que a força de atrito cinético é constante e que a compressão máxima da mola é agora d0 .
Fazendo uso novamente do teorema trabalho-energia, Eq. (26), obtemos no presente caso (notamos que ∆K =
−mv2 /2 permanece válido)

Wtot = ∆K
kd02 mv2
− − µc mgd0 = − (32)
2 2
1 2 02
µc = (mv − kd )
2mgd0

Neste ponto, devemos notar que o enunciado pede para expressarmos µc apenas em termos de: d, d0 , k, m e g.
Assim, devemos eliminar v da equação acima, o que é possı́vel se fizermos uso do resultado do item anterior, Eq. (28):
mv2 = kd2 . Inserindo este último resultado na Eq. (32), chegamos a:

k
µc = (d2 − d02)
2mgd0

Notamos por fim que µc > 0 se d > d0 , como esperado.


UNIVERSIDADE FEDERAL DO RIO DE JANEIRO

INSTITUTO DE FÍSICA

FÍSICA I – 2012/1

PROMEIRA PROVA (P1) – 27/04/2012

VERSÃO: A

INSTRUÇÕES: LEIA COM CUIDADO!


1. Preencha CORRETA, LEGÍVEL E TOTALMENTE os campos em branco do cabeçalho do caderno
de resolução, fornecido em separado.
2. A prova constitui-se de duas partes:
• uma parte objetiva, perfazendo um total de 5,0 pontos, constituı́da por dez (10) questões objetivas
(de múltipla escolha), cada uma das quais valendo 0,5 ponto, sem penalização por questão errada.
• uma parte discursiva, perfazendo um total de 5,0 pontos, constituı́da por duas (2) questões discursivas
(ou argumentativas ou dissertativas), cada uma das quais valendo 2,5 pontos.
3. Acima da tabela de respostas das questões objetivas, na primeira página do caderno de resolução, INDI-
QUE CLARAMENTE A VERSÃO DA PROVA (A, B,. . . ).
4. O item considerado correto, em cada uma das questões objetivas, deve ser assinalado, A CANETA (de
tinta azul ou preta), na tabela de respostas correspondente do caderno de resolução
5. É vedado o uso de qualquer instrumento eletro-eletrônico (calculadora, celular, iPod, etc)
6. Seja organizado e claro.

Formulário

sen2 θ + cos2 θ = 1, sen2θ = 2senθcosθ


sen(α ± θ) = senαcosθ ± cosαsenθ, cos(α ± θ) = cosαcosθ ∓ senαsenθ
d n xn+1
Z
x = nxn−1 xn dx = (n 6= −1)
dx n+1
d d
senax = acosax, cosax = −asenax
dx dx

a b c
Lei dos senos: = =
senα senβ senγ
Lei dos cossenos: a2 = b2 + c2 − 2bc cosα

1
Seção 1. Múltipla escolha (10×0,5 = 5,0 pontos)

1. Um carro sobe uma ladeira em linha reta com 4. Uma partı́cula descreve um movimento circular,
velocidade constante em relação a um referencial com velocidade de módulo constante e igual a V .
fixo ao chão da ladeira; considere a Terra como um Num intervalo de tempo em que percorre 1/4 da
referencial inercial. Uma pessoa num helicóptero circunferência, o módulo de seu vetor velocidade
observa que a velocidade do carro em relação a média é igual a
ele é zero. Qual destas afirmações em relação ao √
2 2
referencial do helicóptero é verdadeira? (a) π V
1
(b) 4 V
(a) A aceleração do helicóptero em relação ao
referencial fixo do chão é diferente de zero (c) 2V
π
(b) O referencial do helicóptero é inercial (d) 2 V

2
(c) As leis de Newton não são aplicáveis nesse (e) 2 V
referencial
5. O vetor posição de um corpo em função do tempo,
(d) Como a velocidade do helicóptero não é t, é ~r(t) = ~r0 + ~v0 t + 12 ~at2 , onde ~r0 (posição ini-
dada não é possı́vel saber se o referencial cial), ~v0 (velocidade inicial) e ~a(aceleração) são ve-
do helicópetero é inercial ou não. tores constantes. Afirma-se que: I) se ~v0 e ~a tem
(e) Nenhuma das respostas anteriores a mesma direção o movimento é retilı́neo. II) a
trajetória é um arco de parábola para qualquer ~a.
III) esta equação descreve um movimento circular
2. Um corpo de massa m ao ser largado de uma al- uniforme. IV) se ~a = 0 o movimento é retilı́neo e
tura H, a partir do repouso, num plano inclinado uniforme. A resposta correta é:
de um ângulo θ em relação a horizontal, atinge (a) I e II estão corretas
uma velocidade de módulo v ao chegar na base do
(b) I e III estão corretas
plano. Há atrito entre o corpo e a superfı́cie do
plano sendo o coeficiente de atrito cinético igual a (c) II e III estão corretas
µ. O mesmo corpo quando largado sob as mesmas (d) I e IV estão corretas
condições em outro plano de mesma inclinação,
(e) nenhuma delas esta correta
mas sem atrito, atinge a base do plano com uma
velocidade cujo módulo é o dobro da situação an- 6. Sobre um corpo atuam duas forças bidimensio-
terior. O valor de µ é igual a: nais F~1 e F~2 e a aceleração do corpo é nula. Qual
2 afirmação é verdadeira?
(a) 3 tan θ
(b) 1
tan θ (a) F~1 e F~2 constituem o par ação e reação
2
(c) 3
tan θ (b) A força F~1 é igual à força F~2
4
(d) tan θ (c) Se em relação a um sistema de referência
1 a componente de F~1 em relação a ı̂ é posi-
(e) cos θ
2 tiva e a componente de F~1 em relação a ̂
é negativa então nesse mesmo referencial
a componente de F~2 em relação a ı̂ é ne-
3. Ao ser disparado verticalmente um projétil atinge gativa e a componente de F~2 em relação
uma altura máxima h. Se o mesmo projétil é a ̂ é positiva
disparado numa direção que faz um ângulo θ
(d) A força F~1 tem o mesmo módulo que a
(θ < π/2) com a horizontal, a altura máxima atin-
gida é igual a: força F~2 pois as componentes de F~1 e
F~2 são iguais em qualquer sistema de re-
(a) h cos θ ferência
(b) (h/2) tan θ (e) A força F~1 tem o mesmo módulo que a
(c) (h/2) sen θ força F~2 mas não existe nehuma relação
entre as componentes destas forças pois
(d) h sen 2θ
estas dependem do sistema de referência
(e) h sen2 θ escolhido

2
7. Uma única força conservativa atua em uma 8. Um corpo de massa m é visto descendo um plano
partı́cula paralela ao eixo horixontal OX de um com velocidade constante; só há o plano e o corpo.
sistema de coordenadas. A energia potencial Sabendo-se que o ângulo de inclinação do plano
desta força é dada pela figura abaixo. Qual opção em relação à horizontal é igual a θ, pode-se afir-
representa corretamente os vetores(direção, inten- mar que a resultante das forças que o plano incli-
sidade e sentido) das forças que atuam sobre a nado exerce sobre o corpo tem módulo igual a
partı́cula nos pontos A e B respectivamente.
(a) m g senθ
(b) mg
(c) m g cosθ
(d) m g (1 − senθ)
(e) m g (1 + cosθ)

A 9. Considere um pêndulo constituido de um fio de


massa desprezı́vel de comprimento L e um corpo
de massa m. Preso ao teto o fio é esticado hori-
B zontalmente e o pêndulo abandonado a partir do
repouso. Para que o pêndulo movimente-se sem
o fio arrebentar, a intensidade de tração mı́nima
que o fio deve suportar é:
(a) mg
1) 2) 3) 4) 5) √
(b) 2mg
3
(c) 2 mg
(d) 3mg
(a) diagrama 4 (e) 4mg
(b) diagrama 1
10. Um observador parado no solo vê um pacote
(c) diagrama 5 caindo de um avião, com uma velocidade de
(d) diagrama 2 módulo igual a v1 , mas que faz um certo ângulo
com a vertical; considere a Terra um referêncial
(e) diagrama 3
inercial. Simultaneamente o piloto do avião, que
voa na horizontal com velocidade constante, vê o
mesmo pacote caindo na vertical, com velocidade
de módulo igual a v2 . O módulo da velocidade do
avião em relação ao observador no solo é
p
(a) v12 − v22
p
(b) v12 + v22
(c) v2 + v1
(d) v2 − v1
(e) v1 − v2

3
Seção 2. Questões discursivas (2×2,5 = 5,0 pontos)

1. Um bloco de massa M está sob a ação de uma força horizontal


F~ constante sobre uma superfı́cie horizontal sem atrito. Sobre
ele há um bloco de massa m, preso à esquerda por um fio ideal.
Este fio passa por uma roldana ideal que encontra-se fixa a
uma parede vertical, e o conecta ao bloco de massa M . Os
segmentos do fio são paralelos ao plano horizontal; vide a figura
ao lado. Suponha que haja atrito entre os blocos em contato
entre si. Considere como conhecidos os valores dos coeficientes
de atrito estático µe e cinético µc , as massas m e M e o módulo
da aceleração da gravidade g.

a) Isole os blocos e represente por meio de um diagrama


de corpo livre todas as forças que atuam em cada um deles.
b) Suponha inicialmente que os blocos estejam em repouso.
Determine o valor máximo do módulo de F~ , Fmax para que o
sistema permaneça em repouso.
c) Considere agora que F~ , cujo módulo é igual a F ′ , seja capaz
de colocar os blocos em movimento com aceleração constante.
Para o intervalo de tempo no qual os blocos permanecem em
contato entre si, determine os vetores aceleração de cada bloco,
em função de F ′ , µc , m, M e g.
d) Determine o módulo da tração do fio, para o caso do item
anterior c).

2. Um bloco de massa m e dimensões desprezı́veis encontra-se


sobre um plano horizontal. Ele comprime uma mola de cons-
tante elástica k no ponto A de uma distância d em relação à
posição O; conforme mostra a figura. Liberado neste ponto
a partir do repouso ele percorre o trajeto A-O-B perdendo
contato com a mola no ponto O, onde a mola está relaxada.
Somente entre os pontos O e B, separados de uma distância C
desconhecida há atrito. O coeficiente de atrito cinético en-
tre as superfı́cies do bloco e do plano é µc na região O-B. H
Após o ponto B há uma rampa sem atrito. A partir do
ponto C, final da rampa, a superfı́cie é horizontal e tem uma
altura H em relação à horizontal do trecho A-O-B; vide a figura. A O B

a) Determine a velocidade do bloco no ponto O;


b) Determine a distância D entre os pontos O e B, supondo
que a velocidade do bloco em B é nula;
c) Determine a compressão mı́nima, xmin , da mola necessária
para que o bloco atinja o ponto C no topo da rampa.

4
Universidade Federal do Rio de Janeiro
Centro de Ciências Matemáticas e da Natureza
Instituto de Fı́sica
Primeira Prova de Fı́sica IA - 27/04/2012
Respostas para provas hı́bridas

Gabarito das Questões objetivas

Versão A Versão B
Questão (a) (b) (c) (d) (e) Questão (a) (b) (c) (d) (e)
1 1
2 2
3 3
4 4
5 5
6 6
7 7
8 8
9 9
10 10

Versão C Versão D
Questão (a) (b) (c) (d) (e) Questão (a) (b) (c) (d) (e)
1 1
2 2
3 3
4 4
5 5
6 6
7 7
8 8
9 9
10 10

1
Questão discursiva 1
a) valor=1.0 pontos
Diagrama de forças:

As forças P~m e P~M são as forças peso, T~ a tração do fio, N


~m e N
~ M as forças normais, F~ 0
at
é a força de atrito que age sobre M e F~at a sua reação agindo sobre m, F~ a força aplicada
sobre M e N ~ 0 a reação de N
~ m.
m

b) valor=0.5 pontos
Na situação estática F~at ≡ F~e e na dinâmica F~at ≡ F~d . Os módulos das tensões nos
extremos da corda têm igual módulo pois a corda e a roldana têm massas desprezı́veis e são
consideradas ideais. Vamos considerar que estamos na iminência do movimento dos blocos
(max)
nesse caso F~at = F~e = µe Nm ı̂. Nesta situação o módulo da força aplicada |F~ | = Fmax
é a força necessária para estarmos no limiar do movimento (ainda a velocidade é nula
mas qualquer valor levemente superior a Fmax fará o sistema se movimentar). Portanto a
segunda lei de Newton para o bloco de massa m é:
~ m + F~at(max) + T~ + P~m = m~a = ~0
N (1)

que escrita em componentes fica,

ı̂) Fe(max) − T = 0 ⇒ T = Fe(max) = µe Nm (2)

e
̂) Nm − Pm = 0 ⇒ Nm = mg . (3)
A segunda lei de Newton para o bloco de massa M é:
~ M + F~at(max)0 + T~ + P~M + N
N ~ 0 + F~max = M~a = ~0, (4)
m

2
que escrita em componentes fica,

ı̂) Fmax − Fe(max)0 − T = 0 ⇒ Fmax = µe mg + T = 2µe mg , (5)

e
0
̂) NM − Nm − Pm = 0 ⇒ NM = (m + M)g , (6)

Note que N ~m e N~ 0 assim como F~at(max) e F~~at(max)0 constituem os respectivos pares ação e
m
reação. Ao substituimos o valor de Nm achado na Eq. (3) na Eq. (2) obtemos o valor de
T . Finalmente da Eq. (5) obtemos o valor

Fmax = 2µe mg

c) valor=0.5 pontos
Na situação dinâmica, para o bloco de massa m temos que F~at = Fc ı̂ = µc Nm ı̂ = µc mg ı̂.
Então a componente na direção ı̂ da segunda lei de Newton da Eq.(1) é,

ı̂) Fc − T = ma1 , ⇒ µc mg − T = ma1 . (7)

Na direção ̂ a equação é análoga a equação (3).


Para o bloco de massa M a componente ı̂ da segunda lei de Newton na Eq.(4) é:

ı̂) F 0 − Fc − T = Ma2 ⇒ , F 0 − µc mg − T = Ma2 . (8)

A componente ̂ é análoga à equação (6).


Como o fio é inextensı́vel ~a2 = a ı̂ = −~a1 (a > 0), ou seja, enquanto o bloco de massa
M acelera para a direita o bloco de massa m acelera para a esquerda. Assim, finalmente
temos o sistema de equações:

 µc mg − T = −ma

F 0 − µc mg − T = Ma

onde as incógnitas são T e a. Resolvendo o sistema temos que:

T = µd mg + (F 0 − 2µc mg)/(m + M) (9)

e a = (F − 2µc mg)/(m + M). Portanto:

~a1 = −(F 0 − 2µc mg)/(m + M) ı̂ , (10)

e a aceleração do bloco de massa M é:

~a2 = (F 0 − 2µc mg)/(m + M) ı̂ . (11)

d) valor= 0.5 pontos


A valor da tração que atua no fio é:
m
T = (F 0 − µc (M − m)g). (12)
m+M

3
Questão discursiva 2

a) valor=1.0 pontos
A energia mecânica conserva-se no trecho A-O pois não há atrito, a força de reação
normal não realiza trabalho e o peso e a força da mola são conservativas. Escolhendo
o trecho A-O-B como o “zero” da energia potencial gravitacional, temos que a energia
mecânica no ponto A é dada unicamente pela energia potencial elástica, pois é solto do
repouso, assim: EA = kd2 /2, no ponto O, a energia mecânica é dada unicamente pela
energia cinética, pois a mola encontra-se relaxada, logo EO = mv02/2. Pela conservação
da energia mecânica encontramos a velocidade no ponto O

r
kd2 mvO2 k
EA = EO ⇒ = → vO = d.
2 2 m

b) valor=1.0 pontos
Temos duas maneiras de resolver o problema, e em ambas, é necessário calcular o
trabalho da força de atrito no trecho O-B:

WF~at = F~at · OB
~ = |F~at||OB|
~ cos π = −|F~at||OB|
~ = −µc ND = −µc mgD.

Maneira 1: Usemos que a variação da energia mecânica é igual ao trabalho das forças
não-conservativas, então
=0 −µc mgD
z}|{ z }| {
∆K = KB − KO = WF~at
|{z}
mv 2
O
2

mvO2 vO2 kd2


∴ − = −µc mgD ⇒ D= → D= ,
2 2µc g 2µc mg
onde usamos o resultado da alı́nea (a) na última passagem.

Maneira 2: Neste caso utilizemos diretamente o teorema do trabalho energia cinética


=0 −µc mgD
z}|{ z }| { mv 2
∆K = KB − KO = WF~at ⇒ − O = −µc mgD
|{z} 2
mv 2
O
2

vO2 kd2
∴ D= → D= ,
2µc g 2µc mg
aqui, o resultado da alı́nea (a) na última passagem também foi usado.

4
c) valor=0.5 pontos
A compressão mı́nima da mola, xmin é tal que o bloco atinge o ponto C com velocidade
nula. Usando que a variação da energia mecânica é igual ao trabalho das forças não-
conservativas temos:
kx2
min
2 −µc mgD
z}|{ z }| {
∆E = EC − EA = WF~at
|{z}
mgH

kx2min
∴ mgH − = −µc mgD
2  
kx2min kd2
= mgH + µc mg
2 2µc mg
r
2mgH
xmin = + d2 .
k

5
UNIVERSIDADE FEDERAL DO RIO DE JANEIRO
INSTITUTO DE FÍSICA
FÍSICA I – 2012/2
PRIMEIRA PROVA(P1) – 06/12/2012– Turma EQN
VERSÃO: A

Nas questões em que for necessário, considere que g é o módulo da aceleração de gravidade.

Seção 1. Múltipla escolha (10×0,5 = 5,0 pontos)

1. Uma pedra é lançada verticalmente, do solo, 3. Um paraquedista após saltar de uma altura
com velocidade de módulo v0 . Verifica-se que muito alta h, verifica com um medidor de velo-
ao retornar ao solo o módulo de sua veloci- cidades, que num certo intervalo de tempo ∆t,
dade é vf , onde vf < v0 . Durante o movimento a sua velocidade ficou constante em módulo.
da pedra, considere somente a ação das forças Desprezando a força de empuxo do ar e consi-
peso P~ , e a força de resistência do ar f~ar . Para derando que neste intervalo de tempo a queda
o trabalho da força de resistência do ar no per- foi vertical, onde as forças que atuaram sobre
curso ida e volta, a resposta correta abaixo é ele na queda foram a força peso P~ e a força de
dada por: resistência do F~ar . No intervalo de tempo ∆t
a opção correspondente a esta situação é:
(a) Wfar > 0, pois v0 > vf .
(b) Wfar = 0, pois a posição inicial coincide (a) |P~ | > |F~ar |;
com a posição final da pedra. (b) P~ + F~ar = ~0;
(c) Wfar > 0, pois v0 < vf . (c) |P~ | < |F~ar |;
(d) Wfar < 0, pois v0 > vf . (d) P~ − F~ar = ~0;
(e) nenhuma resposta anterior está correta. (e) nenhuma opção acima está correta.

2. Duas molas ideais de constantes elásticas k1 e


k2 são ligadas em série. A extremidade livre
de uma delas ṕresa a um bloco e a extremi-
dade livre da outra mola é presa a uma parede
fixa. O sistema formado pelas molas e o corpo
é comprimido de uma distância d em relação 4. Uma partı́cula move-se no plano horizontal
à posição de equilı́brio; como mostra a figura sem atrito com movimento circular e uniforme.
abaixo. O módulo da força resultante que atua O raio da trajetória é R e a velocidade angu-
sobre o bloco, imediatamente após ele ser libe- lar do movimento igual a ω. O produto escalar
rado é dada por: entre o vetor velocidade da partı́cula, ~v (t), e o
seu vetor posição, ~r(t), em relação ao centro
k1 k2 da trajetória para um instante qualquer é:
(a) d.
k1 + k2
(a) nulo;
(b) k1 d.
k22 (b) π/4;
(c) d. (c) π/2;
k1
(d) (k1 + k2 )d. (d) π;
(e) (k1 − k2 )d. (e) nenhuma das respostas anteriores.

1
5. Dois blocos idênticos e de mesma massa m, 6. Um dipositivo é constituido de dois arames (1)
(A) e (B), deslizam sobre duas superfı́cies ho- e (2) ligando os pontos A e B. O arame (1)
rizontais, sem atrito, com velocidade constante está esticado formando um segmnento de reta
e de mesmo módulo v. As superfı́cies estão se- e o (2) apresenta uma curvatura. Duas contas
paradas de uma altura h; como mostra a fi- idênticas e de mesma massa são soltas a partir
gura abaixo. Para a energia mecânica E, po- do repouso no ponto A. Elas deslizam pelos
tencial U e cinética K de cada bloco, a opção arames com atrito desprezı́vel e chegam em B,
correta que relaciona as suas respectivas gran- com velocidades de módulo v1 e v2 respectiva-
dezas para os blocos é: (na superfı́cie onde o mente, que correspondem aos arames por onde
bloco (B) se desloca, Ug = 0). deslizaram. A resposta correta abaixo é:

(a) Ug,A > Ug,B , EA = EB e KA = KB


(a) v1 > v2 pois o arame (1) sempre está
(b) Ug,A = Ug,B , EA = EB e KA = KB
mais alto que o arame (2).
(c) Ug,A > Ug,B , EA = EB e KA < KB
(b) v1 = v2 .
(d) Ug,A < Ug,B , EA = EB e KA = KB
(c) v1 < v2 pois a distância percorrida pela
(e) Ug,A > Ug,B , EA > EB e KA = KB conta ao longo do caminho (2) é maior
que a distância percorrida no caminho
(1).
(d) O trabalho da força peso da conta que
passa por (1) de A até B , é maior que
o trabalho da força peso da conta que
passa por (2) de A até B .
(e) todas as opções acima estão erradas.

7. Um bloco de gelo desliza sobre uma mesa ho-


rizontal sem atrito com velocidade vetorial ~v
constante. Qual resposta é correta?
(a) Ele está em equlı́brio instável pois o
módulo da sua velocidade é diferente de
zero
(b) Não existe força alguma atuando sobre
ele
(c) Ele está em equlı́brio estável
(d) Ele não está em equilı́brio
(e) Nenhuma das respostas anteriores estão
corretas

2
8. Uma pequena esfera é abandonada a partir do 10. Considere as afirmações: I) Um corpo sob
repouso, da borda de um poço semicircular de a ação de uma força F~ , se move sempre na
raio R. Ao passar pelo ponto mais baixo do direção e sentido desta força. II) No movi-
poço, a sua velocidade tem módulo igual v, a mento retilı́neo uniformemente variado a ace-
força que o fundo do poço exerce sobre a es- leração tem sempre o mesmo sentido da ve-
~ e o seu peso é P~ ; não há atrito entre
fera é N locidade. III) O vetor posição de um corpo
a esfera e a superfı́cie do poço e a resistência num instante t qualquer é dado por, ~r(t) =
do ar é desprezı́vel. A opção correta é: ~r0 + ~v0 t + 21 ~at2 , onde ~r0 é o vetor posição ini-
(a) P~ + N ~ = ~0. cial, ~v0 é o vetor velocidade inicial e ~a o vetor
aceleração onde todos são vetores constantes
(b) N~ − P~ = ~0.
em módulo direção e sentido, neste caso a tra-
(c) |N~ | − |P~ | = mv 2 /R. jetória do corpo é necessariamente parabólica.
~ | = mv 2 /R. Estão corretas as afirmações:
(d) |N
(e) Nenhuma das afirmações anteriores. (a) I) e II)
(b) I) e III)
9. Uma partı́cula de massa m está dentro de um
funil de vidro e percorre o seu interior com um (c) II) e III)
movimento circular uniforme de raio R. Não (d) somente I)
há atrito entre a parede do funil e a partı́cula.
O ângulo que a parede do funil faz com o seu (e) nenhuma das afirmações está correta
eixo de simetria é igual a θ; como mostra a fi-
gura. Desprezando a presença do ar o módulo
da velocidade é dado por:

p
(a) gR.
p
(b) gR/senθ.
p
(c) gRcosθ.
p
(d) gR/tanθ.
p
(e) gRsenθ.

3
Seção 2. Questões discursivas (2×2,5 = 5,0 pontos)

1. Um pequeno bloco de massa m, após ser empurrado por uma mola de constante elástica k, com-
primida de ∆x, desliza sobre uma superfı́cie horizontal. Depois de percorrer uma certa distância, o
bloco começa a subir uma rampa vertical semicircular de raio R. Uma vez na rampa o bloco atinge
uma posição, cuja direção faz um ângulo θ com a vertical, onde para momentaneamente e volta des-
cendo a rampa; como mostra a figura. Desprezando-se o atrito em todo o percurso e considerando a
mola ideal calcule:
a) o ângulo θ alcançado;
b) o valor da módulo da força normal, ao atingir o ângulo θ;
c) a velocidade máxima (módulo) do bloco durante o seu movimento e onde ela ocorre; justifique a
sua resposta.

2. Dois blocos estão ligados por meio de um cabo ideal e constituem um dispositivo como indicado na
figura abaixo. O bloco colocado sobre a mesa horizontal tem massa m1 e bloco suspenso pela roldana
tem massa m2 . Não há atrito entre as superfı́cies do bloco de massa m1 e a mesa horizontal e as
roldanas são consideradas como ideais.
a) Isole cada bloco e num diagrama represente TODAS as forças que atuam em cada bloco.
b) Calcule o módulo da aceleração com que cada bloco se move blocos se movem.
c) Determine o valor da tração do fio que age sobre o bloco de massa m1 .

4
Universidade Federal do Rio de Janeiro
Centro de Ciências Matemáticas e da Natureza
Instituto de Fı́sica
Primeira Prova de Fı́sica IA - 18/12/2012 - Turma EQN
Respostas para provas hı́bridas

Gabarito das Questões objetivas

Versão A Versão B
Questão (a) (b) (c) (d) (e) Questão (a) (b) (c) (d) (e)
1 1
2 2
3 3
4 4
5 5
6 6
7 7
8 8
9 9
10 10

Versão C Versão D
Questão (a) (b) (c) (d) (e) Questão (a) (b) (c) (d) (e)
1 1
2 2
3 3
4 4
5 5
6 6
7 7
8 8
9 9
10 10

1
4. Uma partı́cula move-se sob a ação de uma 6. Duas partı́culas A e B movem-se no plano hori-
UNIVERSIDADE FEDERAL DO RIO DE JANEIRO única força conservativa F , no percurso fe- zontal XOY , respectivamente com velocidades
chado A → B → C → A, como indica a figura vA = vA ı̂ e vB = −vB ĵ constantes. A direção
INSTITUTO DE FÍSICA abaixo. Afirma-se que para o trabalho desta do movimento de A é perpendicular ao eixo
FÍSICA I – 2012/2 força nos trechos AB, BC e CA: I) WA→B + OY e a direção do movimento de B é perpen-
WB→C + WC→A = 0, II) WA→C = −WC→A , dicular ao eixo OX; vide a figura abaixo. Seja
PRIMEIRA PROVA(P1) – 07/12/2012
III) WA→C = WA→B + WB→C , IV) como a vAB a velocidade de A com relação a B a opção
VERSÃO: A força F é conservativa o trabalho desta força é correta é:
sempre positivo em qualquer trecho e sentido
do percurso. A opção abaixo correta para as
afirmativas I), II) III) e IV) é:
Nas questões em que for necessário, considere que g é o módulo da aceleração de gravidade.

Seção 1. Múltipla escolha (10×0,5 = 5,0 pontos)

1. Atira-se duas vezes uma bola, verticalmente, 3. Uma partı́cula desloca-se ao longo do eixo x (a) vAB = 0.

de uma mesma altura em relação ao solo; des- sob a ação de uma força conservativa F , cor- (b) |vAB | = vA2 + vB2 .
preze o efeito do ar. Na primeira vez (situação respondente a um potencial U (x), dado pela (a) somente I) e II) estão corretas;
A) a velocidade inicial tem sentido para cima e figura abaixo. Para este potencial entre as (c) vAB = vA ı̂ − vB ĵ.
(b) somente II) e IV) estão corretas;
no segundo caso (situação B) o sentido é para opções abaixo a única incorreta é: (d) |vAB | = |vA |, pois vA é perpendicular a
(c) somente I) e III) estão corretas;
baixo. Nos dois casos as velocidade iniciais vB .
tem o mesmo módulo. Na situação A, a bola (d) somente I), II) e III) estão corretas;
(e) todas as opções acima estão erradas.
chega ao solo com velocidade vA e na situação (e) todas estão corretas.
B, com velocidade vB . É correto afirmar que:
(a) |vA | > |vB |
(b) |vA | < |vB |
(c) |vA | = |vB |
(d) Não é possı́vel determinar a relação
entre |vA | e |vB |, pois a altura de
lançamento não é conhecida.
(a) na posição xB a força sobre a partı́cula 7. Um disco horizontal gira em torno de um
(e) Não é possı́vel determinar a relação en- é nula; eixo vertical que passa pelo seu centro, com
tre |vA | e |vB |, pois ambas dependem velocidade angular constante. Coloca-se um
(b) na posição xD , tem-se a condição de
das velocidades iniciais. corpo de pequenas dimensões e massa m so-
equilı́brio estável;
2. Um pequeno bloco de massa m desliza sobre bre o disco a uma distância D do seu centro.
(c) no deslocamento do corpo de xA para Verifica-se que o atrito entre o corpo e a su-
um plano de inclinação 0 < θ < π/2 com a ho- 5. Um pequeno bloco de massa m está sobre uma
xC o trabalho realizado pela força F é perfı́cie do disco é suficiente para que ele per-
rizontal, sem atrito. Sobre ele atuam: a força superfı́cie horizontal sem atrito ligado a uma
positivo; maneça na mesma posição do disco em que foi
normal N  exercida pelo plano e o peso P . A mola de constante elástica k cuja outra extre-
opção correta abaixo é: (d) o sentido da força F na posição xE é midade é fixa em uma parede, como mostra a colocado. Ou seja ele não desliza sobre o disco.
negativo; figura. O bloco é deslocado para x = A, a par- O disco dá uma volta completa no intervalo
(a) N
 = P cosθ. de tempo T . Sabendo-se que o coeficiente de
(e) na posição xC a força F é nula. tir da posição de equilı́brio, xeq = 0 e liberado
(b) O módulo da aceleração do bloco é igual a partir do repouso. Para qualquer instante atrito estático entre o corpo e a mesa é μ, o
a g. posterior, o módulo da sua velocidade é dado trabalho realizado pela força de atrito numa
por: volta completa é igual a:
(c) O módulo da força resultante sobre o 
bloco é: Psenθ (a) v = k/m(A2 − x2 )
 (a) zero
(d) O módulo da força normal é: Psenθ (b) v = k/m(A2 + x2 ), (b) −2πμmgD

(e) A aceleração do bloco tem sempre o (c) v = 2kx/m(A − x) (c) −4π 2 μmD/T 2
mesmo sentido da velocidade, indepen- 
(d) v = k/mx (d) 4π 2 μmD/T 2
dente de qual seja a sua velocidade ini- 
cial. (e) v = k/mA (e) −μmD/T 2

1 2
8. Considere as seguintes afirmações sobre os ve- 10. Nas figuras abaixo, a parábola representa a Seção 2. Questões discursivas (2×2,5 = 5,0 pontos)
tores velocidade e aceleração de um corpo em trajetória de um lançamento oblı́quo para θ0 =
movimento: I) A velocidade pode ser zero e a 0, de um projétil nas proximidades da su-
1. Um conhecido brinquedo consiste em uma pista contendo um “loop” circular de raio R e um trecho
aceleração ser diferente de zero. II) O módulo perfı́cie da Terra. No ponto mais alto da tra-
horizontal OA. No ponto O, localiza-se um lançador, constituido de uma mola ideal relaxada, de
do vetor velocidade pode ser constante, com o jetória do projétil, o diagrama que melhor re-
constante elástica k. Um carrinho (representado por um pequeno bloco na figura abaixo) de massa m
vetor velocidade mudando com o tempo. III) presenta os vetores velocidade v e aceleração,
é colocado inicialmente em O. Empurra-se o carrinho, comprimindo-se a mola de Δx até o ponto C.
O vetor velocidade pode ser constante mas seu a, neste ponto é: (despreze o efeito da re-
Neste ponto o carrinho é liberado a partir do repouso perfazendo o percurso C − O − A − B − A − D,
módulo variar com o tempo. IV) O vetor ve- sistência do ar)
perdendo contato com a mola no ponto O e percorrendo todo o ”‘loop”sem perder contato com a
locidade pode mudar de sentido com o tempo
sua superfı́cie. Desprezando-se o atrito em todo o percurso e considerando como dados do problema:
mesmo que o vetor aceleração permaneça cons-
R, k, Δx, m e g, calcule:
tante. São verdadeiras as afirmações:
a) o módulo da velocidade do carrinho ao passar por A;
(a) Todas as afirmações b) o módulo da velocidade do carrinho ao passar por B;
(b) I, II e III c) represente em um diagrama as forças que atuam sobre o carrinho ao passar por B e determine o
módulo da força de contato, |N
 |, neste ponto;
(c) II e III
(a) (II) d) a compressão mı́nima da mola ΔXmin para que, ao passar por B, o carrinho esteja na iminência
(d) I, II e IV de perder contato com o “loop”.
(b) (I)
(e) Nenhuma das afirmações anteriores.
(c) (III)
9. Uma partı́cula de massa m pendurada por um
fio ideal de comprimento  é abandonada de (d) (IV)
um ângulo θ0 a partir do repouso com o fio (e) nenhum dos diagramas
totalmente estendido, como mostra a figura
abaixo. Sejam a tração no fio T e o peso P
as forças que atuam na partı́cula e despreze
a resistência do ar. Qual das afirmações está
correta?

2. Um projétil é lançado com velocidade v0 formando um ângulo α com a horizontal. O ponto de
lançamento está localizado a uma altura h acima do solo. A figura mostra o sistema de referência
XOY fixo, que está localizado no solo e tem o eixo vertical OY alinhado verticalmente com o ponto
(a) |P | varia com o ângulo θ. de lançamento. De acordo com este referencial, pressupondo que a resistência do ar é desprezı́vel e
(b) 0 módulo da aceleração da partı́cula |a| que a Terra é um referencial inercial:
é constante a) escreva os vetores posição r(t) e velocidade v (t), como funções do tempo t, usando os unitários ı̂
(c) no ponto mais baixo da trajetéria, e ĵ dos eixos OX e OY , respectivamente, indicados na figura;
|T | = mv 2 / onde v é o módulo da ve- b) calcule o tempo que o projétil leva para atingir a altura máxima;
locidade neste ponto c) calcule o tempo de voo do projétil.
(d) para θ = ±θ0 , aceleração é nula d) determine, quando o projétil toca o solo, o módulo da sua velocidade vS .

(e) No ponto mais baixo da trajetória |T | >


|P |.

3 4
Universidade Federal do Rio de Janeiro
Centro de Ciências Matemáticas e da Natureza
Instituto de Fı́sica
Primeira Prova de Fı́sica IA - 7/12/2012
Respostas para provas hı́bridas

Gabarito das Questões objetivas (valor=5.0 pontos)

Versão A Versão B
Questão (a) (b) (c) (d) (e) Questão (a) (b) (c) (d) (e)
1 1
2 2
3 3
4 4
5 5
6 6
7 7
8 8
9 9
10 10

Versão C Versão D
Questão (a) (b) (c) (d) (e) Questão (a) (b) (c) (d) (e)
1 1
2 2
3 3
4 4
5 5
6 6
7 7
8 8
9 9
10 10

1
Questão discursiva 1 (valor=2.5 pontos)
a) valor=0.5 ponto
Escolhendo o zero do potencial gravitacional no solo, a energia mecânica do ponto C será:

k (∆x)2
EC = KC + UgC + UelC = . (1)
2
Já a energia mecânica no ponto A será:

mvA2
EA = KA + UgA = . (2)
2
A energia mecânica conserva-se, pois não há forças dissipativas atuando no sistema,
então:
r
mvA2 k (∆x)2 k
EA = EC =⇒ = ⇒ vA = ∆x. (3)
2 2 m

b) valor=0.5 ponto
A energia mecânica no ponto B será:

mvB2
EB = KB + UgB = + 2mgR. (4)
2
A energia mecânica conserva-se, então:
2
r
mvB2 k (∆x) k
EB = EC =⇒ + 2mgR = =⇒ vB = (∆x)2 − 4gR. (5)
2 2 m

c) valor=1.0 ponto
O diagrama de forças no ponto B é dado pela figura abaixo onde ûr = r̂.

As forças presentes no ponto B, são a normal, N ~ = −|N|r̂


~ e o peso, P~ = −mgr̂.
Na direção radial, a força resultante é a força radial dirigida para o centro (força
“centrı́peta”), F~c = − (mvB2 /R) r̂.

2
Assim na direção radial:

~ | + mg = mvB2 ~ = mvB2
|N =⇒ |N| − mg. (6)
R R
~ em função dos dados do problema é obtido, substituindo a Eq.
O resultado final de |N|
(5) na Eq. (6), encontra-se assim:

~ = k
|N| (∆x)2 − 5mg. (7)
R

d) valor=0,5 ponto
O carrinho completa o “loop” quando não perde o contato com o mesmo (|N| ~ =6 0). O
caso limite ocorre quando o carrinho está na iminência de perder o contato no ponto B,
~ B | = 0.
ou seja, |N

Maneira 1:
Nesta situação a vB é mı́nima (pois acima do valor mı́nimo de vb , o carrinho consegue
completar o “loop”) e o lançador deve ser comprimido de ∆Xmin . Usando que no ponto
~ = 0 na Eq. (7), temos
B, |N|
r
k 2 5mgR
0 = (∆Xmin ) − 5mg =⇒ ∆Xmin = . (8)
R k

Maneira 2:
Nesta situação a vB é mı́nima (pois acima disto, o carrinho consegue completar o “loop”),
~ = 0:
cujo valor pode ser encontrado através da Eq. (6), na condição crı́tica, |N|

mvB2 min p
0= − mg =⇒ vB min = gR. (9)
R
Substituindo a Eq. (9) na Eq. (5), onde a compressão ∆x é mı́nima:
r r
p k 2 5mgR
gR = (∆Xmin ) − 4gR =⇒ ∆Xmin = . (10)
m k

3
Questão discursiva 2 (valor=2.5 pontos)
a) valor=1,0 ponto
O projétil executa um movimento com aceleração constante, pois ~a = ~g . Assim a posição
e a velocidade do projétil após o lanamento são dadas pelas expressões:

~at2
~r(t) = ~r0 + ~v0 t + , (11)
2
~v(t) = ~v0 + ~at. (12)

Usando o sistema de coordenadas indicado na figura:

~r0 = h̂ (13)


~v0 = |~v0| cos αı̂ + |~v0|sen α̂ = v0 cos αı̂ + v0sen α̂ (14)
~a = ~g = −g̂ (15)

O vetor posição do projétil, ~r, é obtido substituindo-se as Eqs. (13), (14) e (15) na
Eq. (11):

~at2 gt2
~r(t) = ~r0 + ~v0t + h̂ + (v0 cos αı̂ + v0sen α̂)t − ̂ (16)
2 2
gt2
 
~r(t) = v0 cos αtı̂ + h + v0sen αt − ̂ (17)
2
O vetor velocidade do projétil, ~v, é obtido substituindo-se as Eqs. (14) e (15) na Eq.
(12):

~v (t) = ~v0 + ~at = v0 cos αı̂ + v0 sen α̂ − gt̂ = v0 cos αı̂ + (v0 sen α − gt) ̂ (18)

b) valor=0,5 ponto
Na altura máxima, a componente vertical da velocidade do projétil anula-se. Seja tH ,
o tempo necessário para que o projétil atinja o ponto mais alto da trajetória. Fazendo
vy (tH ) = 0 na Eq. (18), temos que:
v0 sen α
v0sen α − gtH = 0 =⇒ tH = . (19)
g

c) valor=0,5 ponto
O tempo de voo, tS , é o tempo que o projétil leva até atingir o solo. Isto ocorre, no
sistema de coordenadas da figura, quando ry (tS ) = 0. Usando esta condição na Eq. (17),
encontramos:

gt2S
 
1
q
2
h + v0 sen αtS − = 0 =⇒ tS = v0sen α + v0 sen α + 2gh ,
2 (20)
2 g
onde desprezamos a solução tS < 0.

4
d) valor=0,5 ponto
O módulo da velocidade ao atingir o solo, vS = |~v(tS )|, pode ser obtido de várias maneiras:

Maneira 1:
Como a aceleração é constante podemos utilizar a equação de Torricelli:

v 2(tS ) = v02 + 2~a · ∆~r =⇒ vS2 = v02 + 2(−g̂) · (∆xı̂ − h̂) (21)
q
2 2
∴ vS = v0 + 2gh =⇒ vS = v02 + 2gh. (22)

Maneira 2:
q
Pela definição, temos que o módulo de vS é vS = |~v(tS )| = vx2(tS ) + vy2(tS ), usando as
Eqs. (18) e (20):

q
2
vS = v02 cos2 α + (v0sen α − gtS )
s  2
q
2 2
= v0 cos α + v0sen α − v0sen α + v0 sen α + 2gh
2 2

s q 2
2 2
= v0 cos α +
2 v0 sen α + 2gh
2

q q
= v0 cos α + v0 sen α + 2gh = v02 + 2gh
2 2 2 2 (23)

Maneira 3:
Por considerações de conservação de energia mecânica temos:
1 1
E0 = mgh + mv02 e Esolo = mvS2
2 2
Como ∆E = 0, q
E0 = Esolo ⇒ |~vS | = vS = v02 + 2gh

Maneira 4:
Aplicando o Teorema-Trabalho Energia, ∆K = W T OT AL, logo:
1 2 1 2
mv − mv = WP eso = −∆U = mgh
2 S 2 0
q
∴ |~vS | = vS = v02 + 2gh

5
3. Forças conservativas são tais que: 6. A figura mostra um trilho no plano horizontal no qual
UNIVERSIDADE FEDERAL DO RIO DE JANEIRO uma partı́cula desloca-se da posição A para a posição
(a) Não produzem variação de energia cinética.
INSTITUTO DE FÍSICA B. Dentre os vetores ~a1 , ~a2 e ~a3 indicados na fi-
(b) Não produzem variaçãode energia potencial. gura, não pode(m) representar uma aceleração da
FÍSICA I – 2012/2 (c) Não produzem trabalho jamais. partı́cula, nas respectivas posições 1, 2, e 3,
PRIMEIRA PROVA – 22/05/2013 (d) Produzem trabalho em trajetórias fechadas
VERSÃO: A (e) Não produzem trabalho em trajetórias fecha-
das.
(a) ~a1 ;
Nas questões em que for necessário, considere que g é o módulo da aceleração da gravidade.
(b) ~a2 ;
(c) ~a3 ;
Seção 1. Múltipla escolha (10×0,5 = 5,0 pontos) (d) ~a1 e ~a2 ;
(e) ~a2 e ~a3 ;
1. Uma massa m está suspensa no campo gravitacional 2. Um pequeno bloco de massa m sobre uma mesa ho-
por uma mola de contante elástica k presa ao teto. rizontal comprime uma mola de constante elástica k 4. Um projétil é lançado do solo com uma velocidade que
Efeitos de atrito são desprezı́veis e observa-se que o de uma distância d, a partir da posição relaxada da faz um ângulo θ0 com a horizontal (0 < θ0 < π/2).
sistema oscila verticalmente em torno de sua posição mola em O, como mostra a figura. Liberado a partir Ignorando efeitos de resistência do ar e considerando
de equilı́ibrio. A opção correta é: do repouso, ele realiza um movimento retilı́neo hori- o intervalo de tempo decorrido entre o instante do
zontal para a direita percorrendo depois da posição O lançamento e o instante em que o projétil atinge a
(a) A energia mecânica do sistema não se conserva,
uma distância d ′ , quando atinge o repouso; há uma altura máxima, pode-se afirmar que o ângulo entre
pois além da força gravitacional sobre a massa
força de atrito constante da superfı́cie da mesa sobre o vetor velocidade média e o vetor aceleração média
há, adicionalmente, a força elástica exercida
o bloco em toda a extensão do movimento. O coefici- é:
pela mola.
ente de atrito cinético µc entre a superfı́cie e o bloco (a) igual a θ0
(b) A energia mecânica do sistema se conserva, se é:
considerarmos que esta é a soma das energias (b) igual a θ0 /2
cinética e energia potencial gravitacional da (c) maior do que π + θ0 /2 7. Uma partı́cula desloca-se ao longo do eixo x sob a
massa.
(d) menor do que π/2 + θ0 ação de uma força conservativa F~ , correspondente a
(c) A energia mecânica do sistema se conserva, se um potencial U (x), dado pela figura abaixo. Para este
(e) igual a π/2 − θ0 potencial entre as opções abaixo a única incorreta é:
considerarmos que esta é a soma das energias
cinética e energia potencial elástica da mola.
(d) A energia potencial total, dada como a soma k
das contribuições de energia potencial gravi- (a) (d − d ′ );
2mg
tacional e energia potencial elastica, é conser-
vada. k
(b) (d + d ′ );
2mg
(e) A energia mecânica do sistema se conserva, se
k
considerarmos que esta é a soma das energias (c) (d + d ′ );
mg 5. Dois carros A e B (considerados como partı́culas) par-
cinética e das energias potencial gravitacional
k tem da mesma posição no instante t = 0 e percorrem
e potencial elástica da mola. (d) (d ′ );
mg estradas perpendiculares, seguindo para o norte e o
leste, respectivamente, com velocidades constantes ~vA
k
(e) (d ′ ). e ~vB . A distância d entre os dois carros no instante t (a) na posição xB a força sobre a partı́cula é nula;
2mg satisfaz à relação
(b) na posição xD , tem-se a condição de equilı́brio
(a) |~vA | t < d < |~vB | t; instável;
(b) |~vB | t < d < |~vA | t; (c) no deslocamento do corpo de xA para xB o tra-
balho realizado pela força F~ é negativo;
p
(c) d= |~vA |2 + |~vB |2 t ;
o sentido da força F~ na posição xE é positivo;
p
(d) d = |~vA |2 − |~vB |2 t; (d)
(e) d = |~vA | t − |~vB | t. (e) na posição xC a força F~ não é nula.

1 2
8. Um bloco encontra-se em equilı́brio suspenso por 10. Em um lago tranquilo três barcos A, B e C têm veloci- Seção 2. Questões discursivas (2×2,5 = 5,0 pontos)
uma mola de constante elástica k, presa ao teto e com dades respectivas ~vA = vı̂ − v̂, ~vB = vı̂ e ~vC = −vı̂ + v̂
alongamento d. O bloco é levantado até a posição (v uma constante dada), todas relativas ao referen-
1. A figura seguinte representa um sistema formado por duas cunhas A e B, ambas com massa igual a m, sobre uma
em que o alongamento é nulo e abandonado a partir cial constituı́do por um ônibus estacionado na margem
superfı́cie horizontal perfeitamente lisa; não há contato entre a cunha A e essa superfı́cie horizontal. O coeficiente
do repouso. Ao descer uma altura d/2 o bloco ganha com um sistema de eixos OXY Z, sendo OXY na ho-
de atrito estático entre as superfı́cies das cunhas é igual a µe . Sob a ação de uma força F~ horizontal constante,
uma energia cinética, rizontal, conforme a figura. Considere o barco A como
aplicada em B, como mostra a figura, o sistema se move em movimento retilı́neo uniformemente acelerado, sem
um referencial com sistema de eixos OA XA YA ZA , cada
que a cunha A deslize sobre a cunha B. O módulo de F~ e demais condições são tais que o atrito entre as cunhas
um deles com mesma direção e sentido dos respectivos
(a) (1/2)kd2 ; tem que impedir a cunha A de deslizar para cima sobre a B. Considere como dados m, µe , F~ , o ângulo θ indicado
eixos de OXY Z. Em relação ao referencial do barco
(b) (1/4)kd2 ; na figura e a aceleração da gravidade ~g .
A as velocidades do ônibus, do barco B e do barco C
(c) (3/4)kd2 ; são, respectivamente, a) Determine a força resultante sobre a cunha A.
b) Calcule o módulo da força total que a cunha B exerce sobre a cunha A.
(d) (3/8)kd2 ; c) Represente em um diagrama todas as forças que agem sobre cada uma das cunhas.
(e) 2kd2 . d) Calcule o módulo da força de atrito entre as cunhas.
YA e) Determine a máxima intensidade da força aplicada F~ , de modo que a cunha A ainda não suba deslizando sobre
a cunha B.

OA XA
vA
9. Uma partı́cula de massa m está dentro de um funil de
vidro e percorre a sua superfı́cie interior com um mo-
vimento circular uniforme horizontal. Não há atrito vB
Y
entre a parede do funil e a partı́cula. O ângulo que a
parede do funil faz com o seu eixo de simetria é igual
a θ, como mostra a figura. Desprezando a presença do
ar, o módulo da força da superfı́cie sobre a partı́cula O X vC
e o módulo da aceleração centrı́peta da partı́cula são,
respectivamente,

2. Um objeto de massa m e dimensões desprezı́veis parte do repouso e de uma altura 4R, deslizando por uma rampa
(a) mg/cosθ e g/tanθ (a) ~vO′ = −vı̂ + v̂, ~vB′ = v̂ e ~vC′ = −2vı̂ + 2v̂. suave até encontrar uma superfı́cie horizontal, por onde segue até encontrar uma rampa circular de raio R, sobre a
(b) mg/senθ e g/cotθ (b) ~vO′ = vı̂ + v̂, ~vB′ = v̂ e ~vC′ = 2vı̂ + 2v̂ qual continua seu movimento (veja a figura). Não há atrito em todo o percurso do objeto, que se dá em um mesmo
plano vertical (da figura). Expressando suas respostas em termos dos unitários horizontal ı̂ e vertical ̂ indicados
(c) mg/cosθ e g/cotθ (c) ~vO′ = −vı̂ + v̂, ~vB′ = v̂ e ~vC′ = −2vı̂ − 2v̂ na figura, determine
(d) mg/senθ e g/senθ (d) ~vO′ = vı̂ − v̂, ~vB′ = v̂ e ~vC′ = 2vı̂ + 2v̂ a) o vetor velocidade, ~v , nos pontos A, B e C;
(e) mg/senθ e g/tanθ (e) nenhuma das respostas anteriores. b) o vetor de força normal, N ~ , nos pontos A, B e C.

3 4
Universidade Federal do Rio de Janeiro
Centro de Ciências Matemáticas e da Natureza
Instituto de Fı́sica
Primeira Prova de Fı́sica IA - 22/05/2013
Respostas para provas hı́bridas

Gabarito das Questões objetivas (valor=5.0 pontos)

Versão A Versão B
Questão (a) (b) (c) (d) (e) Questão (a) (b) (c) (d) (e)
1 1
2 2
3 3
4 4
5 5
6 6
7 7
8 8
9 9
10 10

Versão C Versão D
Questão (a) (b) (c) (d) (e) Questão (a) (b) (c) (d) (e)
1 1
2 2
3 3
4 4
5 5
6 6
7 7
8 8
9 9
10 10
Observação: a questão 2 da prova A foi anulada e demais correspondentes das provas B,
C e D; tarja preta. O texto não cita que o bloco está permanentemente preso à mola e o
desenho não mostra esta condição, embora a única opção compatı́vel seja a resposta (a)
da questão, admtindo que o bloco está preso à mola.

1
Questão discursiva 1 (valor=2.5 pontos)
a) valor=0.5 ponto
Pela segunda lei de Newton, o sistema de massa 2m se move com uma aceleração dada por
~a = F~ /(2m). A cunha A se move com a mesma aceleração ~a do conjunto. A resultante
das forças que atuam na cunha A corresponde à soma vetorial das forças N~ A , P~ e f~e .
Usando a segunda lei de Newton para a cunha A,
~ A + P~ + f~e = m~a = m F~ /(2m) = F~ /2,
N
ou diretamente neste caso:
F~A = m~a → F~A = F~ /2

b) valor=0.5 ponto
A força total exercida pela cunha B na cunha A é dada pela soma N ~ A + f~e . Pela equação
acima,
~ A + f~e | = |F~ /2 − P~ | = (F/2)2 + (−P )2 = F 2/4 + (mg)2
p p
|N

c) valor=0.5 ponto
O diagrama de forças das cunhas A e B são mostrados na figura abaixo, onde os vetores
indicados por (0 ) correspondem às reações das forças correspondentes.

d) valor=0,5 ponto
Projetando as forças nas direções dos eixos x e y, e aplicando a
segunda lei de Newton,


 NA senθ + fe cos θ = m ax = F/2

NA cos θ − fe senθ = m ay = 0

Resolvendo esse sistema de equações lineares nas incógnitas NA e


fe , obtemos o módulo fe da força de atrito entre as cunhas:
fe = (1/2)F cosθ − mg senθ

2
e) valor=0,5 ponto
A intensidade máxima Fmax da força aplicada corresponde à situação em que a cunha A
está na iminência de deslizar sobre a cunha B. Nesse caso o módulo da força de atrito
estático atinge o seu valor máximo µe N. Usando os resultados do item (d), obtemos o
módulo da força normal N.

NA = (1/2)F senθ + mg cosθ

Com a condição de que o atrito máximo dado por |f~e | = µe NA , temos:

(1/2)Fmax cosθ − mg senθ = µe [(1/2)Fmax senθ + mg cosθ]

!
senθ + µe cosθ
∴ Fmax = 2mg
cosθ − µe senθ

3
Questão discursiva 2 (valor=2.5 pontos)
a) valor=1,3 pontos
Usando a lei de conservação da energia mecânica podemos calcular os módulos das veloci-
dades em A, B e C. Os sentidos e direções dos vetores velocidade são dados em função do
movimento se dar de A → B → C e de que o vetor velocidade é tangente à trajetória.
√ √
Em A: 4mgR = 12 mvA2 . Assim, vA = 2 2gR → ~vA = 2 2gRı̂.
√ √
Em B: 4mgR = mgR + 12 mvB2 . Assim, vB = 6gR → ~vB = 6gR̂.
√ √
Em C: 4mgR = 2mgR + 12 mvC2 . Assim, vC = 2 gR → ~vC = −2 gRı̂.

b) valor=1,2 pontos
No trecho circular do movimento ascendente, a projeção da resultante na direção radial é
identificada à forca centrı́peta. Com os valores das velocidade obtidos do item anterior,
temos,
mvA2 ~ A = 9mg̂.
Em A: NA − mg = = 8mg. Assim, NA = 9mg → N
R
mvB2 ~ B = −6mgı̂.
Em B: NB = = 6mg. Assim, NB = 6mg → N
R
mvC2 ~ C = −3mg̂.
Em C: NC + mg = = 4mg. Assim, NC = 3mg → N
R

4
5. Em um trecho retilı́neo de um rio as águas tem velo- 7. Uma partı́cula se desloca ao longo do eixo OX, da
UNIVERSIDADE FEDERAL DO RIO DE JANEIRO
cidade relativa às margens ~vA/M constante e a mesma origem até a posição x3 = 3d, onde d é uma distância
INSTITUTO DE FÍSICA em todos os pontos do trecho do rio. Um piloto de- positiva. A única componente da força resultante so-
FÍSICA I – 2013/2 seja atravessar o rio de modo que a velocidade ~vB/M bre a partı́cula, Fx , varia com a posição x conforme
de seu barco relativa às margens tenha o mesmo o gráfico da figura, linearmente da origem a x2 = 2d,
PRIMEIRA PROVA – 07/10/2013 módulo que a velocidade das águas relativa às mar- cortando o eixo OX em x1 = d, e também linearmente
VERSÃO: A gens, |~vB/M | = |~vA/M |. Além disso, ele deseja seguir de x2 = 2d até x3 = 3d; no gráfico também está in-
uma trajetória retilı́nea de um ponto P1 da margem dicado o valor máximo F0 da força. Denotando por
até o ponto P2 diretamente em frente a P1 na margem K0 , K1 , K2 e K3 as energias cinéticas nas posições
Nas questões em que for necessário, considere que g é o módulo da aceleração da gravidade. oposta, como indica a figura. Podemos afirmar que o x = 0, x1 = d, x2 = 2d e x3 = 3d, respectivamente,
piloto deve imprimir ao barco uma velocidade ~vB/A , podemos afirmar sobre as variações ∆K1 = K1 − K0 ,
relativa às águas de módulo ∆K2 = K2 − K0 e ∆K3 = K3 − K0 que
Seção 1. Múltipla escolha (10×0,5 = 5,0 pontos)

1. Três macacos A, B e C, de mesmo peso, estão pen- 3. Um lustre é pendurado em um teto horizontal por
durados em repouso, cada um em um cipó, com os dois fios, considerados ideais, que se juntam e são
cipós amarrados em um galho de árvore. A partir de ligados ao lustre, como mostra a figura. Em um
um dado instante o macaco A sobe em seu cipó com dos fios, que faz um ângulo θ1 com o teto, a tensão
velocidade constante, o macaco B sobe no seu com tem módulo T1 ; no outro, que faz um ângulo θ2 com
aceleração para cima constante e o macaco C desce o teto, a tensão tem módulo T2 . Considerando o
no seu com aceleração para baixo constante. Supondo sistema de eixos OXY com OY na vertical, como √
que os cipós sejam fios ideais (inextensı́veis com mas- indicado na figura, obtemos que a razão T2y /T1y entre (a) 2|~vA/M |;
sas desprezı́veis), que os movimentos se processem na as componentes verticais T2y e T1y das tensões é (b) |~vA/M |;
vertical, que as acelerações mencionadas não sejam (a) sen θ2 /cos θ1 ; √
3
nulas, e denotando por TA , TB e TC as tensões nos (c) |~vA/M |;
(b) sen θ2 /sen θ1 ; 2
cipós dos respectivos macacos A, B e C, podemos di- √ (a) ∆K1 < ∆K2 < ∆K3 ;
zer que (c) tan θ2 /tan θ1 ; 2
(d) |~vA/M |; (b) ∆K1 > ∆K2 > ∆K3 ;
(d) cos θ1 /cos θ2 ; 2
(a) TA = TB = TC ; 1 (c) ∆K1 = ∆K3 < ∆K2 ;
(e) cos θ2 /tan θ1 . (e) |~vA/M |.
(b) TA > TB > TC ; 2 (d) ∆K1 = ∆K3 > ∆K2 ;
(c) TA < TB = TC ; (e) ∆K1 > ∆K2 e ∆K2 < ∆K3 .
(d) TA < TB e TB > TC ;
(e) TA > TB = TC .
8. Um corpo de dimensões desprezı́veis e de massa m é
4. Uma partı́cula em trajetória circular de raio R tem 6. Um bloco abandonado em repouso está descendo com liberado a partir do repouso da borda de um poço
2. Um sistema de coordenadas tem origem no solo de um vetor aceleração ~a que em um certo instante faz velocidade constante a superfı́cie de um plano incli- semiesférico de raio R. Ele desliza pela parede do
uma região plana horizontal, e dessa origem é lançado um ângulo π/3 com o vetor velocidade ~v . Podemos nado. A força de atrito que a superfı́cie exerce sobre poço e após algumas oscilações em torno do fundo do
um projétil que faz um ângulo maior que zero com afirmar que nesse instante o bloco é poço permanece em repouso nesta posição. Para as
a horizontal. Ao passar por sua altura máxima o forças presentes desde o inı́cio até o fim do movimento
(a) o módulo |~v | da velocidade da partı́cula é igual (a) paralela à superfı́cie do plano inclinado com
projétil tem uma velocidade ~vH e ao atingir o solo do corpo é correta a opção
a |~a| cos(π/3); módulo menor do que o peso do bloco;
tem vetor posição ~rA . Podemos afirmar sobre os pro-
dutos escalares desses vetores com o vetor aceleração (b) o módulo |~v | da velocidade da partı́cula é igual (b) perpendicular à superfı́cie do plano inclinado
com módulo menor do que o peso do bloco; (a) o trabalho da força peso é −mgR;
que a |~a|sen(π/3);
(c) paralela à superfı́cie do plano inclinado com (b) o trabalho da força normal que a superfı́cie do
(a) ~vH · ~g > 0 e ~rA · ~g = 0; (c) a aceleração da partı́cula tem módulo |~a| =
módulo maior do que o peso do bloco; poço exerce sobre o corpo não é nulo;
|~v |2 /R;
(b) ~vH · ~g < 0 e ~rA · ~g = 0; (c) o trabalho total das forças dissipativas é nulo;
(d) o módulo |~v | da velocidade da partı́cula está (d) perpendicular à superfı́cie do plano inclinado
(c) ~vH · ~g > 0 e ~rA · ~g < 0; com módulo maior do que o peso do bloco; (d) o trabalho total das forças dissipativas é mgR.
aumentando;
(d) ~vH · ~g > 0 e ~rA · ~g > 0. (e) paralela à superfı́cie do plano inclinado com (e) o trabalho total das forças dissipativas é
(e) o módulo |~v | da velocidade da partı́cula está
(e) ~vH · ~g = 0 e ~rA · ~g = 0; diminuindo. módulo igual ao peso do bloco. −mgR;

1 2
9. Os pneus de um carro de massa M têm coeficiente de 10. Uma partı́cula move-se no plano OXY em uma tra- Seção 2. Questões discursivas (2×2,5 = 5,0 pontos)
atrito estático µe com a pista horizontal de uma ro- jetória retilı́nea ao longo do eixo OX, da origem ao
dovia em uma curva circular de raio R. A velocidade ponto (d, 0), e a seguir ao longo de uma trajetória
1. Uma esfera muito pequena de massa m está amarrada a uma haste vertical giratória por dois fios ideais de
máxima com que o carro pode fazer essa curva sem retilı́nea do ponto (d, 0) ao ponto (d, d), como indi-
comprimento ℓ. A haste gira de modo que os fios fiquem tensos formando com a haste um triângulo que mantem
derrapar é vmax . Supondo que o carro faça a curva cado na figura. Durante todo o percurso agem sobre
seu formato inalterado durante o movimento, conforme ilustra a figura. Sabendo que os fios de cima e de baixo
com velocidade de módulo constante vmax /2, pode- a partı́cula três forças constantes F~1 , F~2 e F~3 de mesmo
fazem o mesmo ângulo θ com a horizontal e que a velocidade de rotação da esfera tem módulo constante v, determine
mos dizer que, nesse caso, a força de atrito estático módulo. Os ângulos que as forças F~1 ,F~2 e F~3 fazem
entre os pneus e a pista tem módulo igual a com o eixo OX são, respectivamente, π/4, π/2 e 3π/4. a) o módulo e a orientação (direção e sentido) da força resultante F~R sobre a esfera;
Denotando os respectivos trabalhos das três forças em b) o módulo T1 da tensão no fio superior e o módulo T2 da tensão no fio inferior;
(a) µe M g todo o percurso de (0, 0) a (d, d) por W1 , W2 e W3 , c) o limite mı́nimo vmin do módulo da velocidade de rotação da esfera em torno da haste para que o fio inferior se
(b) 2
M vmax /(4R) podemos afirmar que mantenha esticado.
2
(c) M vmax /(2R)
2
(d) M vmax /R
(e) µe M g/2

(a) W1 > 0, W2 = 0 e W3 < 0;


(b) W1 < 0, W2 = 0 e W3 = 0;
(c) W1 > 0, W2 > 0 e W3 = 0
(d) W1 < 0, W2 = 0 e W3 < 0; 2. Um anel de massa m, ligado a duas molas de constantes elásticas iguais a k1 e k2 , pode deslizar sem atrito sobre
(e) o trabalho realizado por cada uma das três um trilho rı́gido horizontal, como mostra a figura, na qual está indicado um sistema de eixos OXY , com OX na
forças é o mesmo. direção do trilho. Inicialmente o anel se encontrava em repouso sobre o trilho, na posição x = 0 onde as duas molas
estão relaxadas. A partir do instante t = 0 o anel passou a sofrer a ação de uma força constante F~ , numa direção
que faz um ângulo θ com a horizontal, levando o anel a oscilar em torno de um novo ponto de equilı́brio.
a) Qual a força total que o trilho exerce sobre o anel?
b) Em que posição é nula a resultante das forças que atuam sobre o anel ?
c) Escreva a energia potencial elástica do sistema em função da coordenada x do anel.
d) Escreva a energia potencial associada à força constante F~ em função da coordenada x do anel.
e) Expresse o módulo da velocidade do anel em função de sua coordenada x.

3 4
Universidade Federal do Rio de Janeiro
Centro de Ciências Matemáticas e da Natureza
Instituto de Fı́sica
Primeira Prova de Fı́sica IA - 07/10/2013
Respostas para provas hı́bridas

Gabarito das Questões objetivas (valor=5,0 pontos)

Versão A Versão B
Questão (a) (b) (c) (d) (e) Questão (a) (b) (c) (d) (e)
1 1
2 2
3 3
4 4
5 5
6 6
7 7
8 8
9 9
10 10

Versão C Versão D
Questão (a) (b) (c) (d) (e) Questão (a) (b) (c) (d) (e)
1 1
2 2
3 3
4 4
5 5
6 6
7 7
8 8
9 9
10 10

1
Questão discursiva 1 (valor=2,5 pontos)
a) valor=0,5 ponto
A partı́cula está em movimento horizontal circular uniforme de raio ` cos θ em torno da haste,
com aceleração centrı́peta de módulo v 2/(` cos θ). Pela segunda lei de Newton, a força resul-
tante F~R é horizontal, orientada da esfera para a haste e de módulo |F~R| = mv 2/(`cos θ).

b) valor=1,5 pontos
Pela segunda lei de Newton T~1 + T~2 + m~g = m~arad, onde ~arad é a aceleração centrı́peta. Usando
em um instante qualquer um eixo OX horizontal apontando da partı́cula para a haste e um
eixo OY vertical apontando para cima, a segunda lei de Newton nos fornece as projeções
T1 cos θ + T2 cos θ = marad e T1 sen θ − T2 sen θ − mg = 0, isto é,

v2
(T1 + T2 ) cos θ = m , (1)
` cos θ
(T1 − T2) sen θ = mg , (2)

ou seja,  ma
 T 1 + T 2 =
cos θ

 T1 − T2 = mg


sen θ
A resolução desse sistema de equações permite obter, respectivamente,

mv 2 mv 2
   
1 mg 1 mg
T1 = + , T2 = − . (3)
2 ` cos2 θ sen θ 2 ` cos2 θ sen θ

c) valor=0,5 ponto
O fio fica esticado se, e somente se, T2 > 0, de modo que no limite mı́nimo T2 = 0. Graças a
2
segunda equação em (3), essa condição é equivalente a mvmin /(` cos2 θ) = mg/ sen θ, isto é,
r
g` cos2 θ
vmin = . (4)
sen θ

2
Questão discursiva 1 (valor=2,5 pontos)
a) valor=0,5 ponto
~ que o trilho exerce sobre o anel é vertical, N
O trilho é liso, logo a força total N ~ = Ny ̂. Como
não há movimento vertical do anel, pela segunda lei de Newton Ny + F sin θ − mg = may = 0,
isto é, Ny = mg − F sin θ. Portanto,

~ = (mg − F sin θ)̂.


N (1)

b) valor=0,5 ponto
A força elástica em um deslocamento x é Fel = −k1 x − k2 x, isto é, Fel = −(k1 + k2 )x. Na
nova posição de equilı́brio x0, as forças horizontais se anulam, Fel + F cos θ = 0, isto é, −(k1 +
k2 )x0 + F cos θ = 0. Portanto,

x0 = F cos θ/(k1 + k2 ). (2)

c) valor=0,5 ponto
Tendo em vista a expressão da força elástica, a energia potencial elástica é a mesma de uma
única mola de constante elástica k1 + k2 . Usando o zero dessa energia potencial em x = 0, ela
é dada por,

Uel (x) = (1/2)(k1 + k2 )x2 . (3)

d) valor=0,5 ponto
Para a força constante F~ , a energia potencial em x, com o zero dessa energia em x = 0,
corresponde ao trabalho que a força constante realizaria se a partı́cula fosse de x a x = 0, isto
é, UF (x) = Fx (0 − x) . Portanto,

UF (x) = −(F cos θ)x. (4)

e) valor=0,5 ponto
Como as forças que realizam trabalho são conservativas, há conservação da energia mecânica,
dada por E = (1/2)mv 2 + (1/2)(k1 + k2 )x2 − (F cos θ)x. Igualando o valor dessa energia ao seu
valor em x = 0, em que v = 0, obtemos (1/2)mv 2 + (1/2)(k1 + k2 )x2 − (F cos θ)x = 0, ou seja,
(1/2)mv 2 = (F cos θ)x − (1/2)(k1 + k2 )x2. Portanto,
  1/2
2 1
v= (F cos θ)x − (k1 + k2 )x2 . (5)
m 2

3
4. Uma partı́cula desloca-se ao longo do eixo OX, de 6. Considere as seguintes afirmações sobre os vetores velo-
UNIVERSIDADE FEDERAL DO RIO DE JANEIRO
unitário ı̂, sob a ação de uma força conservativa cidade e aceleração de uma partı́cula em movimento: I)
INSTITUTO DE FÍSICA ~ = F (x)ı̂, correspondente ao potencial U (x), dado
F o vetor velocidade pode ser nulo e o vetor aceleração ser
FÍSICA I – 2014/1 pelo gráfico da figura. Para este potencial, entre as diferente de zero. II) O módulo do vetor velocidade pode
alternativas abaixo, a única INCORRETA é ser constante, com o vetor velocidade mudando com o
PRIMEIRA PROVA – 31/03/2014 tempo. III) O vetor velocidade pode ser constante mas
VERSÃO: A seu módulo variar com o tempo. IV) O módulo do vetor
velocidade é constante quando o vetor aceleração é sem-
pre perpendicular ao vetor velocidade. São verdadeiras
Nas questões em que for necessário, considere que g é o módulo da aceleração da gravidade. as afirmações:
(a) Todas as afirmações;
Seção 1. Múltipla escolha (10×0,5 = 5,0 pontos) (b) I, II e III;
(c) I, II e IV;
1. Um projétil é lançado do solo com uma velocidade que 3. Vista de um referencial OXY Z fixo nas estrela e cen- (d) II e III;
faz um ângulo θ0 com a horizontal (0 < θ0 < π/2). Igno- trado no Sol, a Terra tem vetor velocidade dado em
(e) Nenhuma das afirmações anteriores.
rando efeitos de resistência do ar e considerando o inter- um certo instante por ~v = vx ı̂ + vy ̂ e um foguete em
valo de tempo decorrido entre o instante do lançamento translação tem velocidade constante V~ = −V ̂, como
e o instante em que o projétil volta a altura em que mostra a figura. Considerando o foguete como um re-
7. Em uma balança dentro de um elevador em movimento
foi lançado, pode-se afirmar que o ângulo entre o vetor ferencial com eixos coordenados O′ X ′ , O′ Y ′ e O′ Z ′ de
vertical é colocado um bloco de massa m e a balança
velocidade média e o vetor aceleração média é: mesmas direções e sentidos que os respectivos eixos OX, (a) na posição xO a força sobre a partı́cula é nula;
indica sobre seu prato uma força vertical de módulo N
(a) π/2 OY e OZ, o vetor velocidade da Terra relativo a ele é (b) nas posições xb e xc tem-se a condição de exercida pelo bloco. Em relação a um sistema de eixos
(b) θ0 /2 equilı́brio estável; OXY Z, fixo na Terra, com OY vertical orientado para
(c) no delocamento da partı́cula de xc para xd o tra- cima a aceleração do elevador é dada por uma compo-
(c) θ0
~ é negativo;
balho realizado Wc−d pela força F nente ay igual a
(d) π + θ0 /2
(d) no delocamento da partı́cula de xb para xc a va- (a) N/m;
(e) zero riação da energia cinética é ∆K > 0; (b) N/m-g/2;
(e) ~ é positivo.
na posição xa o sentido da força F (c) N/m+g;
(d) N/m-g;
(e) todas as opções acima estão erradas.

8. Dois blocos de massas 2m e m estão em movimento re-


tilı́neo sobre uma superfı́cie horizontal sem atrito e li-
5. Dois fios ideais AC e BC estão presos a um teto hori- gados por um fio ideal horizontal que se mantém tenso
durante o movimento. Sobre os blocos agem forças F ~
2. Um carro, considerável como uma partı́cula, sobe uma zontal e a uma parede vertical nos respectivos pontos A
e B, como indica a figura. Na extremidade comum C constantes na direção do movimento, como mostra a fi-
lombada circular de centro de curvatura em O, como
dos fios está suspenso e em equilı́brio um bloco de peso gura. O módulo T da tensão do fio é
indica a figura. O módulo da velocidade do carro vai (a) v ′ = vx ı̂ + vy ̂;
~
diminuindo a medida que ele sobe a lombada. Dadas as P . O fio BC é perpendicular à parede e o fio AC faz
(b) v ′ = vx ı̂ + (vy + V )̂;
~
setas identificadas pelos números 1, 2, 3, 4 e 5 da figura, um ângulo θ com o teto (0 < θ < π/2). O módulo T da
a que pode representar a aceleração do carro no ponto (c) v ′ = (vx + V )ı̂ + (vy + V )̂;
~ tensão no fio AC é igual a
P da subida indicado é a número (d) v ′ = vx ı̂ + (vy − V )̂;
~
(a) 2 (e) v ′ = −(vx + V )ı̂ + −vy ̂.
~ (a) P cot θ; (a) T = 3F ;
(b) 1 (b) P/cos θ; (b) T = 2F ;
(c) 4 (c) P sen θ; (c) T = F/2;
(d) 5 (d) P tan θ; (d) T = (2/3)F ;
(e) 3 (e) P/sen θ. (e) T = F/3.

1 2
9. Uma partı́cula descreve um movimento circular de raio 10. Uma partı́cula move-se no plano horizontal OXY sob a Seção 2. Questões discursivas (2×2,5 = 5,0 pontos)
R, com velocidade de módulo constante e igual a V . ação de uma força constante F~ =Fı̂. Em um dado ins-
Num intervalo de tempo em que percorre 1/2 da circun- tante a sua posição é ~ri = aı̂ + b̂ e em um instante Não serão consideradas respostas sem justificativa; expresse-as somente em função dos dados fornecidos.
ferência, o módulo de seu vetor aceleração média é posterior a sua posição é ~
rf = aı̂ − b̂, onde a e b são
(a) zero; constantes. O trabalho realizado por essa força do ins- 1. Uma partı́cula de massa m é suspensa por uma mola ideal de
tante inicial ao final é constante elástica k presa ao teto. A partı́cula gira em movi-
2 2
(b) V ; mento circular uniforme horizontal, de raio R e centrado na
πR (a) nulo;
2
vertical que passa pelo ponto de suspensão P da mola no teto,
V (b) 2F a; como indica a figura. A mola mantém-se reta e distendida de
(c) ;
R um comprimento ∆ℓ (a ser determinado). A reta que passa
(c) 2F b;
3 pelo ponto P de suspensão da mola e a partı́cula mantém
(d) V 2; (d) −F 2b;
2πR √ um angulo constante θ com a vertical, descrevendo uma su-

2 2 (e) F a2 + b 2 pefı́cie cônica no espaço. Considere como dados m, k, θ, R e
(e) V .
2πR o módulo g da aceleração da gravidade.
a) Represente em um diagrama as forças que agem sobre a
partı́cula em um instante arbitrário.
b) Calcule a distensão ∆ℓ da mola.
c) Determine o módulo da velocidade da partı́cula.

2. A figura mostra o perfil suave de uma calha com um trecho inclinado AB, seguido de um trecho horizontal BC, que
é seguido de um outro trecho inclinado CD; as alturas do ponto A e do ponto D acima do solo são iguais a h0 e o
comprimento do trecho horizontal BC é igual a 2h0 . Um bloco de massa m e dimensões desprezı́veis desce a partir do
ponto A, onde o módulo da sua velocidade é vA e percorre os trechos AB, BC e CD. O coeficiente de atrito cinético
entre o bloco e a calha no trecho BC é µ, e entre o bloco e a calha nos trechos AB e CD é zero. Considerando como
dados m, vA , h0 e o módulo g da aceleração da gravidade, calcule:
a) os trabalhos das forças peso e normal nos trechos AB, BC e CD;
b) o trabalho da força de atrito no trecho BC;
c) a variação da energia cinética do ponto A ao ponto D e o módulo da velocidade do bloco em D;

3 4
Gabarito para Versão A 4. Uma partı́cula desloca-se ao longo do eixo OX, de 6. Considere as seguintes afirmações sobre os vetores velo-
unitário ı̂, sob a ação de uma força conservativa cidade e aceleração de uma partı́cula em movimento: I)
~ = F (x)ı̂, correspondente ao potencial U (x), dado
F o vetor velocidade pode ser nulo e o vetor aceleração ser
Seção 1. Múltipla escolha (10×0,5 = 5,0 pontos)
pelo gráfico da figura. Para este potencial, entre as diferente de zero. II) O módulo do vetor velocidade pode
alternativas abaixo, a única INCORRETA é ser constante, com o vetor velocidade mudando com o
1. Um projétil é lançado do solo com uma velocidade que 3. Vista de um referencial OXY Z fixo nas estrela e cen- tempo. III) O vetor velocidade pode ser constante mas
faz um ângulo θ0 com a horizontal (0 < θ0 < π/2). Igno- trado no Sol, a Terra tem vetor velocidade dado em seu módulo variar com o tempo. IV) O módulo do vetor
rando efeitos de resistência do ar e considerando o inter- um certo instante por ~v = vx ı̂ + vy ̂ e um foguete em velocidade é constante quando o vetor aceleração é sem-
valo de tempo decorrido entre o instante do lançamento translação tem velocidade constante V~ = −V ̂, como pre perpendicular ao vetor velocidade. São verdadeiras
e o instante em que o projétil volta a altura em que mostra a figura. Considerando o foguete como um re- as afirmações:
foi lançado, pode-se afirmar que o ângulo entre o vetor ferencial com eixos coordenados O′ X ′ , O′ Y ′ e O′ Z ′ de
velocidade média e o vetor aceleração média é: mesmas direções e sentidos que os respectivos eixos OX, (a) Todas as afirmações;
(a) π/2 OY e OZ, o vetor velocidade da Terra relativo a ele é (b) I, II e III;
(b) θ0 /2 (c) I, II e IV;
(c) θ0 (d) II e III;
(d) π + θ0 /2 (e) Nenhuma das afirmações anteriores.
(e) zero

7. Em uma balança dentro de um elevador em movimento


vertical é colocado um bloco de massa m e a balança
(a) na posição xO a força sobre a partı́cula é nula;
indica sobre seu prato uma força vertical de módulo N
(b) nas posições xb e xc tem-se a condição de exercida pelo bloco. Em relação a um sistema de eixos
equilı́brio estável; OXY Z, fixo na Terra, com OY vertical orientado para
(c) no delocamento da partı́cula de xc para xd o tra- cima a aceleração do elevador é dada por uma compo-
~ é negativo;
balho realizado Wc−d pela força F nente ay igual a
(d) no delocamento da partı́cula de xb para xc a va- (a) N/m;
riação da energia cinética é ∆K > 0; (b) N/m-g/2;
(e) ~ é positivo.
na posição xa o sentido da força F
(c) N/m+g;
(a) v ′ = vx ı̂ + vy ̂;
~ (d) N/m-g;

(b) v ′ = vx ı̂ + (vy + V )̂;


~ (e) todas as opções acima estão erradas.

(c) v = (vx + V )ı̂ + (vy + V )̂;


~ ′

(d) v ′ = vx ı̂ + (vy − V )̂;


~ 8. Dois blocos de massas 2m e m estão em movimento re-
(e) v ′ = −(vx + V )ı̂ + −vy ̂.
~ tilı́neo sobre uma superfı́cie horizontal sem atrito e li-
5. Dois fios ideais AC e BC estão presos a um teto hori- gados por um fio ideal horizontal que se mantém tenso
durante o movimento. Sobre os blocos agem forças F ~
2. Um carro, considerável como uma partı́cula, sobe uma zontal e a uma parede vertical nos respectivos pontos A
e B, como indica a figura. Na extremidade comum C constantes na direção do movimento, como mostra a fi-
lombada circular de centro de curvatura em O, como
dos fios está suspenso e em equilı́brio um bloco de peso gura. O módulo T da tensão do fio é
indica a figura. O módulo da velocidade do carro vai
diminuindo a medida que ele sobe a lombada. Dadas as P . O fio BC é perpendicular à parede e o fio AC faz
setas identificadas pelos números 1, 2, 3, 4 e 5 da figura, um ângulo θ com o teto (0 < θ < π/2). O módulo T da
a que pode representar a aceleração do carro no ponto tensão no fio AC é igual a
P da subida indicado é a número
(a) 2 (a) P cot θ; (a) T = 3F ;
(b) 1 (b) P/cos θ; (b) T = 2F ;
(c) 4 (c) P sen θ; (c) T = F/2;
(d) 5 (d) P tan θ; (d) T = (2/3)F ;
(e) 3 (e) P/sen θ. (e) T = F/3.

1 2
9. Uma partı́cula descreve um movimento circular de raio 10. Uma partı́cula move-se no plano horizontal OXY sob a Seção 2. Questões discursivas (2×2,5 = 5,0 pontos)
R, com velocidade de módulo constante e igual a V . ação de uma força constante F~ =Fı̂. Em um dado ins-
Num intervalo de tempo em que percorre 1/2 da circun- tante a sua posição é ~ri = aı̂ + b̂ e em um instante Não serão consideradas respostas sem justificativa; expresse-as somente em função dos dados fornecidos.
ferência, o módulo de seu vetor aceleração média é posterior a sua posição é ~
rf = aı̂ − b̂, onde a e b são
(a) zero; constantes. O trabalho realizado por essa força do ins- 1. Uma partı́cula de massa m é suspensa por uma mola ideal de
2 2 tante inicial ao final é constante elástica k presa ao teto. A partı́cula gira em movi-
(b) V ; mento circular uniforme horizontal, de raio R e centrado na
πR (a) nulo;
V 2 vertical que passa pelo ponto de suspensão P da mola no teto,
(c) ; (b) 2F a;
R como indica a figura. A mola mantém-se reta e distendida de
3 (c) 2F b; um comprimento ∆ℓ (a ser determinado). A reta que passa
(d) V 2; (d) −F 2b; pelo ponto P de suspensão da mola e a partı́cula mantém
2πR
√ √ um angulo constante θ com a vertical, descrevendo uma su-
2 2 (e) F a2 + b 2
(e) V . pefı́cie cônica no espaço. Considere como dados m, k, θ, R e
2πR o módulo g da aceleração da gravidade.
a) Represente em um diagrama as forças que agem sobre a
partı́cula em um instante arbitrário.
b) Calcule a distensão ∆ℓ da mola.
c) Determine o módulo da velocidade da partı́cula.

2. A figura mostra o perfil suave de uma calha com um trecho inclinado AB, seguido de um trecho horizontal BC, que
é seguido de um outro trecho inclinado CD; as alturas do ponto A e do ponto D acima do solo são iguais a h0 e o
comprimento do trecho horizontal BC é igual a 2h0 . Um bloco de massa m e dimensões desprezı́veis desce a partir do
ponto A, onde o módulo da sua velocidade é vA e percorre os trechos AB, BC e CD. O coeficiente de atrito cinético
entre o bloco e a calha no trecho BC é µ, e entre o bloco e a calha nos trechos AB e CD é zero. Considerando como
dados m, vA , h0 e o módulo g da aceleração da gravidade, calcule:
a) os trabalhos das forças peso e normal nos trechos AB, BC e CD;
b) o trabalho da força de atrito no trecho BC;
c) a variação da energia cinética do ponto A ao ponto D e o módulo da velocidade do bloco em D;

3 4
Universidade Federal do Rio de Janeiro
Centro de Ciências Matemáticas e da Natureza
Instituto de Fı́sica
Primeira Prova de Fı́sica IA - 31/03/2014
Respostas para provas hı́bridas

Gabarito das Questões objetivas (valor=5,0 pontos)

Versão A Versão B
Questão (a) (b) (c) (d) (e) Questão (a) (b) (c) (d) (e)
1 1
2 2
3 3
4 4
5 5
6 6
7 7
8 8
9 9
10 10

Versão C Versão D
Questão (a) (b) (c) (d) (e) Questão (a) (b) (c) (d) (e)
1 1
2 2
3 3
4 4
5 5
6 6
7 7
8 8
9 9
10 10

1
Questão 1 (valor=2,5 pontos)
a) valor=0,5 ponto
A vizinhança da partı́cula é constituı́da pela Terra, que exerce
a força peso P~ , e pela mola, que exerce uma força F~ orientada da
partı́cula para o ponto de suspensão P da mola, conforme a figura
ao lado, num tempo t arbitrário.

b) valor=1,0 ponto
Pela segunda lei de Newton F~ + P~ = m~a, onde ~a é a aceleração
centrı́peta da partı́cula. Projetando os vetores dessa equação em
um eixo horizontal na direção e sentido da aceleração centrı́peta e
em um eixo vertical para cima, obtemos

v2
F sin θ = m e F cos θ − mg = 0 . (1)
R
Da segunda dessas equações obtemos para a força da mola F = mg/ cos θ, que corresponde a
uma elongação ∆` = F/k, isto é,

∆` = mg/(k cos θ)
.
c) valor=1,0 ponto
Eliminando F entre as duas equações em (1), obtemos (mg/ cos θ) sin θ = m(v 2/R), isto é,
g tan θ = v 2/R, donde

p
v= gR tan θ
.

2
Questão 2 (valor=2,5 pontos)
a) valor=0,6 pontos
Vamos usar um sistema de eixos no plano da calha com OX horizontal para a direita e
OY vertical para cima. A força normal à superfı́cie da calha é perpendicular ao desloca-
mento infinitesimal em cada ponto da trajetória do bloco e, portanto, seu trabalho é sem-
pre nulo, WN (A → B) = 0, WN (B → C) = 0 e WN (C → D) = 0. O peso é uma força
constante vertical e, portanto, seu trabalho em qualquer deslocamento de y1 a y2 é igual a
−mg(y2 − y1 ) . Portanto, Wgrav (A → B) = −mg(yB − yA ), Wgrav (B → C) = −mg(yC − yB )
e Wgrav (C → D) = −mg(yD − yC ), ou seja, Wgrav (A → B) = mgh0, Wgrav (B → C) = 0 e
Wgrav (C → D) = −mgh0 .

b) valor=0,7 pontos
No trecho BC o movimento do bloco é horizontal e, portanto, sua aceleração vertical é nula.
Consequentemente, o módulo N da força normal da calha sobre o bloco é igual ao peso do bloco,
N = mg. Daı́ obtemos para o módulo da força de atrito cinético nesse trecho: f = µN = µmg.
Como a força de atrito tem sentido oposto ao movimento e é constante no trecho BC, obtemos
que seu trabalho nesse trecho é Watr (B → C) = −f(xC − xB ) = −µmg(2h0 ) , isto é,

Watr (B → C) = −2µmgh0

c) valor=1,2 pontos
Pelo teorema do trabalho energia, a variação da energia cinética é igual ao trabalho total
realizado pelas forças Normal (N), Peso (grav) e atrito (atr), que atuam sobre o bloco ao longo
do seu deslocamento na calha de A para D,

∆KA−D = WT otal(A → D) = WN (A → D) + Wgrav (A → D) + Watr (A → D)

De acordo com os resultados obtidos no item (a), WN (A → D) = 0 e Wgrav (A → D) =


−mg(yD − yA ) = 0 pois yA = yD = h0 .
A força de atrito atua somente no trecho B − C portanto Watr (A → D) = Watr (B → C) =
−2µmgh0 , de acordo com o item (b). Logo,

∆KA−D = −2µmgh0 .
2
Consequentemente, (1/2)mvD − (1/2)mvA2 = −2mµgh0 , donde
q
vD = vA2 − 4µgh0

Observação: No item (c) podemos obter a variação da energia cinética entre A e D e o


módulo da velocidade em D usando o princı́pio de conservação da energia mecânica, ∆EA−D =
Watr . Onde ∆EA−D = ∆KA−D +∆UA−D ; U é a energia potencial gravitacional. Como yA = yD ,
UA = UD e ∆UA−D = 0, assim
∆KA−D = −2µmgh0
2
p
e, (1/2)mvD − (1/2)mvA2 = −2mµgh0 , → vD = vA2 − 4µgh0

3
Gabarito para Versão A 5. Um corpo de massa m está preso a um fio inextensı́vel de 7. O gráfico representa a velocidade de uma partı́cula que
massa desprezı́vel e gira em um plano horizontal, consti- se move ao longo do eixo x em função do tempo. O mo-
tuindo um pêndulo cônico. Se o comprimento do fio é L vimento é iniciado em um instante t < t0 e em t0 ela
Seção 1. Múltipla escolha (10×0,5 = 5,0 pontos)
e o ângulo que o fio forma com a vertical é θ, o módulo se encontra na origem. Das seguintes afirmativas, estão
da velocidade v do corpo será: corretas:
1. Um caixote de massa m está em repouso apoiado sobre 3. Um corpo de massa m parte do repouso, a partir de uma (I ) a aceleração da partı́cula no instante t0 é nula; (II)
um plano inclinado de um ângulo θ. Se o coeficiente de altura h, deslizando sobre uma superfı́cie sem atrito, ! no instante t4 a partı́cula volta à origem; (III) existe
(a) v= Lg sen(θ)
atrito estático entre o bloco e a superfı́cie é µe , podemos atingindo uma velocidade v na base da superfı́cie. Um ! θ mais de um instante em que a aceleração é nula; (IV) a
afirmar que: outro corpo de massa 5m desce a mesma superfı́cie (b) v = Lg cos(θ) L partı́cula muda seu sentido de deslocamento nos instan-
de uma altura H, também partindo do repouso, atin- (c) v
!
= Lg tan(θ) m tes t0 , t2 e t4 ; (V) no ponto D sua aceleração é positiva;
(a) O módulo da força de atrito entre o caixote e a gindo a velocidade de 3v na base. A razão H/h é: ! (VI) o deslocamento entre t1 e t3 é maior que entre t1 e
(d) v = Lg/sen(θ) t2 .
superfı́cie vale µe m g cosθ !
(e) v = Lg tan(θ)sen(θ)
(b) A força normal sobre o caixote é a reação ao peso
m
deste caixote. (a) 3/5 (a) I, III e V v
(c) O módulo da força resultante que a superfı́cie do (b) 9/5 (b) III, IV e V
plano inclinado faz sobre o caixote vale m g cosθ (c) II, IV, VI
(c) 3 h
(d) " + f"e = P" , onde N
N " , f"e , P" são respectivamente D
(d) 9 (d) IV, V e VI t3
as forças normal, atrito estático e peso.
(e) 15 (e) I, IV, V t0 t1 t2 t4 t
(e) Nenhuma opção é verdadeira

8. Uma partı́cula executa um movimento pendular num


plano vertical xy, oscilando entre duas posições extre-
m mas A e B, como mostra a figura. Considerando o
θ trajeto A → B pode-se afirmar que:
4. Um bloco de massa m é abandonado sobre um trilho
contido em um plano vertical e desliza a partir do ponto
mais alto A, como representado na figura. Ao chegar no 6. Uma partı́cula é deslocada, sob a ação de uma força (a) nos pontos A e B a aceleração é nula;
ponto B, ele já iniciou seu movimento circular. Despre- conservativa F" , no percurso fechado A → B → C → A, (b) o vetor velocidade média entre A e B é nulo;
zando o atrito entre o trilho e o bloco, podemos afirmar como indica a figura. Afirma-se que para o tra-
(c) o vetor velocidade média entre A e B tem a
que: balho desta força nos trechos AB, BC e CA: I)
direção e o sentido do eixo x negativo;
WA→B + WB→C + WC→A = 0, II) WA→C = −WC→A ,
(a) O bloco tem velocidade máxima ao passar pelo (d) o vetor aceleração média entre A e B é nulo;
2. A figura mostra o gráfico da energia potencial, U (x), as-
ponto C. III) WA→C = WA→B + WB→C , IV) Como a força F" é
sociada à força resultante que atua sobre uma partı́cula conservativa o trabalho desta força é sempre positivo (e) o vetor aceleração média entre A e B tem a
que se move em uma dimensão, onde x0 corresponde à (b) O módulo da força normal no ponto B é igual ao em qualquer trecho e sentido do percurso. A opção direção e sentido do eixo y positivo.
posição na qual a energia potencial é mı́nima. De acordo módulo da força peso. abaixo correta para as afirmativas I), II), III) e IV) é
com a figura, afirma-se: (I) Na posição xA a força é (c) O módulo da velocidade no ponto E é o mesmo
nula. (II) Na posição x0 tem-se a condição de equilı́brio que no ponto C. y
(a) somente I) e II) estão corretas;
estável. (III) Para x > x0 a força aponta para a origem. (d) O módulo da velocidade no percurso B − C − D
(IV) quando a partı́cula está na posição xC , o módulo é constante. (b) somente II) e IV) estão corretas;
da força é maior que na posição xB . Estão corretas: (c) somente I), II) e III) estão corretas;
(e) A aceleração no ponto C é perpendicular à velo- A B
cidade. (d) somente I) e III) estão corretas; x
(e) todas estão corretas.
A m
(a) apenas (II), (III) e (IV)
(b) apenas (II) e (III) D
(c) apenas (I), (II) e (IV) E
C
(d) apenas (I), (II) e (III) B
(e) apenas (III) e (IV)

1 2
9. Dois pequenos blocos de massas m1 e m2 (m2 > m1 ) 10. Um conjunto de dois blocos de massas m1 e m2 é colo- Seção 2. Questões discursivas (2×2,5 = 5,0 pontos)
deslizam sobre uma cunha simétrica fixa no solo, como cado sobre um plano horizontal e liso (sem atrito). Ao
representa a figura. As duas rampas da cunha são incli- aplicar-se uma força F" horizontal de módulo constante Não serão consideradas respostas sem justificativa; expresse-as somente em função dos dados fornecidos.
nadas de um ângulo θ em relação à horizontal, e os dois sobre o bloco de massa m2 o conjunto move-se com ace-
blocos estão unidos por um fio inextensı́vel, que perma- leração constante de módulo a (não há movimento re- 1. A figura representa a trajetória do lançamento vertical de um projétil de massa m, a partir do solo, com velocidade
nece esticado durante todo o movimento. Num instante lativo entre os blocos), como mostra a figura. Se f é o inicial "v (0) = v0 ̂, com v0 > 0. Além de estar sujeito à aceleração da gravidade, durante o voo o projétil também
em que o bloco de massa m1 sobe uma das rampas com módulo da força de atrito entre os blocos, a opção correta sofre a ação de um vento que sopra na direção horizontal e que imprime uma aceleração constante adicional "av ao
velocidade de módulo v em relação ao solo, o vetor ve- é, projétil, da forma "av = a1 ı̂, onde a1 > 0. Nas soluções dos ı́tens abaixo, utilize quando necessário os unitários ı̂ e ̂ as-
locidade do bloco de massa m2 em relação ao de massa (a) f = m1 a; sociados ao sistema de eixos coordenados OXY, e as quantidades m, v0 , a1 e o módulo g da aceleração da gravidade local.
m1 é (b) f = (m1 + m2 )a; y
a) Determine os vetores velocidade "v (t) e posição "r (t) do projétil em
(a) −2 v senθ ̂ (c) f = |F" |;
função do tempo, durante seu voo.
(b) −2 v cosθ ı̂ (d) f = |F" | − m1 a;
(c) v (cosθ ı̂ − senθ ̂) b) Calcule a altura máxima atingida pelo projétil.
(e) f = |F" | − m1 g.
(d) 2 v senθ ı̂ c) Determine a que distância do ponto de lançamento o projétil atinge
(e) v (2 senθ ı̂ − cosθ ̂) o solo.
̂
d) Qual o trabalho realizado pela resultante das forças que atuam no
o ı̂ x
projétil, entre o instante de lançamento e o instante em que o
projétil atinge a altura máxima?
Resolução:
1. O projétil é submetido a uma aceleração resultante "a = "av + "g = a1 ı̂ − g ̂.

a)valor=1,0 ponto Considerando as condições iniciais do movimento, "r(0) = 0 e "v (0) = v0 ̂, e levando-se em conta
que a aceleração "a é constante,

"v (t) = "v (0) + "a t = v0 ̂ + (a1 ı̂ − g ̂) t = a1 tı̂ + (v0 − gt)̂

1 1 1 1
"r(t) = "r(0) + "v (0) t + "a t2 = v0 t̂ + (a1 ı̂ − g ̂) t2 = a1 t2 ı̂ + (v0 t − g t2 )̂
2 2 2 2
b)valor=0,5 pontos A altura máxima hmax é atingida no instante t = tm que satisfaz à condição vy (tm ) = 0:

v0 − g tm = 0 ⇒ tm = v0 /g .

Logo,
1 2 v2
hmax = y(tm ) = v0 tm − g tm = 0
2 2g
c)valor=0,5 ponto O projétil atinge o solo no instante t = tf > 0 que anula a coordenada y(t). Para determinar o
alcance do projétil, basta calcular x(tf ) :
1
y(tf ) = 0 ⇒ v0 − g tf = 0 ⇒ tf = 2 v0 /g
2
1 2 v02 a1
x(tf ) = a1 t2f =
2 g2
d)valor=0,5 ponto Pelo teorema trabalho-energia, o trabalho realizado pela força resultante F" que atua no projétil
entre os instantes t = 0 e t = tm é dado por
!)
(F 1 1 1 a2
W(0→tm ) = m [v(tm )2 − v(0)2 ] = m [(a1 tm )2 + (v0 − g tm )2 − v02 ] = − m v02 (1 − 21 )
2 2 2 g

3 4
2. A figura mostra três blocos A, B e C de massas respectivas mA , mB e mC . O bloco B está sobre uma mesa horizontal
com a qual tem coeficiente de atrito cinético µ. Nele estão presas as extremidades de dois fios ideais horizontais que Bloco A : T"A + P"A = mA"aA
passam por duas roldanas ideais, uma de cada lado do bloco, e as outras extremidades dos fios estão presas aos blocos
A e C. As massas dos blocos são tais que o bloco B sobre a mesa se desloca para a direita, com a aceleração também Bloco B : " + f"at + P"B + T" " = mB"aB
T"A" + N C
para a direita, de módulo desconhecido e denotada por "a, como indicado na figura; os fios permanecem tensos durante
todo o movimento. Considere como dados mA , mB , mC , µ e o módulo g da aceleração da gravidade. Bloco C : T"C + P"C = mC"aC

a) Faça o diagrama de forças sobre o bloco B, identificando cada uma das forças. Como os fios que ligam os blocos entre si são ideais temos: |T"A | = |T"A" | = TA , |T"C | = |T"C" | = TC
b) Determine o módulo a da aceleração do bloco B. e como os blocos adquirem a mesma aceleração |"aA | = |"aB | = |"aC | = a, podemos reescrever as equações na direção dos
c) Qual deve ser a massa mC do bloco C para que os blocos se desloquem com velocidades constantes? respectivos movimentos,


 TA − PA = mA a



(i) TC − TA − fat = mB a e (ii) N − mB g = 0




PC − TC = mC a

A solução do sistema de equações (i) permite obter a aceleração do bloco B, de (ii) e com a substituição de |f"at | =
" | = µmB g,
µ|N
(mC − mA − µmB )g
a=
(mA + mB + mC )

c) valor=0,3 ponto

Para que o bloco B mova-se com velocidade constante, a aceleração a dos blocos deve ser nula. Assim,

Resolução: (mC − mA − µmB )g


0= → mC = ma + µmB
(mA + mB + mC )
a) valor=0,5 ponto !
De acordo com a figura temos:

N" a força que a mesa exerce sobre o bloco B, P"B a força gravitacional exercida pela Terra, f"at a força de atrito entre
as superfı́cies do bloco e da mesa e T"A" e T"C" as reações das trações devido ao tracionamento dos fios pelos blocos A e C
respectivamente.

b) valor=1,7 pontos
O módulo a da aceleração do bloco B.

Aplicando a segunda Lei de Newton para cada bloco temos de acordo com as forças representadas no diagrama,

5 6
Questão 3) Um cilindro de massa M, raio R e momento de inércia I = (1/2)MR2 ,em relação ao
Instituto de Fı́sica - UFRJ seu eixo de rotação, está sobre uma mesa horizontal com atrito suficiente para poder rolar sem
deslizar. Uma corda inextensı́vel e de massa desprezı́vel é presa pelo eixo de rotação do cilindro.
Segunda Prova de Fı́sica IA - 2011/1 A corda passa por uma polia com inércia desprezı́vel e mantém-se esticada horizontalmente por
meio de um bloco de massa m pendurado verticalmente na sua outra extremidade; conforme
Obs: em todas as questões em que for necessário, considere que g é o módulo da aceleração da gravidade representado na figura abaixo.

Questão 1) Uma rã de massa m está apoiada sobre a extremidade de uma tábua homogênea de 
massa M e comprimento L em repouso sobre a superfı́cie de um lago congelado. Em um dado
instante a rã salta ao longo da tábua com uma velocidade de módulo v0 em relação à tábua,  
segundo um ângulo θ com a horizontal e mantendo a sua trajetória no plano vertical ZOX.  
Após o salto ela cai sobre a outra extremidade da tábua. Desprezando o atrito da superfı́cie do
lago, determine: 
a) o vetor velocidade da tábua logo após o salto;
b) o deslocamento ΔX da tábua após a rã atingir a extremidade da tábua;
c) o que aconteceria com a tábua se a rã retornasse à posição inicial do primeiro pulo? Justi-
fique a sua resposta.
a) Isole o bloco e o cilindro e faça um diagrama para as forças agindo sobre cada um deles
especificando-as.
b) Escreva as equações da Dinâmica do movimento do cilindro e do bloco;
c) Determine, em função dos dados M, R, m, g, a aceleração do centro de massa do cilindro e
o módulo da tração que age no cilindro;
Onde se lê: ....em relação à tábua. d) Qual é sentido da força de atrito que atua no cilindro? Justifique a sua resposta.
Mudar para: ....em relação ao referencial fixo na Terra ZOX,

Questão 2) Uma granada de massa M desloca-se no plano horizontal XY com velocidade V  Questão 4) A figura abaixo mostra, vista de cima, uma mesa horizontal sem atrito, onde uma
constante segundo a direção que faz um ângulo θ, desconhecido, com o eixo horizontal OX; esfera de mass m e outra de massa 2m estão em repouso, ligadas por uma haste de comprimento
conforme a figura abaixo. Ao passar pela origem O a granada explode em dois fragmentos de 2d e massa desprezı́vel, constituindo assim um haltere. Uma outra esfera de massa m move-se
massas m1 = (1/4)M e m2 = (3/4)M e o fragmento de massa m1 é observado após a explosão com velocidade vo segundo o eixo OX positivo em trajetória de colisão com o haltere. Após a
deslocando-se com velocidade v1 = 2V ao longo do eixo vertical OY no sentido negativo. O colisão o sistema esfera-haltere permanece unido. Tomando o instante da colisão como t = 0,
fragmento m2 é visto deslocando-se sobre o eixo OX com velocidade v2 desconhecida. Determine considerando desprezı́vel o tamanho de todas as esferas e usando o sistema de coordenadas
de acordo com os vetores unitários associados ao sistema de coordenadas indicado na figura: indicados na figura, determine:
a) o vetor velocidade, v2 , do fragmento de massa m2 após a explosão; a) as coordenadas xCM (t) e yCM (t) da posição do centro de massa do sistema em função do
b) o vetor velocidade, V   , do centro de massa do sistema formado pelos fragmentos após a tempo após a colisão;
explosão. Justifique a sua resposta; b) as expressões para o vetor momento angular do sistema, L O , em relação ao ponto O, antes
c) o ângulo θ. e depois do colisão.
c) o módulo da velocidade angular ω de rotação do sistema, após a colisão, que gira em torno
do seu centro de massa.


 

 

 


Questão 1

a) valor = (1,0 pontos)


Ao longo da direção X, não há forças externas. Como consequencia, podemos afirmar
que a aceleração do centro de massa CM, é nula ao longo deste eixo. O que implica que a
sua velocidade é constante, pois o momento linear nesta direção é conservado. Portanto,
segundo o referencial na Terra fixo ZOX, vx = −v0cos(θ) e V a velocidade da tábua.

x −mv0cosθ + MV x
VCM (antes do salto) = 0 = = VCM (depois do salto)
m+M
Logo,
m m
V = v0cos(θ) ⇒ V~ = v0cos(θ)ı̂
M M

b) valor = (1,0 pontos)


A conservação do momento linear do sistema, na direção X, rã-tábua nos informa que
se o centro de massa estava em repouso, ele permanecerá nesta posição quaisquer que
sejam os movimentos relativos entre a rã e a tábua nesta direção. Portanto a posição do
centro de massa CM é conservada.
Estabelecendo um sistema de coordenadas orientado a partir da posição do centro da
tábua, L/2, temos:

m(L/2) + M.0
xiCM =
m+M

m(−L/2 + ∆x) + M(+∆x)


xfCM =
m+M
A variação ∆x corresponde a quanto o centro da prancha se desloca e portanto o
quanto também a prancha se desloca quando a rã pula para a outra extremidade.
Como xiCM = xfCM
m
∆x = L
m+M

c) valor = (0,5 pontos)


A tábua retornaria à posição inicial. Observe que não importa o modo como a rã volta
à posição inicial, a força que a rã exerce sobre a tábua e a força que a tábua exerce sobre a
rã são internas ao sistema de partı́culas rã+tábua e não afetam o estado de movimento do
sistema. Sendo assim, o movimento da tábua e da rã é correlacionado de forma a preservar
a posição do centro de massa. Portanto quando a rã retorna para a extremidade original,
a tábua também retorna à sua posição original.

1
Questão 2

a) valor = (0,8 pontos)


O momento linear total é conservado, sob a condição de que a resultante das forças
externas é nula e a explosão decorre de forças internas que não afetam o estado de movi-
mento do sistema.
P~i = P~f
Segundo o sistema de coordenadas indicado na figura temos:

 P~i = −MV (cos θı̂ + sen θ̂)  |P~i |2 = Pi2 = M 2 V 2


 


 ~  ~ 2
Pf = ~p1 + ~p2 |Pf | = Pf2 = P~f ◦ P~f = p21 + p22 + 2~p1 ◦ p~2

Como ~p1 é a perpendicular ~p2 e |P~i | = |P~f |, M 2 V 2 = p21 + p22 = m21v12 + m22 v22,
s
2 2 2 2
M V − m v
1 1 M 2 V 2 − m21 v12
v22 = ⇒ v 2 = ±
m22 m22

Para os valores de v1 = 2V , m1 = (1/4)M e m2 = (3/4)M e de acordo com a figura


(granada no primeiro qaudrante) somente é possı́vel o sinal negativo de v2 , obtemos:
s
M 2 V 2 − (1/4)2 M 2 (2V )2 2 2
v2 = − 2
→ v2 = − √ V ⇒ ~v2 = − √ V ı̂
(3/4.M) 3 3

b) valor = (1,0 pontos)


A velocidade V~ 0 pode ser obtida de M V~ 0 = P~f , ou seja, M V~ 0 = ~p1 + p~2 . Portanto,
~p1 + ~p2
V~ 0 = , após a substituição dos valores de m1, m2, v1 e v2,
M
√ √
~ 0 (1/4)M(−2V )̂ + (3/4)M(−2)/ 3ı̂ ~ 0 3 1
V = ⇒ V =− V ı̂ − V ̂
M 2 2

Justificativa: a velocidade do centro de massa dos fragmentos ~v 0 somente pode ser


obtida pela condição de que o momento linear total é conservado. O centro de massa
dos fragmentos mantém a sua direção e sentido de propagação mesmo após a granada
explodir.
Observe que |V~ 0 | = |V~ | !!!

c) valor = (0,7 pontos)


Para calcular o ângulo θ devemos notar que a direção de propagação do centro de
massa não se altera, logo,

Pf Y p1 −(1/4)M2V 3
tan θ = = → tan θ = √ = ou θ = 30◦
Pf X p2 −(3/4)M2/ 3 3

2
Questão 3

a) valor = (0,6 pontos)


Veja o diagrama ao lado.

Escolhemos as orientações mostradas na figura. Para o CM do cilindro o eixo horizontal


X positivo no sentido cilindro para a polia e o eixo Z vertical para cima. O sentido positivo
da rotação será de OZ para OX. Para o bloco de massa m escolhemos um eixo Z 0 vertical
para baixo de acordo com o sentido do movimento do bloco.
Sobre o cilindro agem : o peso Mg, a reação normal N, a tração da corda T 0 e a força
de atrito estática fa.
Sobre o bloco, apenas : a tração na corda T , a mesma que T 0, pois a polia não tem
inércia , e o peso mg.

b) valor = (0,9 pontos)


Temos para o cilindro:
1)a rotação em torno do CM
Iα = Rfat i)
2)o movimento horizontal do CM

Max = T − fa ii)

Para o bloco
maz0 = mg − T iii)

c) valor = (0,8 pontos)


A condição de rolamento sem deslizamento implica em ax = Rα e az0 = ax . Obtemos
resolvendo o sistema de equações i) ii) e iii)
m
ax = g
I/R2 +M +m

(I/R2 + M)m
T = g
I/R2 + M + m
d) valor = (0,2 pontos)
mI/R2
fa = g
I/R2 + M + m
Como o sinal da fa calculado é positivo, o sentido adotado está fisicamente certo no
desenho.

3
Questão 4
a) valor = (1,0 pontos)
Antes da colisão para o sistema esfera-haltere temos,
m.v0 + m.0 + 2m.0 v0
vCM,x = =
4m 4
~ ~
Como não há atrito e P + N = ~0, a resultante das forças externas é nula. Portanto
há conservação do momento linear total do sistema, ou equivalentemente, a velocidade
do CM permanece constante. O resultado acima vale para qualquer instante de tempo t.
Depois da colisão, xCM (t) = xCM (0) + vCM,x t e como no momento da colisão, xCM (0) = 0,
para qualquer instante t temos:
v0 v0
vCM,x (t) = ; xCM (t) = t
4 4
vCM,y (t) = 0; yCM (t) = cte = 0

b) valor = (1,0 pontos)


Antes da colisão só a esfera livre se movimenta, em movimento retilı́neo uniforme. O
módulo do momento angular do sistema, L ~ a , em relação a O é:
O
~ a | = |~r × m~v0| = r⊥ mv0 = mdv0
|L O

Pela regra da mão direita, o vetor momento angular aponta para dentro da página.
Pelo sistema de coordenadas indicado, o eixo z deve apontar para fora da página. Logo:
~ a = −mdvok̂
L O

Depois da colisão, o sistema gira em torno do CM. Para cada massa m:


|~li/CM | = mi d2 ω
i

Também neste caso (rotação horária), o momento angular aponta para dentro da
página:
~d = ~li/CM ⇒ L ~ d = −(m + m)d2 ω k̂ − 2md2 ω k̂ = −4md2 ω k̂
X
LCM CM

O momento angular total do sistema é dado, em relação ao ponto O,


~ d = ~rCM × M~vCM + L
L ~d
O CM

Mas para o ponto O indicado, ~rCM e ~vCM são paralelos, de forma que o primeiro termo
se anula:
~d = L
L ~ d = −4md2 ω k̂
O CM

c) valor = (0,5 pontos)


Como não há torques externos e os torques envolvidos na colisão são internos ao
sistema, o momento angular total se conserva, L~f = L
~ i , ou seja, L
~a = L
~d .
O CM
Então de acordo com os resultados anteriores,
v0
−mdv0k̂ = −4md2 ω k̂ ⇒ ω =
4d

4
Instituto de Fı́sica - UFRJ

Segunda Prova de Fı́sica IA - 2011/2

Obs: em todas as questões em que for necessário, considere que g é o módulo da aceleração da gravidade

Questão 1) Dois blocos de massas m1 = m e m2 = 2m estão presos por uma barra fina, rı́gida e
de massa desprezı́vel. O sistema blocos-barra está em repouso, sobre uma mesa horizontal sem
atrito, na configuração mostrada na figura abaixo de acordo com o sistema de coordenadas YOX.
Num dado instante t0 = 0s aplicam-se simultaneamente duas forças F ~1 e F~2 nas massas m1 e m2
respectivamente. Estas forças têm módulos F1 = 4F e F2 = F e sentidos opostos sendo as suas
direções paralelas ao eixo horizontal OX. As forças aplicadas são mantidas constantes durante
um certo tempo t. Durante este tempo, considerando a translação do sistema, determine:
a) o vetor aceleração do centro de massa do sistema, ~aCM ;
b) as coordenadas do centro de massa do sistema RX (t) e RY (t) para um instante qualquer
entre t0 e t;
c) de acordo com o item anterior qual a trajetória do centro de massa do sistema;
d) Se substituirmos a barra por uma mola(também de massa desprezı́vel), o que acontece com
os resultados obtidos nos itens anteriores? Justifique a sua resposta.

Questão 2) Uma partı́cula de massa m e velocidade inicial ~u = uı̂, colide elasticamente com
outra de massa M, inicialmente em repouso no referencial do laboratório. Após a colisão, a
partı́cula de massa
√ m foi defletida por um ângulo de 90 , e o módulo de sua velocidade foi

reduzido para u/ 3. A partı́cula de massa M emerge da colisão com velocidade de módulo v,


numa direção que faz um ângulo θ com a direção ı̂. Determine:
a) o ângulo θ;
b) a razão λ = M/m e o valor de v;
c) o vetor velocidade do centro de massa do sistema, antes da colisão;
d) o vetor velocidade do centro de massa após a colisão.
e) Compare os resultados obtidos nos itens c) e d). Justifique a sua resposta.
Questão 3) Dois patinadores, cada um com massa m, deslizando sobre uma pista de gelo de
atrito desprezı́vel, aproximam-se um do outro com velocidades ~vA e ~vB de módulos iguais a
v e de sentidos opostos, segundo retas paralelas separadas por uma distância d, como esque-
maticamente mostra a figura. Adote o sistema de referência indicado na figura, com o eixo z
apontando perpendicularmente para fora da folha.

a) Obtenha o vetor momento angular do sistema em relação ao centro de massa.


b) Quando os patinadores estão frente à frente e a distância entre eles é d, estendem os braços
e dão-se as mãos, passando a girar em torno do centro de massa, mantendo entre eles a
distância d. Calcule a velocidade angular de rotação dos patinadores.
c) Calcule a energia cinética nesta situação.
d) Em um dado instante, os patinadores puxam-se um ao outro, reduzindo sua distância para
d/2. Qual a nova velocidade angular de rotação?
e) Qual a nova energia cinética? Ela se conserva? Por quê?

Questão 4) Um carretel de massa M e momento de inércia I = MR2 , em relação ao eixo que


passa longitudinalmente pelo seu centro de massa, é colocado sobre uma fita fina e rı́gida presa
à parede e ao chão inclinada de um ângulo θ em relação ao chão. O carretel possui raios internos
e externos iguais a r e R respectivamente. Ele parte do repouso e durante o seu movimento rola
sem deslizar e não tomba para os lados.
a) Escreva as equações de Newton para o movimento de translação e rotação do carretel.
b) Determine a aceleração do centro de massa do carretel.
c) Determine a inclinação máxima da fita para que o carretel role sem deslizar. Considere que
o coeficiente de atrito estático entre o carretel e a fita é igual µe .
Questão 1

a) valor = 1,0 pontos


O sistema está sujeito à ação de duas forças constantes durante o intervalo de tempo
[t0, t]: F~1 = 4Fı̂ e F~2 = −Fı̂. O movimento do centro de massa (CM) depende apenas
das forças externas e a resultante sobre o sistema blocos-barra é

F~ ext = F~1 + F~2 = 3Fı̂ (1)


Sendo a força resultante constante, a aceleração também o é. A segunda lei de Newton
para a translação do CM fica,

F~ ext = (m1 + m2) ~aCM


A massa total do sistema é m1 + m2 = 3m, portanto o vetor aceleração do CM é,
F
~aCM = ı̂ (2)
m

b) valor = 0,5 pontos


Como foi visto no item acima, a aceleração é constante e não apresenta componente
na direção Y. Integrando a equação 2 duas vezes com relação ao tempo, e de acordo com
~ CM (t) do CM no instante t ≥ t0 . Temos então
as condições iniciais, obtemos a posição R

~ CM (t) = R
R ~CM (t0)(t − t0) + 1 ~aCM (t − t0)2
~ CM (t0 ) + V (3)
2
Sabemos, contudo, que t0 = 0s e que o sistema parte do repouso, ou seja, a velocidade
inicial do CM, V~CM (t0), é zero. A posição inicial do CM é

~ CM (t0) = 1 d
R [m1r~1 (t0) + m2r~2 (t0)] = ̂ (4)
(m1 + m2 ) 3
de maneira que a posição do CM no instante t é

~ CM (t) = d ̂ + F t2ı̂
R (5)
3 2m

c) valor = 0,5 pontos


A equação 5 é vetorial, de forma que as equações horárias para as componentes RX (t)
e RY (t) do vetor posição do CM são:

1F 2 d
RX (t) = t e RY (t) = (6)
2m 3
onde vemos que a posição RY (t) do CM é constante, o que se deve ao fato de não haver
velocidade inicial nem aceleração na direção Y . Por outro lado, o movimento na direção
X é uniformemente acelerado. Assim a trajetória do CM (descrita pelo seu vetor posição)
d
no plano YOX é uma reta, paralela ao eixo X, atravessando o eixo Y em Y = .
3

1
Figure 1: Trajetória do centro de massa(CM) representada pela linha pontilhada.

d) valor = 0,5 pontos


A barra sem massa utilizada inicialmente produzirá forças sobre as massas adjacentes,
porém tais forças são internas ao sistema, não sendo capazes de alterar o estado de movi-
mento do CM. No caso de uma mola (sem massa) no lugar da barra, as respostas
anteriores não se alterariam, visto que as forças elásticas também seriam internas,
não interferindo no movimento do CM, sujeito somente à força externa resultante.

2
Questão 2

a) valor = (0,5 pontos)


O momento linear total conserva-se pois a resultante das forças externas sobre o sis-
tema é zero.

P~

= muı̂
 Antes


  (1)
~ u u
 PDepois = m √ ̂ + M (v cos θ ı̂ − v sen θ ̂) = Mv cos θ ı̂ + m √ − Mv sen θ ̂


3 3

Como o momento P~Antes = P~Depois, as componentes x e y da conservação do momento linear


são

x: mu = Mv cos θ, (2)
u u
y: 0 = Mv sen θ − m √ ⇒ m √ = Mv sen θ (3)
3 3
Para determinar θ, calcula-se (3)/(2):
Mv sen θ mu 1
=√ ⇒ tan θ = √ ⇒ θ = 30◦ . (4)
Mv cos θ 3mu 3

b) valor = (0,5 pontos)


A energia cinética conserva-se pois a colisão é elástica, KAntes = KDepois . Então
2
2u2
  
m 2 m u M 1 M 2
u = √ + v2 ⇒ 1− u2 = v ⇒ λv 2 = . (5)
2 2 3 2 3 m 3
Elimina-se θ das equações da conservação do momento linear calculando-se (2)2 + (3)2 :

4u2
 
2 2 2 2 2 1
u2 ⇒ λv 2 =

M v cos θ + sen θ = m 1 + . (6)
3 3λ
O valor de λ é obtido igualando as equações (5) e (6)

2u2 4u2
= (7)
3 3λ

λ = 2. (8)
Substituindo (8) em (5), encontra-se v:

2u2 u2 u
2v 2 = ⇒ v2 = ∴ v=√ . (9)
3 3 3

3
c) valor = (0,5 pontos)
A velocidade do centro de massa antes da colisão é calculada pela definição de veloci-
~CM = P m~vi/ P mi, e usando que λ = 2:
dade do centro de massa, V

muı̂ uı̂ ~ Antes = u ı̂ ~ Antes = u ı̂.


V~CM
Antes
= = ⇒VCM ⇒ VCM (10)
m+M 1 + M/m 1+λ 3

d) valor = (0,5 pontos)


Maneira 1
A velocidade do centro de massa após a colisão é calculada pela definição e usando os
resultados dos itens (a) e (b):

 
m √u3 ̂ + M (v cos θ ı̂ − v sen θ ̂) √u ̂ +λ √u cos 30◦ ı̂ − √u sen 30◦ ̂
3 3 3
~ Depois =
V =
CM
m+M 1+λ
 √ 
√u ̂ +2 √u × 3
ı̂ − √u × 1
̂  
3 3 2 3 2 u 1 u u
= = ı̂ + √ − √ ̂
3 3 3 3 3
u
= ı̂. (11)
3
Maneira 2
Neste caso, pode-se usar diretamente que como o momento linear se conserva, tem-se
que P~Antes = P~Depois .
Logo,
M V~ Antes = M V~ Depois CM CM
u
V~CM
Antes
= V~CM
Depois
= ı̂,
3
conforme calculado no item (c).

e) valor = (0,5 pontos)


V~CM
Antes
= V~CM
Depois
. A justificativa foi apresentada na segunda solução para o item (d). A
velocidade do centro de massa matém-se invariante independentemente do tipo de colisão!

4
Questão 3

a) valor = (0,5 pontos)


Inicialmente, o momento angular do sistema é L ~ = ~rA × ~pA + ~rB × p~B , onde ~rA e ~rB
são os vetores posição dos patinadores em relação ao centro de massa. Assim,

~ = −( mvd )k̂ − ( mvd )k̂ = −mvdk̂.


L
2 2

b) valor = (0,5 pontos)


Como não há torques externos(P~A + N~ A = ~0 e P~B + N
~ B = ~0) e a força radial que
atua em cada patinador não produz torque, o momento angular do sistema  se2conserva,
2
~ = −mvdk̂ = I~ω . Como o momento de inércia é I = 2 × m d = md ,
de modo que L
2 2
então,
2v
ω
~ = − k̂.
d

c) valor = (0,5 pontos)


A energia cinética é K = 21 Iω 2,
md2 2v
Temos do item anterior: I = e ~ω = − k̂.
2 d
Logo,

1 md2 2v
K= × ( )2 ⇒ K = mv 2.
2 2 d

d) valor = (0,5 pontos)


Mais uma vez o momento angular do sistema se conserva, pois não há torques externos.
Assim, I~ω = I 0~ω 0, onde I 0 e ~ω 0 são, respectivamente, o momento de inércia e a velocidade
2
md2
 
0 d
angular depois do puxão. O momento de inércia I = 2 × m = , de modo que
4 8
obtemos assim a velocidade angular,
8v
~ω 0 = 4~ω = − k̂.
d
e) valor = (0,5 pontos)
A nova energia cinética é K 0 = 12 I 0ω 02 = 4mv 2. A energia cinética aumenta, por conta
do trabalho realizado pelos patinadores ao puxarem-se um ao outro.

5
Questão 4

a) valor = (1.0 pontos)


~ peso P~ , e de atrito f~at ; vide
As forças que agem no carretel são as forças normal, N,
diagrama abaixo. As equações de Newton para translação e rotação são:

translação: N~ + P~ + f~at = M~acm


rotação: ~τN + ~τP + ~τfat = I α
~

b) valor = (1.0 pontos)


A relação entre a aceleração angular e a aceleração tangencial do carretel para a
rotação é dada por α~ × ~r = ~aT . Como o carretel rola sem deslizar o módulo da aceleração
tangencial |~aT | é igual ao módulo da aceleração do centro de massa do carretel |~acm |.
Portanto de acordo com o sistema de coordenadas indicado e os torques obtidos com
relação ao centro do carretel(~τP e ~τN se anulam) temos:

 (P senθ − fat)ı̂ = Macm ı̂ 
P senθ − fat = Macm
rf (−k̂) = I(−αk̂) ⇒
 at fatr = Iacm /r
αr = acm

A solução do sistema de equações da direita nos dá:


P senθ Mgsenθ r2 gsenθ
acm = = ∴ acm = .
(I/r + M)
2 (MR2 /r2 + M) (R2 + r2 )

c) valor = (0,5 pontos)


Na condição de inclinação máxima(θmax) o carretel estará no limite do deslizamento.
A força de atrito limite será dada por |f~at| = µe |N|.
~ Nesta situação |N|
~ = Mgcosθmax ,

2 r2 gsenθmax MR2
fat = µe Mgcosθmax = acm I/r ⇒ µe Mgcosθmax = × 2
(R2 + r2 ) r
(R2 + r2 )
tan θmax = µe .
R2

6
UNIVERSIDADE FEDERAL DO RIO DE JANEIRO

INSTITUTO DE FÍSICA

FÍSICA I – 2012/1

SEGUNDA PROVA (P2) – Turma EQN – 28/06/2012

VERSÃO: A

INSTRUÇÕES: LEIA COM CUIDADO!


1. Preencha CORRETA, LEGÍVEL E TOTALMENTE os campos em branco do cabeçalho do
caderno de resolução, fornecido em separado.
2. A prova constitui-se de duas partes:
• uma parte objetiva, perfazendo um total de 5,0 pontos, constituı́da por dez (10) questões
objetivas (de múltipla escolha), cada uma das quais valendo 0,5 ponto, sem penalização por
questão errada.
• uma parte discursiva, perfazendo um total de 5,0 pontos, constituı́da por duas (2) questões
discursivas (ou argumentativas ou dissertativas), cada uma das quais valendo 2,5 pontos.
3. Acima da tabela de respostas das questões objetivas, na primeira página do caderno de resolução,
INDIQUE CLARAMENTE A VERSÃO DA PROVA (A, B,. . . ).
4. O item considerado correto, em cada uma das questões objetivas, deve ser assinalado, A CANETA
(de tinta azul ou preta), na tabela de respostas correspondente do caderno de resolução
5. É vedado o uso de qualquer instrumento eletro-eletrônico (calculadora, celular, iPod, etc)
6. Seja organizado e claro.

Formulário

sen2 θ + cos2 θ = 1, sen2θ = 2senθcosθ


sen(α ± θ) = senαcosθ ± cosαsenθ, cos(α ± θ) = cosαcosθ ∓ senαsenθ
Z
d n xn+1
x = nxn−1 xn dx = (n 6= −1)
dx n+1
d d
senax = acosax, cosax = −asenax
dx dx

a b c
Lei dos senos: = =
senα senβ senγ
2 2 2
Lei dos cossenos: a = b + c − 2bc cosα

1
Seção 1. Múltipla escolha (10×0,5 = 5,0 pontos)

1. Na figura vê-se duas partı́culas, 1 e 2, inci- 4. Uma chapa quadrada de lado a é presa pelo seu
dindo horizontalmente sobre uma parede de centro por um pino fixo numa mesa horizon-
massa infinita. A partı́cula 1 colide elastica- tal, podendo girar livremente em torno dele.
mente, enquanto a 2 tem uma colisão total- Num dado instante três forças constantes e de
mente inelástica. Elas tem a mesma massa m mesmo módulo F agem sobre a chapa; con-
e velocidades iguais a ~v antes da colisão. A forme a figura. A opção correta para o módulo
duração do tempo de colisão para ambas é ∆t, do torque resultante e o sentido de rotação da
e |F~m1 | e |F~m2 | são as intensidades das forças chapa é:
médias que atuam em cada partı́cula durante (a) zero e não gira.
as respectivas colisões. A opção correta é:
(b) aF ; sentido horário.
(a) |F~m1 | = |F~m2 |
(c) (1/2)aF ; sentido horário.
(b) |F~m1 | < |F~m2 | √
(d) ( 2/2)aF ; sentido horário.
(c) |F~m1 | 6= 0 e |F~m2 | = 0 (e) (1/2)aF ; sentido anti-horário.
(d) |F~m1 | = 2|F~m2 |
(e) Nenhuma das respostas anteriores
2. Um disco de raio R e peso |P~ | = M g colo-
cado perpendicularmente sobre uma superfı́cie
horixontal, move-se sob a ação de uma força
horizontal F~ constante. Aplicada no seu cen-
tro de massa, tal força faz com que ele role sem
deslizar. Para a força de atrito atuando sobre
o disco, onde ICM = (1/2)M R2 , a resposta
correta é:
(a) |f~at | = µe M g; µe é coeficiente de atrito
estático
(b) |f~at | = µc M g; µc é coeficiente de atrito
cinético 5. Um homem de massa m encontra-se sobre uma
prancha de massa M (m 6= M ), em uma de
(c) |f~at | = 0 no ponto de contato do disco suas extremidades. A prancha está em re-
com a superfı́cie pouso sobre um lago congelado com atrito des-
(d) |f~at | = F , pois o disco rola sem deslizar prezı́vel. Num dado instante ele anda hori-
zontalmente de d sobre a prancha para o lado
(e) |f~at | = (1/3)|F~ |
oposto e para. Para um obervador em repouso
3. Um disco de massa M e raio R rola sem desli- fora da prancha a opção correta é:
zar sobre uma superfı́cie horizontal com velo-
(a) A velocidade do centro de massa
cidade angular ω e velocidade de translação
prancha-homem varia com o tempo.
vT constantes. A afirmativa correta sobre
o módulo da velocidade v, em determinadas (b) A posição do centro de massa do sis-
posições do disco, para um referencial fixo no tema prancha-homem não muda pois o
solo é: momento linear é conservado.
(a) Na base do disco v é nula (c) A prancha anda de d no sentido oposto
ao deslocamento do homem.
(b) No tôpo do disco é v = ωR
(d) Quando ele para a prancha continua em
(c) Na base do disco é v = ωR movimento.
(d) No centro do disco v é nula (e) Nada se pode afirmar pois o compri-
(e) No centro do disco v = (1/2)ωR mento da prancha não é conhecido

2
6. Uma granada move-se com velocidade ~v cons- 8. Livre de forças externas, dois blocos colidem
tante sobre uma mesa horizontal sem atrito. frontalmente. O primeiro bloco tem massa
Num dado instante ela explode em três frag- m1 = m e velocidade ~v e o segundo bloco massa
mentos de massas diferentes. Após a explosão m2 = 2m e velocidade −~v . Após a colisão eles
verifica-se que eles permanecem sobre a mesa ficam unidos. Se P~ é o momento linear total
deslocando-se com velocidades diferentes. A dos blocos e ~vCM é a velocidade do centro de
opção correta a respeito dos fragmentos é: massa dos blocos, a opção correta é:
(a) a posição do centro de massa dos (a) O momento linear P~ não é conservado
fragmentos varia com o quadrado do pois a colisão é inelástica.
tempo; (b) |~vCM | = 0 antes da colisão.
(b) a soma vetorial dos momentos lineares (c) |~vCM | = 0 depois da colisão.
dos fragmentos é nula;
(d) ~vCM = −(1/3)~v , antes e depois da co-
(c) a energia cinética total dos fragmentos lisão.
é nula;
(e) Nenhuma das respostas anteriores.
(d) o centro de massa move-se com veloci-
dade ~v ; 9. Um fio de massa desprezı́vel de comprimento
d é preso ao teto num ponto fixo O. Na sua
(e) nada se pode afirmar pois desconhece-
extremidade, um corpo de massa m gira em
se as direções de cada fragmento.
torno do eixo Z vertical com velocidade linear
v constante descrevendo um cone, onde α é o
ângulo que o fio faz com o eixo Z; vide a fi-
gura. O momento angular ℓZ deste corpo na
direção de Z em relação a O é:
(a) ℓZ = mvd(sen α)
(b) ℓZ = mvd(cos α)
(c) ℓZ = mvd
(d) ℓZ = mvd(tan α)
7. Uma chapa quadrada de lado 2a, homogênea (e) ℓZ = 2πd(sen α)mv
e de massa M , apresenta um corte de lado a.
Em relação ao sistema de coordenadas indi- 10. Uma esfera de massa M e raio R é colocada
cado na figura, o vetor R~ CM que representa o para girar em torno de um eixo Z ′ que tan-
seu centro de massa é igual a: gencia a sua periferia com velocidade angular
constante. Se o momento de inércia da esfera
2
(a) a(ı̂ + ̂) segundo um eixo que passa pelo seu centro é
3 I0 = (2/5)M R2 , a opção correta é:
1
(b) a(ı̂ + ̂) (a) IZ ′ = I0
3
1 (b) IZ ′ = (1/5)M R2
(c) a(ı̂ + ̂)
2 (c) IZ ′ = (7/5)M R2
1 1
(d) ( aı̂) + ( ̂) (d) IZ ′ = (7/10)M R2
3 2
5 (e) Nada se pode afirmar pois a velocidade
(e) a(ı̂ + ̂) angular não é conhecida.
6

3
Seção 2. Questões discursivas (2×2,5 = 5,0 pontos)

1. Uma esfera homogênea de massa M e raio R cujo momento de inércia I = (2/5)M R2 em relação
a um eixo que passa pelo seu centro, é colocada sobre um trilho como mostra a figura abaixo. A
trajetória entre A e B é reta e a distância AB é igual a d. Na posição A ela é liberada a partir do
repouso e rola sem deslizar até atingir a posição B. Do ponto B ao ponto C não há atrito e a altura
centro a centro entre A e C é igual a H. Determine justificando as respostas:
a) o trabalho da força peso e da força normal entre A e B;
b) a velocidade angular ωB da esfera na posição B;
c) o momento linear em C;
d) o momento angular em relação ao centro de massa da esfera na posição C.

2. Duas partı́culas movem-se no plano XOY , onde o atrito é desprezı́vel, com velocidades constantes.
A partı́cula 1 tem massa m1 e velocidade ~v1 enquanto a partı́cula 2 tem massa m2 e velocidade ~v2 .
As suas trajetórias e sentidos de movimento estão indicados na figura. Elas colidem no ponto O e
após a colisão permanecem unidas deslocando-se com velocidade V~ desconhecida, cuja direção faz
um âgulo β com a vertical. Determine justificando as respostas:
a) a módulo da velocidade do centro de massa das partı́culas antes da colisão;
b) o módulo da velocidade V~ após a colisão;
c) compare os resultados a) e b) anteriores. O que você pode concluir?
d) o ângulo β.

4
Universidade Federal do Rio de Janeiro
Centro de Ciências Matemáticas e da Natureza
Instituto de Fı́sica
Segunda Prova de Fı́sica IA - Turma EQN - 28/06/2012
Respostas para provas hı́bridas

Gabarito das Questões objetivas

Versão A Versão B
Questão (a) (b) (c) (d) (e) Questão (a) (b) (c) (d) (e)
1 1
2 2
3 3
4 4
5 5
6 6
7 7
8 8
9 9
10 10

Versão C Versão D
Questão (a) (b) (c) (d) (e) Questão (a) (b) (c) (d) (e)
1 1
2 2
3 3
4 4
5 5
6 6
7 7
8 8
9 9
10 10

1
Gabarito da Prova P2 da Disciplina Fı́sica IA: Questões Discursivas

I. QUESTÃO DISCURSIVA 1: 2.5 PONTOS

A. Item (a): 0.5 pontos

Escolhemos inicialmente um sistema de coordenadas tal que:

1. O eixo Ox seja paralelo ao trecho A − B e a coordenada x aumente da esquerda para a direita;

2. O eixo Oy seja perpendicular ao trecho A − B e a coordenada y aumente de baixo para cima;

3. O eixo Oz seja perpendicular à página e a coordenada z aumente “em direção ao leitor”.

Desta forma, o deslocamento total no trecho A − B é dado por

d~ = dı̂ . (1)

O peso da esfera P~ pode ser escrito como

P~ = M g sin θı̂ − M g cos θ̂ . (2)

Portanto, o trabalho WP realizado por P~ , que é constante, no trecho A − B é

WP = P~ · d~ = M gd sin θ .

Alternativamente, o resultado acima pode também ser obtido se nos lembrarmos que o trabalho realizado pelo
peso é igual a menos a variação da energia potencial gravitacional, WP = −∆Ugrav . Escolhendo um novo sistema de
coordenadas especificado por

1. O eixo Ox0 seja paralelo à superfı́cie da Terra;

2. O eixo Oy 0 seja perpendicular à superfı́cie da Terra e que a coordenada y 0 aumente de baixo para cima;

3. O eixo Oz 0 seja perpendicular à página e a coordenada z 0 aumente “em direção ao leitor”.

temos que Ugrav = M gy 0 . Posicionando a origem de forma que a coordenada y 0 do ponto B seja igual a zero, yB
0
= 0,
0
temos que yA = +d sin θ e assim

0 0
WP = −∆Ugrav = − (M gyB − M gyA ) = − [M g(0) − M g(d sin θ)] = M gd sin θ , (3)

resultado idêntico ao obtido anteriormente.


Por outro lado, a força normal exercida pela superfı́cie sobre a esfera é sempre perpendicular ao deslocamento d~ = dı̂
e portanto o trabalho Wn por ela realizado é nulo,

Wn = 0
2

B. Item (b) 0.7 pontos

Conforme vimos no item anterior, a energia potencial gravitacional no ponto A é igual a Ugrav,A = M gd sin θ. À
medida em que a esfera rola em direção ao ponto B, Ugrav,A é convertida em energias cinética de translação, Kt , e
de rotação, Kr . O rolamento da esfera é devido à existência de uma força de atrito estático entre a base da esfera e a
superfı́cie, que não realiza trabalho lı́quido sobre a esfera. Assim, como o trabalho devido à normal é nulo, como vimos
no item anterior, a energia mecânica total é conservada ao longo do trajeto A − B, EA = EB . Como EA = Ugrav,A e
EB = Kt,B + Kr,B , temos
2 2
M vcm,B Iωz,B
M gd sin θ = + , (4)
2 2

onde vcm,B é a velocidade do centro de massa (CM) da esfera no ponto B e ωz,B é a velocidade angular em relação
ao CM da esfera no mesmo ponto. Como não há deslizamento, tais grandezas estão relacionadas por

vcm,B = Rωz,B . (5)

Impondo esta condição à Eq. (4), e usando o dado I = (2/5)M R2 , obtemos

7
M gd sin θ = M R2 ωz,B
2
, (6)
10

e finalmente
r
1 10dg sin θ
ωz,B = .
R 7

C. Item (c): 0.7 pontos

A partir do ponto B não mais há atrito entre a base da esfera e a superfı́cie, de forma que o torque é nulo e assim a
velocidade angular (rotação em relação ao CM) no ponto C é igual àquela no ponto B, ωz,C = ωz,B . Desta forma, a
energia cinética de rotação permanece constante no trecho B − C e temos, usando o resultado do item anterior para
ωz,B , que a energia cinética de rotação no ponto C é dada por
2 2
Iωz,C Iωz,B M 10dg sin θ 2M dg sin θ
Kr,C = = = = , (7)
2 2 5 7 7

Por outro lado, o trabalho realizado pelo peso no trecho A − C é igual a menos a variação da energia potencial
gravitacional ao longo deste percurso, ou seja

0 0
WP = −∆Ugrav = −M g (yC − yA ) = M gH . (8)

Mas, como vimos, a única força a realizar trabalho sobre a esfera é justamente o seu peso, de forma que o trabalho
WP devido à esta força ao longo do trecho A − C é igual à soma da energia cinética de translação do CM no ponto
C, Kt,C = 21 M vcm,C
2
, e da energia cinética de rotação no mesmo ponto, dada pela Eq. (7). Ou seja,
2 2 2
M vcm,C Iωz,C M vcm,C 2M dg sin θ
WP = M gH = Kt,C = + = + . (9)
2 2 2 7
3

De forma a obter o módulo do momento linear do CM no ponto C, pcm,C = M vcm,C , isolamos vcm,C na equação
acima e obtemos
r
4dg sin θ
vcm,C = 2gH − . (10)
7

Por fim
r
4dg sin θ
pcm,C = M vcm,C = M 2gH − .
7

D. Item (d): 0.6 pontos

Como mencionamos no item anterior, no trecho B − C não há atrito e portanto a velocidade angular de rotação
em relação ao CM é constante, ωz,C = ωz,B . Evidentemente, o mesmo ocorre com o momento angular da esfera
em relação ao seu CM, Lcm,z,C = Lcm,z,B (note-se que a velocidade angular e o momento angular somente possuem
componente z, na direção perpendicular ao plano que contem a figura). Adicionalmente, o eixo de rotação, fixo, é
um eixo de simetria da esfera, de forma que Lcm,z,C = Iωz,C . Assim, usando o resultado anteriormente obtido para
ωz,C = ωz,B no item (b), concluı́mos que o módulo do momento angular em relação ao CM no ponto C é
r
2M R 10dg sin θ
Lcm,z,C = .
5 7

~ cm,C = −Lcm,z,C k̂ (aponta “para dentro da página”).


Note que a esfera gira no sentido anti-horário e assim L

II. QUESTÃO DISCURSIVA 2: 2.5 PONTOS

A. Item (a): 0.7 pontos

A velocidade da partı́cula 1 antes da colisão é ~v1 = v1 ı̂, enquanto que a da partı́cula 2 é ~v2 = v2 ̂. A velocidade do
centro de massa (CM) para um sistema de N partı́culas, i = 1, · · · , N , é definida por
P
mi~vi
~vcm = Pi . (11)
i mi

No presente caso N = 2 e as velocidades de cada uma das partı́culas já foi especificada. Assim, a velocidade do CM
antes da colisão é dada por

m1 v1 ı̂ + m2 v2 ̂
~vcm = , (12)
m1 + m2

cujo módulo é
p
m21 v12 + m22 v22
vcm = .
m1 + m2
4

B. Item (b) 0.7 pontos

Não há forças externas agindo sobre o sistema composto formado por 1 e 2. Desta forma, o momento total do
sistema, P~ é conservado: P~i = P~f . Antes da colisão temos

P~i = p~1 + p~2 = m1~v1 + m2~v2 . (13)

~ e portanto
Após a colisão completamente inelástica as partı́culas movem-se juntamente com velocidade V

P~f = p~cm = (m1 + m2 )V


~ . (14)

~ é então obtida a partir da conservação do momento total, P~i = P~f :


V

~ = m1 v1 ı̂ + m2 v2 ̂ ,
V (15)
m1 + m2

Finalmente,
p
m21 v12 + m22 v22
V = .
m1 + m2

C. Item (c): 0.4 pontos

Conforme dissemos anteriormente, não há forças externas atuando sobre o sistema. Como conseqüência, o momento
linear total do sistema é conservado. Para qualquer sistema composto, podemos identificar o momento linear total do
sistema, P~ , com o momento linear do centro de massa do mesmo, p~cm . Na presente situação, como P~ é constante, o
mesmo ocorrerá com p~cm , e logo a velocidade do centro de massa não se altera após a colisão. Verificamos tal fato
explicitamente nos itens anteriores, calculando: a velocidade do CM antes (a) e depois (b) da colisão na origem.

D. Item (d): 0.7 pontos

Podemos escrever, de acordo com a figura, que

~ = Vx ı̂ + Vy ̂ = V sin βı̂ + V cos β̂ ,


V (16)

ou seja, Vx /Vy = tan β. Por outro lado, da Eq. (15) temos

m 1 v1 m2 v2
Vx = e Vy = . (17)
m1 + m2 m1 + m2

Logo, tan β = (m1 v1 )/(m2 v2 ) e assim


 
m 1 v1
β = arctan .
m 2 v2
UNIVERSIDADE FEDERAL DO RIO DE JANEIRO
INSTITUTO DE FÍSICA
FÍSICA I – 2012/2
SEGUNDA PROVA(P2)-EQN – 21/02/2013
VERSÃO: A

Nas questões em que for necessário, considere que g é o módulo da aceleração da gravidade.

Seção 1. Múltipla escolha (10×0,5 = 5,0 pontos)

1. Duas partı́culas 1 e 2 deslocam-se ao longo 3. Dois corpos de momentos lineares p~1 e p~2 =
dos eixos OX e OY com os momentos line- −2~p1 , movem-se na ausência de forças exter-
ares p~1 = p1 ı̂ e p~2 = p2 ̂ respectivamente. nas e colidem num instante determinado t. Se
Elas colidem de forma totalmente inelástica antes da colisão ~vCM é a velocidade do centro
deslocando-se com a direção dada pelo ângulo de massa dos corpos, após a colisão a veloci-
β com o eixo OX. O ângulo β é dado por: dade do centro de massa das partı́culas:
(a) tan β = p1 /p2 (a) não permanece constante (é diferente
(b) tan β = p1 /(p1 + p2 ) de antes da colisão);

(c)
p
cos β = p2 / (p21 + p22 ) (b) permanece constante, mas a sua direção
p e sentidos não podem ser determinados;
(d) sen β = p2 / (p21 − p22 )
(c) permanece constante, mas somente o
(e) Nenhuma das opções anteriores. seu sentido é indeterminado;
(d) permanece constante, a sua direção é a
mesma de p~1 ;
(e) nenhuma das opções acima está correta.

2. Uma roda de raio R e momento de inércia I


em relação ao seu eixo de rotação, que passa
por O, está ligada por meio de um cabo ideal
a um corpo de massa m, como mostra a figura
abaixo. O sistema que estava inicialmente em 4. Um disco de raio a, homogêneo e de massa M
repouso é liberado. Após o bloco cair de uma encontra-se em repouso sobre um plano hori-
altura h, a velocidade angular do disco é dada zontal XY , sem atrito, com o seu centro de
por: massa localizado na origem O. Num dado ins-
r tante aplicam-se as forças F~2 = −F ̂, F~1 = Fı̂
2gh
(a) ; e F~3 = F~1 na borda do disco como mostra a
R2 figura. A opção que corresponde ao torque re-
r
2mgh sultante ~τ0 em relação ao ponto O, imediata-
(b) ;
I − mR2 mente após as forças serem aplicadas é:
r
mgh ~0 ;
(c) ; (a)
I + mR2
r (b) aF k̂ ;
mgh
(d) ; (c) −aF k̂ ;
I √
r
2mgh (d) a 2F k̂ ;
(e) √
I + mR2 (e) −a 2F k̂ ;

1
5. Duas partı́culas de massas iguais a m 7. Uma mola ideal de constante elástica k tem
deslocam-se no plano XY , com velocidades uma de suas extremidades fixada a uma pa-
~v1 = vı̂ e ~v2 = −~v1 . A direção da trajetória rede e a outra ligada ao centro de um disco,
da partı́cula 1 é paralela ao eixo OX e pas- O, de momento de inércia IO = (1/2)M R2 ;
sando pelo ponto a no eixo OY . A partı́cula vide a figura abixo. O disco está sobre uma
2 move-se sobre o eixo OX, como mostra a fi- superfı́cie horizontal e pode girar livremente
gura abaixo. O momento angular do sistema em torno do ponto O. A mola é esticada de
em torno de O é: ∆ℓ, em relação à sua posição relaxada. Ao ser
liberada, com o disco em repouso, a mola puxa
o disco de tal forma que ele rola sem deslizar.
A velocidade angular do disco quando a mola
estiver na condição de relaxamento é dada por:
(a) 0;
r
k
(b) ∆ℓ;
IO
r
2k
(c) ∆ℓ;
IO
(a) ~0; r
k
(b) −amv k̂; (d) ∆ℓ;
3IO
(c) −2amv k̂; r
k ∆ℓ
(e) ;
(d) −amvı̂; IO 2
(e) nenhuma das respostas anteriores.

6. Uma barra de comprimento L e massa M


uniformemente distribuida, encontra-se ini-
cialmente em repouso na posição vertical.
A barra está presa na sua base, mas pode
girar livremente em um plano vertical (veja
a figura). Como a posição vertical é uma
situação de equilı́brio instável, após um breve
intervalo de tempo, a barra começará a cair.
Despreze o atrito com o eixo de rotação na
base e a presença do ar. A velocidade angular
com que a barra chega ao solo é: (Dado 8. Um cilindro de massa M e raio R rola
2
ICM = M12L ). sem deslizar sobre uma superfı́cie horizontal.
r
g Sabendo-se que o momento de inércia do cilin-
(a) ;
2L dro em relação ao seu eixo de simetria é igual
r a (1/2)M R2 , pode-se afirmar que a razão en-
3g
(b) ; tre a energia cinética de rotação do cilindro e
L
r a energia cinética de translação é igual a:
3g
(c) 2 ; (a) 1/2
L
r (b) 1/4
3g
(d) ; (c) 1/3
2L
r
g (d) 3/4
(e) ;
3L (e) 1

2
9. Uma chapa metálica quadrada de lado a, ho- 10. Dois discos de raios ra e rb e de mesma massa
mogênea e de massa M encontra-se em re- M estão ligados por uma correia C que passa
pouso sobre um plano horizontal XY , sem por suas periferias como mostra a figura; a cor-
atrito, com o seu centro de massa localizado reia não desliza sobre os discos. O disco de raio
na origem O. Num dado instante aplicam-se rb gira com velocidade angular ωb constante em
as forças F~2 = F ̂ e F~1 = Fı̂ em dois vértices torno de seu eixo de rotação. Com relação a
da chapa como mostra a figura. A opção energia cinética de rotação de cada disco Krot e
que corresponde a aceleração do movimento suas velocidades angulares ω, a opcção correta
de translação a chapa, imediatamente após as é: (dado: o momento de inércia de um disco
forças serem aplicadas é: de raio R e massa M para um eixo que passa
pelo seu centro ICM = (1/2)M R2 ).
(a) ~0;
F
(b) ı̂;
M
2F
(c) (̂ − ı̂)
M
F
(d) (ı̂ + ̂)
2M (a) ωa = ωb
F
(e) (ı̂ + ̂) (b) Krot(a) = Krot(b)
M
(c) ωa > ωb
(d) Krot(a) < Krot(b)
(e) nenhuma opção acima é correta

3
Seção 2. Questões discursivas (2×2,5 = 5,0 pontos)

1. Um bloco A, de massa m, desliza sobre uma superfı́cie horizontal com velocidade ~v = vı̂. Ele se
aproxima de um bloco B, que tem massa m e velocidade −~v , ao qual está presa uma mola com
constante elástica k e de massa desprezı́vel e no seu estado relaxado. como indicado na figura. Uma
colisão se inicia quando o bloco A toca na mola. A partir deste instante, a mola vai sendo comprimida
até chegar à compressão máxima, ∆X. Neste instante, os dois blocos têm momentaneamente a
mesma velocidade. Em seguida, a mola vai voltando ao seu comprimento original e os blocos vão se
separando. Finalmente, o bloco A perde o contacto com a mola quando ela atinge novamente o seu
estado relaxado. Depois disso, os dois blocos se movimentam livremente. Durante todo o processo
de colisão, o atrito entre os blocos e a superfı́cie horizontal é desprezı́vel.
a) Calcule a velocidade do centro de massa do sistema antes e após a colisão.
b) Calcule o valor da compressão máxima da mola ∆X.
c) Calcule a energia cinética total do sistema (blocos A e B) após a colisão.

2. Uma esfera homogênea e uniforme, de massa M e raio R, é colocada em repouso numa calha na
posição A, cujo centro está a uma altura h em relação ao trecho horizontal; como mostra a figura.
Na calha o trecho A-B-C possui atrito e ao ser liberada a esfera rola sem deslizar ao se deslocar de
A até C. No trecho C-D não há atrito. Sabendo-se que o momento de inércia da esfera em relação
ao eixo de rotação que passa pelo seu centro é I = (2/5)M R2 , determine:
a) O trabalho exercido por cada a força: peso, normal e atrito sobre a esfera desde A até C;
b) o módulo da velocidade do centro de massa da esfera ao passar pelo ponto B, localizado na base
da calha;
c) a velocidade angular da esfera entre os C e D. Justifique resumidamente a sua resposta.

4
Universidade Federal do Rio de Janeiro
Centro de Ciências Matemáticas e da Natureza
Instituto de Fı́sica
Segunda Prova de Fı́sica IA - 21/02/2013 - EQN
Respostas para provas hı́bridas

Gabarito das Questões objetivas - 0,5 ponto cada questão

Versão A Versão B
Questão (a) (b) (c) (d) (e) Questão (a) (b) (c) (d) (e)
1 1
2 2
3 3
4 4
5 5
6 6
7 7
8 8
9 9
10 10

Versão C Versão D
Questão (a) (b) (c) (d) (e) Questão (a) (b) (c) (d) (e)
1 1
2 2
3 3
4 4
5 5
6 6
7 7
8 8
9 9
10 10

1
Questão discursiva 1) - 2,5 pontos
a) valor=0,5 ponto
Antes da colisão, temos que:

m~v + m(−~v) ~
V~CM = =0
2m

Como não há forças externas, a velocidade do centro de massa permanece constante, logo
após a colisão:

m~v + m(−~v) ~
V~CM = =0
2m

b) valor=1,0 ponto
A mola tem compressão máxima quando os blocos têm mesma velocidade e igual a V ~CM .
Apenas a força elástica da mola realiza trabalho. A energia mecânica do sistema é con-
servada. A energia mecânica antes da colisão é dada por,
1 1
Ei = mv 2 + mv 2 = mv 2
2 2

e após a colisão,
1
Ef = k(∆X)2
2

Como ∆E = 0, dessa forma,


r
2mv 2 2m
(∆X)2 = → ∆X = v
k k

c) valor=1,0 ponto
Durante a colisão, a energia mecânica se conserva, portanto, após a colisão, quando não
há mais energia elástica, a energia cinética é restaurada permanecendo igual a energia
cinética antes da colisão. Logo,

Kf = mv 2

2
Questão discursiva 2) - 2,5 pontos
a) valor=1,0 ponto
A normal e a força de atrito não realizam trabalho.
O trabalho do peso depende da altura da queda do centro de massa da esfera, logo:

Wpeso = Mg(h − R)

b) valor=1,0 ponto
No trecho AB, a esfera rola sem deslizar, logo

VCM = ωR

Nesse trecho, apenas o peso realiza trabalho, aplicando o Teorema Trabalho-energia:

Wpeso = Kf − Ki

Como a esfera é liberada do repouso, Wpeso = Kf . Portanto:


1 2 1
Mg(h − R) = MVCM + Iω 2
2 2
Como a esfera rola sem deslizar VCM = ωR; subsituindo o valor do momento de ineércia
da esfera.

1 12 V2
2
Mg(h − R) = MVCM + MR2 CM
2 25 R2
7 2
Mg(h − R) = MVCM
10
r
10
∴ VCM = g(h − R)
7

c) valor=0,5 ponto
Entre os pontos B e C não há forças realizando trabalho sobre a esfera, logo a energia
cinética translacional e rotacional da esfera permanecem constantes. Entre os pontos C e
D, o mesmo pode ser afirmado, logo a velocidade angular da esfera nesse trecho é mesma
que ela tem no ponto B.
r
VCM 10 g(h − R)
ω= =
R 7 R2

3
3. Um trenó de massa M move-se horizontalmente com 5. Uma bola desloca-se com velocidade ~v constante e co-
UNIVERSIDADE FEDERAL DO RIO DE JANEIRO velocidade constante em um lago congelado. Ao pas- lide elasticamente na lateral de uma mesa, segundo o
INSTITUTO DE FÍSICA sar por baixo de um viaduto, cai sobre ele vertical- ângulo α definido entre a direção de seu movimento e
mente, um pacote de massa m ficando preso a ele. O a lateral da mesa e é claro que |~v | = |~v ′ |; como mos-
FÍSICA I – 2012/2 atrito entre a superfı́cie do lago e o trenó pode ser tra a figura. O processo de colisão dura um tempo
SEGUNDA PROVA(P2) – 18/02/2013 desprezado. Ao comparar-se, o momento linear, a ∆t. Na figura estão indicadas 4 setas numeradas. A
velocidade e a energia cinética do sistema trenó- opção correta para representar a força média que atua
VERSÃO: A pacote após o pacote cair sobre o trenó, com estas na bola durante a colisão é:
mesmas grandezas na situação inicial do trenó, é cor-
reto afirmar que:
Nas questões em que for necessário, considere que g é o módulo da aceleração da gravidade.
(a) aumenta, diminui e aumenta.
(b) aumenta, permanece constante e aumenta;
Seção 1. Múltipla escolha (10×0,5 = 5,0 pontos) (c) diminui, diminui e diminui;
(d) permanece constante, permanece constante e
1. Um pai de massa M brinca com seu filho de massa m 2. Uma esfera de massa M , raio R e momento de inércia permanece constante; (a) 4;
na superı́cie horizontal de um lago congelado, onde o I = (2/5)M R2 , em relação ao eixo que passa pelo
atrito pode ser desprezado; considere M > m. Cada seu centro de massa, desce um plano inclinado A-B (e) permanece constante, diminui e diminui; (b) 2;
um segura uma das extremidades de uma corda leve (ver figura), rolando sem deslizar. A esfera parte do (c) 3;
e comprida. Eles devem puxar a corda para conseguir repouso e desce até o final do plano inclinado (ponto
(d) 1;
pegar uma bola, colocada na metade da distância en- B), situado a uma altura h em relação ao solo. Neste
tre os dois. Qual afirmação é verdadeira quando am- instante, ela tem velocidade angular ω0 , e entra em (e) nenhuma das respostas anteriores.
bos puxam a corda? queda livre. Afirma-se que: I) Imediatamente an-
tes de tocar
p o solo, a velocidade angular da esfera
(a) O resultado dependerá do esforço que cada um é ω = ω02 + 10gh/7R2 . II) Imediatamente antes
fará. de tocar o solo, a velocidade angular da esfera con-
(b) O pai, alcançará a bola primeiro. tinua com o valor ω0 . III) O momento angular da
(c) O filho alcançará a bola primeiro, qualquer que esfera mantém-se perpendicular ao plano do movi-
seja seu esforço. mento. IV) Ao longo da queda livre, a energia cinética 6. Um menino, de massa m, encontra-se na extremidade
de rotação da esfera permanece constante. Qual(ais) esquerda de um barco de comprimento L e massa
(d) Os dois sempre alcançarão a bola ao mesmo da(s) afirmação(ões) acima está(ão) incorreta(s)? M , distribuı́da homogeneamente. Inicialmente o con-
tempo.
junto, barco e menino, move-se para a direita com
(e) Nenhuma das afirmações anteriores. (a) Somente a I. velocidade constante V~1 , em relação a um referencial
(b) Somente a II. inercial colocado na margem do lago. Num dado ins-
(c) I e IV. tante o menino move-se para a extremidade direita
do barco, onde permanece. Nesta situação o conjunto
(d) II e III. move-se com velocidade constante V~2 em relação ao
4. Um bloco A deslocando-se com momento linear p~A =
(e) I e III. (aı̂ − a̂), colide com um bloco B de momento linear mesmo referencial inercial. Desprezando-se o atrito
p~B = (aı̂ + a̂), onde a é uma constante. Suponha entre o barco e a água, pode-se afirmar, ao fim do
que não há força externa. Após a colisão o centro de processo, que:
massa do sistema segue, de acordo com o sistema de (a) |V~1 | =
6 |V~2 |.
coordenadas ortogonais XY Z e os respectivos vetores
unitários ı̂, ̂ e k̂: (b) |V~1 | = |V~2 |.

(a) na direção x e sentido negativo; (c) |V~2 | = 0.

(b) na direção y e sentido negativo; (d) Não se pode determinar |V~2 |, pois não se co-
nhece a força exercida pelo menino sobre o
(c) na direção x e sentido positivo; barco.
(d) na direção y e sentido positivo; (e) Não se pode determinar |V~2 |, pois não se co-
π
(e) em uma direção formando um ângulo 4
com os nhece a velocidade inicial do centro de massa
eixos OX e OY . do sistema barco-menino.

1 2
7. Um caracol de massa m encontra-se inicialmente em 9. Uma chapa metálica quadrada de lado a, homogênea Seção 2. Questões discursivas (2×2,5 = 5,0 pontos)
repouso na extremidade, ponto O, de uma barra rı́gida e de massa M encontra-se em repouso sobre um plano
e homogênea de comprimento L (conforme a figura), horizontal XY , sem atrito, com o seu centro de massa
1. Uma partı́cula de massa m move-se inicialmente com velocidade de módulo u ao longo de uma trajetória retilı́nea
que gira com velocidade angular ω0 em torno de um localizado na origem O. Num dado instante aplicam-
que forma um ângulo de θ1 com o eixo X, como mostra a figura. Ela colide com uma segunda partı́cula que tem
eixo fixo z que passa pelo ponto O, perpendicular- se as forças F~2 = −F ̂ e F~1 = F ̂ em dois vértices
duas vezes sua massa, isto é m2 = 2m, que está movendo-se ao longo da direção X e sentido positivo com velocidade
mente a barra horizontal. Em um certo instante, o da chapa como mostra a figura. A opção que corres-
de módulo três vezes maior que u, ou seja: v2 = 3u. Depois da colisão, a primeira partı́cula move-se ao longo da
caracol desloca-se até a extremidade oposta da barra, ponde ao torque resultante ~τ0 em relação ao ponto O,
direção Y e sentido negativo, enquanto que a segunda partı́cula continua a mover-se ao longo do eixo X e sentido
ponto B, onde permanece. Nesta situação a barra gira imediatamente após as forças serem aplicadas é:
positivo, com uma velocidade escalar reduzida. Considere que não atuam forças externas sobre as partı́culas. Se
com velocidade angular ω ′ . Sendo I0 , o momento de
(a) ~0; na solução de algum item abaixo for usada alguma lei de convervação, justifique-a.
inércia do sistema com relação ao eixo z quando o ca-
racol está no ponto O e IB o momento de inércia do (b) aF k̂; a) Determine o vetor momento linear total inicial para este sistema de duas partı́culas;
sistema com relação ao eixo z quando o caracol está √ b) Calcule os módulos das velocidades de ambas as partı́culas depois da colisão em função dos dados do problema;
(c) a 2F k̂; c) Determine o vetor velocidade do centro de massa do sistema antes e depois da colisão.
no ponto B, das opções abaixo, a correta é:
(d) −aF k̂; Dados: cos θ1 = 3/5 ; sen θ1 = 4/5.

(a) IB > I0 ; ω < ω0

(e) −a 2F k̂.
(b) IB > I0 ; ω ′ > ω0 10. Um disco de massa M rola sem deslizar, com veloci-
(c) IB < I0 ; ω ′ > ω0 dade angular constante, sobre uma superfı́cie horizon-
tal, mantendo-se sempre alinhado verticalmente. Nos
(d) IB < I0 ; ω ′ < ω0
diagramas abaixo, I), II), III) e IV), aquele que repre-
(e) IB = I0 ; ω ′ = ω0 senta corretamente os vetores velocidades (indicados
8. Um disco uniforme é montado em um eixo horizon- por setas) dos pontos A e C, da periferia do disco e B
tal de massa desprezı́vel, em torno do qual pode girar o seu centro, em relação a um observador parado na
livremente, como mostra o diagrama. O momento Terra é:
de inércia do disco em relação ao eixo é I. O disco
está inicialmente girando com velocidade angular ω0 ;
vide figura. O disco é movido horizontalmente para
esquerda até que sua face entre em contato com um
disco idêntico, que está inicialmente em repouso. De- 2. Uma barra delgada homogênea e uniforme, de massa M e comprimento ℓ, é articulada num pino O, que passa por
pois de um pequeno intervalo de tempo, os dois discos uma de suas extremidades, e pode girar sem atrito num plano vertical. Liberada a partir do repouso na posição
acoplam-se e passam a girar juntos com uma veloci- horizontal, a barra sofre uma colisão elástica com um pêndulo ao passar pela posição vertical, como representa a
dade angular final ωf . Em relação às grandezas: velo- figura abaixo. O pêndulo é formado por uma haste muito fina de massa desprezı́vel, comprimento (2/3)ℓ, e uma
cidade angular final, energia cinética da rotação massa m de dimensões desprezı́veis presa à extremidade da haste. O pêndulo também pode oscilar em torno do
e momento angular do sistema, pode-se afirmar pino, sem atrito, no mesmo plano vertical que a barra. Sabe-se que o pêndulo encontrava-se em repouso antes da
que ao final do processo de acoplamento: colisão com a barra, e que o momento de inércia da barra em relação ao eixo perpendicular ao plano vertical e que
passa pelo pino é I = (1/3)M ℓ2 .
a) Determine a velocidade angular da barra, ω0 , imediatamente antes da colisão com o pêndulo.
b) Quais das seguintes grandezas: o momento linear e o momento angular em relação a O são conservadas na
colisão? Justifique a sua resposta.
c) Determine a velocidade angular da barra, ω ′ , imediatamente após a colisão.
(a) nenhum deles
d) Determine o valor da razão m/M para que a barra permaneça em repouso depois da colisão.
(b) I
(c) II
(a) ωf = ω0 /2, não é conservada, não é conservado;
(d) III
(b) 2ωf = ω0 , é conservada, não é conservado;
(e) IV
(c) ωf = ω0 , é conservada, não é conservado;
(d) ωf = ω0 /2, não é conservada, é conservado;
(e) ωf = ω0 /2, é conservada, é conservado.

3 4
Universidade Federal do Rio de Janeiro
Centro de Ciências Matemáticas e da Natureza
Instituto de Fı́sica
Segunda Prova de Fı́sica IA - 18/02/2013
Respostas para provas hı́bridas

Gabarito das Questões objetivas - 0,5 ponto cada questão

Versão A Versão B
Questão (a) (b) (c) (d) (e) Questão (a) (b) (c) (d) (e)
1 1
2 2
3 3
4 4
5 5
6 6
7 7
8 8
9 9
10 10

Versão C Versão D
Questão (a) (b) (c) (d) (e) Questão (a) (b) (c) (d) (e)
1 1
2 2
3 3
4 4
5 5
6 6
7 7
8 8
9 9
10 10

1
Questão discursiva 1) - 2,5 pontos
a) valor=1,0 ponto
O momento linear total inicial do sistema composto pelas duas partı́culas é dado por:

P~ inicial = p~inicial
1 + p~inicial
2
= m~u + 2m~v2
= m (−u cos θ1 ı̂ − u sen θ1 ̂) + 2m(3uı̂)
= (−mu cos θ1 + 6mu) ı̂ − musen θ1 ̂.
 
3mu 4mu
= − + 6mu ı̂ − ̂.
5 5
27mu 4mu
= ı̂ − ̂. (1)
5 5

b) valor=1,0 ponto
Na ausência de forças externas, o momento linear total do sistema conserva-se: P~ inicial =
P~ final . Comecemos calculando o momento linear total do sistema após a colisão

P~ final = p~final
1 + p~final
2 = −mv1f ̂ + 2mv2f ı̂, (2)
onde v1f e v2f são os módulos das velocidades, após a colisão das partı́culas 1 e 2 respec-
tivamente. Como o momento linear conserva-se:

P~ inicial = P~ final
27mu 4mu
ı̂ − ̂ = 2mv2f ı̂ − mv1f ̂
5 5

27mu 27
para a componente de ı̂ : = 2mv2f =⇒ v2f = u. (3)
5 10
4mu 4
para a componente de ̂ : − = − mv1f =⇒ v1f = u. (4)
5 5

c) valor=0,5 ponto
Como o momento linear conserva-se a velocidade do centro de massa do sistema não se
altera devido à colisão. Podemos calcular a velocidade do centro de massa antes (ou
depois) da colisão e argumentar que elas são iguais.

~ inicial
P~ inicial 27mu
5
ı̂ − 4mu
5
̂ 9u 4u
VCM = = = ı̂ − ̂. (5)
m + 2m 3m 5 15
~
P final 27mu 4mu
ı̂ − 5 ̂ 9u 4u
V~CMfinal
= = 5 = ı̂ − ̂ = V~CMinicial
. (6)
3m 3m 5 15

2
Questão discursiva 2) - 2,5 pontos
a) valor=1,0 ponto
Até ocorrer a colisão, a única força que realiza trabalho sobre a barra é o peso, que é uma
força conservativa. Por conservação de energia mecânica,

(1/2)Iω02 + M g`/2 = M g`
p p
Iω02 = M g` =⇒ ω0 = M g`/I = 3g/`

b) valor=0,5 ponto
A colisão não altera o momento angular total do sistema barra+pêndulo em relação ao
pino, pois o torque externo total em relação ao pino é nulo durante o processo de colisão.

c) valor=0,5 ponto

Como a colisão é elástica,


2
(1/2)Iω 0 + (1/2)mv 2 = (1/2)Iω02
2
I(ω02 − ω 0 ) = mv 2
Expandindo a equação acima

I(ω0 + ω 0 )(ω0 − ω 0 ) = mv 2 → (i)

Como o momento angular é conservado temos:

Iω 0 + mv · (2/3)` = Iω0

I(ω0 − ω 0 ) = (2/3)mv` → (ii)


Dividindo a equação (i) pela (ii),

ω0 + ω 0 = (3/2)v/` → (iii)

Após a susbstituição de I = (1/3)M `2 na equação ii):

ω0 − ω 0 = 2mv/M ` → (iv)

As equações (iii) e (iv) formam um sistema de equações lineares nas incógnitas ω 0 e v;


 0
 w + 2(m/M )v/` = w0

w0 − (3/2)v/` = −wo

A solução do sistema de equações leva a,


1 − (4/3)m/M 1 − (4/3)m/M p
ω0 = ω0 = 3g/`
1 + (4/3)m/M 1 + (4/3)m/M

3
d) valor=0,5 ponto
(maneira 1)
A expressão encontrada no item anterior mostra que ω 0 se anula para

1 − (4/3)m/M = 0 ⇒ m/M = 3/4

(maneira 2)
Calculemos a energia cinética do sistema imediatamente antes e depois da colisão, im-
pondo a condição que a barra permanece em repouso após a colisão e que o pêndulo
adquire velocidade v.
1 1
Ki = I0 ω 2 e Kf = mv 2
2 2

Como a energia cinética é conservada ∆K = 0, pois a colisão é elástica,

I0 ω02 = mv 2 (v)

O momento angular é conservado, de acordo com a equação (ii) e impondo a condição de


que a velocidade angular da barra após a colisão ω 0 é nula, temos,
2 3 I0 ω0
Io ω0 = m`v ∴ v=
3 2 m`

Substituindo o valor de v obtido anteriormente na equação (v):


 3 I ω 2
0 0
I0 ω02 = m
2 m`
Após algumas simplificações e subsituindo-se o valor de I0 , obtemos,

m/M = 3/4

4
3. Um aro elı́ptico homogêneo de massa M é posto para 5. Três partı́culas de massas m1 , m2 e m3 diferentes
UNIVERSIDADE FEDERAL DO RIO DE JANEIRO girar com velocidade angular ω ao redor do seu eixo movem-se com a mesma velocidade vetorial ~v cons-
INSTITUTO DE FÍSICA maior, em relação ao qual tem momento de inércia I1 . tante relativas a um dado referencial. Sendo K a ener-
Em seguida, é colocado para girar com mesma veloci- gia cinética do sistema e KCM a energia cinética do
FÍSICA I – 2013/1 dade angular ao redor de seu eixo menor, em relação seu centro de massa, relativas a esse referencial iner-
SEGUNDA PROVA – 15/07/2013 ao qual tem momento de inércia I2 . Veja as figuras. cial, e K ′ sua energia cinética relativa ao centro de
Sobre os módulos dos momentos angulares L1 e L2 dos massa, é correto afirmar que:
VERSÃO: A aros nas respectivas situações 1 e 2, podemos afirmar
(a) K = K ′ − KCM com K ′ e KCM ambas dife-
que:
rentes de zero;

Nas questões em que for necessário, considere que g é o módulo da aceleração da gravidade. (b) K = K ′ + KCM com K ′ e KCM ambas dife-
rentes de zero;
(c) KCM é sempre nula;
Seção 1. Múltipla escolha (10×0,5 = 5,0 pontos)
(d) K ′ = KCM ;
(e) K ′ = 0.
1. Um pescador está sentado no centro de sua canoa 2. Um disco homogêneo de massa M e raio R encontra-
que repousa em um lago de águas tranquilas. A se sobre uma mesa horizontal completamente lisa, sem
sua frente está um cesto com peixes a uma distância estar preso a ela. Em um dado instante duas forças
d. Num dado instante ele muda a posição do cesto estão aplicadas tangencialmente na beirada do disco,
colocando-o detrás dele na posição d ′ . Considerando uma em um ponto diametralmente oposto ao ponto de (a) L1 < L2 , pois o momento de inércia I1 < I2 ;
o centro de massa do sistema canoa-pescador-cesto e aplição da outra. Sabendo que o momento de inércia
(b) L1 = L2 , pois trata-se do mesmo sistema gi-
desprezando-se o atrito da água com a canoa, é correta do disco relativo ao eixo perpendicular a ele que passa
rando com a mesma velocidade angular;
a opção: pelo seu centro de massa é M R2 /2, e denotando por
acm e α os módulos respectivos da aceleração do centro (c) L1 > L2 , pois o momento de inércia I1 > I2 ;
(a) o centro de massa permanece na posição origi- de massa do disco e da aceleração angular de rotação
nal; (d) L1 > L2 , pois o momento de inércia I1 < I2 ;
em torno do eixo no instante considerado, podemos
(b) o centro de massa move-se para frente de d ′ ; (e) L1 < L2 , pois o momento de inércia I1 > I2 .
dizer que:
6. Quatro contas estão presas nos vértices de um arame
(c) o centro de massa move-se para trás de d;
quadrado de lado a, como mostra a figura. Duas con-
(d) após mudar o cesto de posição a canoa passa a tas possuem massa m e as outras duas massa 2m.
se movimentar com velocidade uniforme, pois Desprezando a massa do arame e de acordo com o
a força resultante sobre o sistema é nula; sistema de referência indicado na figura, a posição do
(e) nada se pode afirmar pois não são conhecidas centro de massa do arranjo é:
as massas da canoa, do pescador e do cesto. 4. Um corpo de massa m está suspenso por um fio de
massa desprezı́vel enrolado numa polia de raio R. A
polia consiste em um disco rı́gido do qual foi reti-
(a) acm = F/M e α = 2F/(M R); rado um pedaço, como mostra a (figura). A polia
(b) acm = F/M e α = 6F/(M R); tem massa M e gira sem atrito em torno de um eixo
que passa pelo seu centro. Se a massa desce com ace-
(c) acm = 2F/M e α = 2F/(M R);
leração de módulo a, o momento de inércia da polia
(d) acm = 3F/M e α = 6F/(M R); é:
(e) acm = F/M e α = 0. g 
(a) mR2 +1 ;
a
 g
(b) mR2 1 − ;
a (a) xcm = 2a/5 e ycm = a/2;
g 
(c) (m + M )R2 +1 ; (b) xcm = a/5 e ycm = a/2 ;
a
g
(c) xcm = a/4 e ycm = a/2;

(d) mR2 −1 ;
a (d) xcm = a/2 e ycm = a/2.
g 
(e) (m + M )R2 −1 ; (e) xcm = 2a/3 e ycm = a/2;
a

1 2
7. Uma partı́cula de massa m está girando em torno 9. Um haltere é formado por duas partı́culas de massa Seção 2. Questões discursivas (2×2,5 = 5,0 pontos)
de um eixo (perpendicular a página) com movimento m, ligadas por uma barra rı́gida de massa desprezı́vel
de rotação uniformemente variado com aceleração ~a, e comprimento 2a, e gira com velocidade angular ω1
1. Uma corda de massa desprezı́vel e inextensı́vel está enrolada ao redor de um disco uniforme de raio R e de massa M
como mostra a figura; vista de cima. O raio da tra- em torno de um eixo fixo perpendicular a barra e pas-
e possui a outra extremidade fixada numa barra (conforme mostra a figura). O disco, liberado a partir do repouso,
jetória é R e o vetor aceleração da partı́cula tem a sua sando pelo seu ponto médio. Um segundo haltere é
cai com a corda na posição vertical e mantém o seu eixo de rotação sempre na mesma direção. Dado: o momento
direção formando um ângulo δ com a direção radial. formado por duas partı́culas de massa m/2, ligadas
de inércia de um disco de raioR e de massa M , segundo um eixo que passa pelo seu centro e perpendicular a sua
Num dado instante ela tem velocidade angular ω e por uma barra rı́gida de massa desprezı́vel e compri-
superı́cie, é igual a (1/2)M R2 .
aceleração angular α. Pode-se afirmar que: mento 4a, e gira com velocidade angular ω2 em torno
de um eixo fixo perpendicular a barra e passando pelo a) Determine a aceleração do centro de massa do disco.
(a) tan δ = ω 2 /α; seu ponto médio. Considerando que os halteres têm o b) Qual é o módulo da tensão na corda?
(b) tan δ = α/ω 2 ; mesmo momento angular, podemos afirmar que: c) Qual é a velocidade do centro de massa após o disco ter descido uma altura h?
√ d) Qual é a razão KR /KT entre as energias cinéticas de rotação KR e de translação KT , na mesma situação do
(c) sen δ = α/ ω 4 − α2 ;
√ item c)?
(d) cos δ = ω 2 / ω 4 − α2 ;

(e) sen δ = ω 4 − α2 /α.

(a) ω2 = ω1
8. A figura mostra um carrinho de massa m, sobre um
trilho de ar, que comprime de x uma mola de cons- (b) ω2 = 2ω1
tante elástica k. O carrinho está inicialmente preso (c) ω2 = ω1 /2
ao suporte do trilho por um fio. O fio é cortado, e √
(d) ω2 = 2ω1
a mola expande-se empurrando o carrinho. Ao pas- √
sar pela posição de equilı́brio da mola, O, o carrinho (e) ω2 = ω1 / 2
perde contato com a mola. No trajeto até perder con-
10. Duas partı́culas 1 e 2 de massas m1 = m e m2 = 4m
tato com a mola, o impulso fornecido ao carrinho foi
movem-se em movimento retilı́neo uniforme com ve-
igual a:
locidades ~v1 = ~v /4 e ~v2 = −~v respectivamente. Num
dado instante ocorre uma colisão entre elas totalmente 2. Dois objetos de mesma massa m e dimensões desprezı́veis colidem no espaço sideral, na ausência de forças externas.
inelástica, pode-se afirmar, sobre a velocidade V~cm do A figura mostra um sistema de eixos OXY de um referencial inercial, com seus unitários ı̂ e ̂. Antes da colisão os
centro de massa das partı́culas que: objetos se movem ao longo do eixo OX com velocidades de sentidos opostos, como indicado na figura, e módulos
(a) depois da colisão V~cm = ~0; v1 = 3v e v2 = 2v, nos quais v é uma constante dada. Após a colisão, um dos objetos adquire velocidade
~v1′ = −vı̂ + v̂ (na figura da direita apenas um dos objetos é mostrado).
(a) 2x(km)1/2 ; 1
(b) antes da colisão V~cm = ~v ; a) Determine a velocidade ~v2′ do outro objeto após a colisão, expressando sua resposta em termos da constante v e
 1/2 4
km 3 dos unitários ı̂ e ̂.
(b) x ; (c) depois da colisão V~cm = − ~v ;
2 4 b) Determine a razão K ′ /K entre a energia cinética K ′ do par de objetos depois da colisão e sua energia cinética
x(2km)1/2 ; K antes da colisão; esta colisão é elástica ou inelástica?
(c) (d) antes da colisão V~cm = ~0 e depois da colisão
(d) x(km)1/2 V~cm = −~v
x 3
(e) (km)1/2 . (e) depois da colisão V~cm = ~v ;
2 4

3 4
Universidade Federal do Rio de Janeiro
Centro de Ciências Matemáticas e da Natureza
Instituto de Fı́sica
Segunda Prova de Fı́sica IA - 15/07/2013
Respostas para provas hı́bridas

Gabarito das Questões objetivas (valor=5.0 pontos)

Versão A Versão B
Questão (a) (b) (c) (d) (e) Questão (a) (b) (c) (d) (e)
1 1
2 2
3 3
4 4
5 5
6 6
7 7
8 8
9 9
10 10

Versão C Versão D
Questão (a) (b) (c) (d) (e) Questão (a) (b) (c) (d) (e)
1 1
2 2
3 3
4 4
5 5
6 6
7 7
8 8
9 9
10 10

1
Questão discursiva 1 (valor=2.5 pontos)
a) valor=1,2 pontos
O ioiô desloca-se com dois modos de movimento dados pela dinâmica de rotação e de translação.

F~ ext = M~aCM
P
translação :

~τ ext = I α
P
rotação : ~

Na direção e sentido do movimento de translação e considerando que o ioiô gira em torno


do seu centro de massa no sentido anti-horário sem deslizar sobre o corda e que os torques são
calculados em relação
 centro de massa, temos:

 Mg − T = MaCM 
 Mg − T = MaCM (i)



T R = Iα →
T = IaCM /R2 (ii)

 


aCM = αR

A solução do sistema de equações da direita permite obter a aceleração do centro de massa,

Mg 2
aCM = como I = (1/2)MR2 ∴ aCM = g
(I/R2 + M) 3

b) valor=0.3 pontos
Para obter o valor da tensão da corda podemos usar a equação (ii) do sistema de equações,
substituindo o valor da aCM obtido do item anterior. Logo,
IaCM 1
T = → T = Mg
R2 3

c) valor=0.5 ponto
A velocidade do centro de massa do ioiô após ele cair de uma altura h, pode ser obtida aplicando
o princı́pio da conservação de energia mecânica. Assim temos:
1 1
Ei = Mgh e Ef = KR + KT = Iω 2 + MvCM
2
2 2
Para Ei = Ef e ω = vCM /R,
r
2 2Mgh 2 4
vCM = com I = (1/2)MR → vCM = gh
(I/R2 + M) 3

d) valor=0,5 ponto
A energia cinética de rotação é KR = (1/2)Iω 2 e de translação KT = (1/2)MvCM
2
. Portanto,
KR (1/2)Iω 2 (1/2)MR2 vCM
2
/R2 KR 1
= 2
= 2
∴ =
KT (1/2)MvCM MvCM KT 2
Este resultado vale para qualquer instante t e também quando o ioiô cai de uma altura h.

2
Questão discursiva 2 (valor=2.5 pontos)
a) valor=1,0 ponto
Não há força externa sobre o sistema constituı́do pelos dois objetos. Portanto, seu momento
linear se conserva, isto é, m(−3vı̂) + m 2vı̂ = m(−vı̂ + v̂) + m~v20, donde

~v20 = −v̂.

b) valor=1,5 pontos
Calculando a energia cinética total antes da colisão temos:

K = (1/2)m(−3v)2 + (1/2)m(2v)2 = (13/2)mv 2

Depois da colisão,
K 0 = (1/2)m((−v)2 + v 2) + (1/2)m(−v)2 = (3/2)mv 2 , donde

K 0 /K = 3/13.

Concluimos assim que o choque é inelástico, pois K 0 6= K.

3
4. Uma partı́cula move-se com velocidade constante 6. Considere uma esfera que rola sem deslizar sobre uma
UNIVERSIDADE FEDERAL DO RIO DE JANEIRO
~v ao longo de uma reta paralela ao eixo OX que calha simétrica que tem dois trechos retilı́neos, I e II,
INSTITUTO DE FÍSICA corta o OY em y = d (d > 0). Sejam L ~ 1, L
~2 e como mostrado na figura. Ela oscila descendo e subindo
~ 3 os momentos angulares da partı́cula, relativos
L nos dois ramos da calha. No trecho I a força de atrito é
FÍSICA I – 2013/2
à origem O, nos pontos 1, 2 e 3, como indicados na f~I e no trecho II, é f~II . Considere que as únicas forças
SEGUNDA PROVA – 18/11/2013 figura. Os módulos dos momentos angulares são tais que que atuam sobre a esfera sejam a força normal e a força
VERSÃO: A de atrito exercidas pela calha, e o seu peso.

Nas questões em que for necessário, considere que g é o módulo da aceleração da gravidade.
(a) ~ 1 | < |L
|L ~ 2 | < |L
~ 3 |;
Seção 1. Múltipla escolha (10×0,5 = 5,0 pontos) (b) ~ ~ ~
|L1 | > |L2 | > |L3 |;;
(c) ~ 1 | = |L
|L ~ 2 | = |L
~ 3 |;
Dentre os diagramas seguintes
1. Uma chapa homogênea quadrada de lado 2a tem um 2. Uma barra fina e homogênea de massa M e comprimento ~ 1 | < |L
~ 2 | = |L
~ 3 |;
(d) |L
canto quadrado de lado a retirado. A chapa restante ℓ é liberada na posição horizontal a partir do repouso e
está disposta no plano OXY como indicado na figura. gira em torno de um eixo horizontal fixo, perpendicular (e) ~ 1 | = 0 e |L
|L ~ 2 | < |L
~ 3 |.
Em relação à origem O, o vetor posição ~rcm do centro a ela e que passa por uma de suas extremidades. Sa-
de massa bendo que não há atrito entre a barra e o eixo e que o
momento de inércia da barra relativo ao eixo é igual a
(1/3)M L2 , podemos afirmar que ao passar pela posição o que melhor representa o sentido e a direção das forças
vertical a velocidade do centro de massa da barra é 5. Um sistema é constituı́do por duas partı́culas de mesma de atrito f~I e f~II nos trechos I e II é
massa e uma barra fina homogênea sobre uma mesa ho- (a) 5;
p rizontal sem atrito. As partı́culas estão em movimentos (b) 3;
(a) 3gL/2; retilı́neos uniformes com trajetórias paralelas e velocida-
√ des respectivas ~v e −~v . Elas colidem, simultaneamente, (c) 1;
(b) 3gL;
p cada uma com uma extremidade da barra, que está em (d) 4;
(c) πgL/2;
p repouso e perpendicular às trajetórias, como indicado na (e) 2.
(d) 3gL/4; figura. Após a colisão as partı́culas permanecem presas
(a) (7/6)a(ı̂ + ̂). à barra e o sistema passa a girar com velocidade angular
(e)
p
πgL/4. 7. Um projétil é lançado de uma plataforma do ponto O
(b) (5/6)a(ı̂ + ̂); constante em torno do centro O da barra. e num dado instante após ser lançado ele se fragmenta
(c) (1/3)a(ı̂ + ̂); em dois pedaços de massas m1 = 3m e m2 = m, que
caem simultaneamente nas posições horizontais x = d e
(d) (1/2)a(ı̂ + ̂); x = (1, 5)d respectivamente. Se o projétil não se frag-
(e) (2/3)a(ı̂ + ̂); mentasse o seu alcance A seria,
3. Uma granada encontra-se em repouso sobre uma mesa
horizontal lisa (sem atrito) e explode em três fragmen-
tos. Os fragmentos adquirem momentos lineares p~1 , p~2 e
p~3 de módulos diferentes de zero, cujas direções formam
ângulos diferentes entre si. O diagrama correto que
Sobre o momento linear do sistema, o seu momento an-
representa a relação entre os momentos lineares destes
gular relativo ao ponto O e a sua energia cinética pode-
fragmentos é
mos afirmar que
(a) conserva-se o momento linear mas não o angular;
(a) 1; (a) (6/5)d;
(b) conservam-se os momentos linear e angular;
(b) 2; (b) (5/3)d;
(c) conserva-se o momento angular mas não o linear;
(c) 3; (c) (11/8)d;
(d) não se conservam os momentos linear e angular;
(d) 4; (d) (10/8)d;
(e) não se conservam o momento linear e a energia
(e) 5. cinética. (e) (9/8)d.

1 2
8. Um chapa quadrada fina e homogênea de lado a 9. Dois halteres (1) e (2) mostrados na figura giram em Seção 2. Questões discursivas (2×2,5 = 5,0 pontos)
encontra-se sobre uma superfı́cie horizontal sem atrito torno de seus respectivos centros de massa em torno de
presa por um pino O em um dos seus vértices. um eixo fixo e perpendicular ao seu comprimento. O Não serão consideradas respostas sem justificativa; expresse-as somente em função dos dados fornecidos.
Num dado instante aplicam-se quatro forças hori- haltere (1) gira com velocidade angular constante ω e o
zontais e de mesma intensidade F perpendiculares haltere (2) com velocidade angular constante ω/2. No 1. Dois blocos A e B de massas mA = 2m e mB = m, respectivamente, encontram-se sobre uma superfı́cie horizontal lisa. O
aos lados do quadrado como mostra a figura. O haltere (1) as massas de sua extremidades são iguais a bloco A desloca-se com velocidade horizontal ~v constante, enquanto o bloco B encontra-se em repouso. Após a colisão
módulo do torque resultante τO em relação a O é m e no haltere (2) 2m. Os módulos de seus momentos os blocos separam-se e observa-se que o bloco de massa mA recua movendo-se com velocidade de módulo 2v na mesma
angulares L1 e L2 em relação aos respectivos centros de direção do seu deslocamento inicial, como mostra a figura.
massa e as suas energias cinéticas K1 e K2 cinéticas são
a) Determine vB , o módulo da velocidade do bloco B após a colisão.
relacionados respectivamente como
(a) (5/2)aF ; b) Calcule o módulo da velocidade do centro de massa dos blocos antes e depois da colisão.
c) Determine a variação da energia cinética do sistema no processo de colisão.
(b) (3/2)aF ;
(c) (7/2)aF ;
(d) (3/4)aF .
(e) (2/3)aF .

(a) L1 = L2 e K 1 > K 2
(b) L1 = L2 e K 1 = K 2
(c) L1 = L2 e K 1 < K 2
(d) L1 < L2 e K 1 < K 2
2. Um disco de raio R e massa M encontra-se fixado a uma parede vertical em um pino que passa pelos seu centro. Um cabo
(e) L1 > L2 e K 1 > K 2 de massa desprezı́vel passando pela sua periferia tem na sua extremidade um bloco de massa m sobre uma superfı́cie
10. Um disco homogêneo de raio R e massa M rola sem horizontal. O cabo é puxado por uma força constante F~ , erguendo o bloco de uma altura h, como mostra a figura.
deslizar sobre uma superfı́cie horizontal com velocidade Desprezando-se o efeito de atrito entre o pino e o disco e considerando que o bloco e o disco estavam em repouso no
angular ω constante. O momento de inércia deste disco momento em que a força F~ começou a agir calcule:
em relação ao eixo que passa pelo seu centro de massa a) a aceleração com que o bloco é erguido;
perpendicularmente ao disco é igual a (1/2)M R2 . A b) o módulo da tração que age sobre o trecho do cabo entre o disco e o bloco;
energia cinética desse disco é c) a energia cinética adquirida pelo disco imediatamente após o bloco ser erguido da altura h.
(a) (1/2)M R2 ω 2 ; Dado: o momento de inércia do disco para um eixo que passa perpendicularmente ao disco e pelo seu centro é (1/2)M R2 .
(b) (3/2)M R2 ω 2 ;
(c) (2/3)M R2 ω 2
(d) (3/4)M R2 ω 2 .
(e) (5/6)M R2 ω 2 ;

3 4
Universidade Federal do Rio de Janeiro
Centro de Ciências Matemáticas e da Natureza
Instituto de Fı́sica
Segunda Prova de Fı́sica IA - 18/11/2013
Respostas para provas hı́bridas

Gabarito das Questões objetivas (valor=5,0 pontos)

Versão A Versão B
Questão (a) (b) (c) (d) (e) Questão (a) (b) (c) (d) (e)
1 1
2 2
3 3
4 4
5 5
6 6
7 7
8 8
9 9
10 10

Versão C Versão D
Questão (a) (b) (c) (d) (e) Questão (a) (b) (c) (d) (e)
1 1
2 2
3 3
4 4
5 5
6 6
7 7
8 8
9 9
10 10

1
Questão discursiva 1 (valor=2,5 pontos)
a) valor=1,0 ponto
O momento linear total é consevado pois a resultante das forças externas é nula,

P~i = P~f

P~i = 2m~v e P~f = 2m(−2~v ) + m~vB


∴ ~vB = 6~v → vB = 6v

b) valor=1,0 ponto
Calculando a velocidade do centro de massa do sistema antes (a) e depois da colisão (d),
2m 2
~vcm,a = ~v → |~vcm,a| = v (1)
2m + m 3
2m(−2~v ) + m(6~v) 2
~vcm,d = → |~vcm,d | = v (2)
2m + m 3
Como o momento linear total é conservado, a velocidade do centro de massa conserva-se inde-
pendentemente do tipo de colisão.

c) valor=0,5 ponto
Calculando as energias cinéticas antes Ka e depois Kd da colisão temos,

1 1
Ka = mA v 2 = 2mv 2 = mv 2 (3)
2 2
1 1
Kd = KA,d + KB,d = 2m(2v) + m6v 2 = 22mv 2
2
(4)
2 2

∴ ∆K = 22mv 2 − mv 2 → ∆K = 21mv 2 (5)

∆K > 0 corresponde a uma colisão inelástica com liberação de energia no processo de


colisão!!

2
Questão discursiva 2 (valor=2,5 pontos)
a) valor=1,6 pontos
Na figura estão representadas as forças que agem no bloco e
no disco, onde P~b é o peso do bloco, P~D o peso do disco, T~
a tração do cabo sobre o bloco e T~ 0 a reação agindo sobre o
disco, F~S a força que o pino exerce sobre o disco e a força F~ .
A dinâmica para o disco é dada pela relação i ~τiext = I α
P
~ e
~
P
para o bloco i Fi = m~a além da condição de vı́nculo a = αR,
admitindo que o cabo não desliza sobre o disco, onde a é o
módulo da aceleração do bloco e α o módulo da aceleração
angular do disco.

Assim temos, considerando o sentido de rotação positivo como anti-horário e que |T~ | = |T~ 0| = T :

 RT − RF = −Iα 
F − T = Ia/R2
T − Pb = ma ⇒
T − Pb = ma (i)
a = αR

A resolução do sistema de equações da direita permite obter a aceleração a, onde I = (1/2)MR2 ,


F − mg
a=
(m + M/2)
b) valor=0,4 ponto
A tração do fio pode ser obtida de (i), após substituirmos o valor de a encontrado no item
anterior,
m(F + Mg/2)
T − Pb = ma → T =
(m + M/2)
c) valor=0,5 ponto
Para a variação da energia cinética do disco temos ∆K = Wtotal = Kf , pois Ki = 0.
O trabalho total é dado pelo trabalho das forças F~ e T~ 0 sobre o disco fazendo-o girar. Como
o cabo é puxado de h, e lembrando que |T~ 0| = T ,

0
h m(F + Mg/2) i
Kf = (F − T )h = F − h
(m + M/2)
1 M(F − mg)
∴ Kf = h
2 (m + M/2)

3
4. Um sistema é constituı́do de três partı́culas de mesma 7. Um menino de massa encontra-se parado em uma das
UNIVERSIDADE FEDERAL DO RIO DE JANEIRO
massa localizadas nos semi-eixos positivos OX, OY e extremidades de um barco que está em um lago tran-
INSTITUTO DE FÍSICA OZ a uma mesma distância a da origem, como indica a quilo. Inicialmente o conjunto (barco-menino) move-se
FÍSICA I – 2014/1 figura. O vetor posição ~rcm do centro de massa do sis- com velocidade constante, em relação a um referencial
tema é inercial fixo colocado na margem do lago. Num dado
SEGUNDA PROVA – 23/05/2014 instante o menino move-se para a outra extremidade do
VERSÃO: A a barco e ao chegar neste lugar ele para. Para o sistema
(a) (ı̂ + ̂ + k̂);
2 barco-menino, desprezando-se o atrito entre o barco e a
a água, pode-se afirmar que
(b) (ı̂ + ̂ + k̂);
Nas questões em que for necessário, considere que g é o módulo da aceleração da gravidade. 3
(a) A sua velocidade antes do menino se movimentar
(c) a(ı̂ + ̂ + k̂); é diferente de depois do menino parar.
a
Seção 1. Múltipla escolha (10×0,5 = 5,0 pontos) (d) (ı̂ + ̂ + k̂); (b) A sua velocidade é a mesma de antes do inı́cio do
6
2a movimento do menino e depois do menino parar.
(e) (ı̂ + ̂ + k̂).
1. Um cilindro sólido rola sem deslizar, com velocidade an- 2. Dois discos de massas m e 3m estão ligados por uma 3 (c) A sua velocidade pode ser nula depois do menino
gular constante, sobre uma superfı́cie horizontal. Dentre mola ideal comprimida sobre uma mesa horizontal lisa. parar.
os diagramas abaixo, I), II), III) e IV), aquele que repre- Em um dado instante a força da mola sobre o primeiro (d) Não se pode determinar a sua velocidade, pois
senta corretamente os vetores velocidade (indicados por disco é F~el e uma força F~ paralela ao plano da mesa está não se conhece a velocidade do menino sobre o
setas) dos pontos A (topo), C (base) e B (centro), pon- aplicada sobre o segundo disco, como mostra a figura 5. Um carro tem uma roda de massa M e raio R com mo- barco.
tos pertencentes a perferia do cilindro (seção reta), em (mesa vista de cima). No instante considerado a ace- mento de inércia Icm = (3/4)M R2 relativo ao seu eixo (e) Não se pode determinar a sua velocidade, a me-
relação a um referencial fixo na Terra é leração do centro de massa do sistema constituı́do pelos de simetria que passa pelo seu centro de massa. O carro nos que sejam conhecidas as massas do menino e
discos e pela mola é arranca em movimento retilı́neo com a roda patinando do barco.
na pista de modo que a velocidade ~vcm de seu centro de
massa esteja relacionada com a sua velocidade angular
F~ de rotação ω por meio de vcm = ωR/2. Nesse caso, a
(a) ; energia cinética da roda é
4m
F~ + F~el (a) (1/4)M R2 ω 2 ;
(b) ;
4m (b) (5/4)M R2 ω 2 ;
F~ + 2F~el (c) (3/4)M R2 ω 2
(c) ;
4m (d) M R2 ω 2 .
F~ − 2F~el (e) (1/2)M R2 ω 2 ; 8. Um corpo rı́gido gira em torno de um eixo fixo com velo-
(d) ;
4m cidade angular constante ω quando, a partir do instante
F~ − F~el t = 0, passa a ter uma aceleração angular constante de
(e) . módulo α e sentido oposto ao da velocidade angular.
4m
Denotando por t1 o tempo gasto pelo corpo para sua
6. Um disco fino e homogêneo de raio a encontra-se em re-
(a) I velocidade angular atingir o valor nulo e por θ1 o seu
pouso sobre uma superfı́cie horizontal sem atrito. Num
3. Uma esfera oca e rı́gida de massa M e raio R tem mo- deslocamento angular do instante t = 0 até o instante
(b) II dado instante aplicam-se na periferia do disco três forças
mento de inércia Icm = (2/3)M R2 relativo a um eixo t1 , podemos afirmar que
(c) III tangenciais e uma radial, todas de mesma intensidade F
que passa pelo seu centro de massa. Relativamente a 1 ω2
com direções, sentidos e pontos de aplicação conforme (a) t1 = e θ1 = ;
(d) IV um eixo que tangencia a periferia da esfera e é paralelo
indicados na figura. Nesse instante, o módulo do tor- ω α
ao eixo que passa pelo centro de massa, o momento de 2
(e) nenhum deles que resultante τO em relação ao centro O do disco é ω ω
inércia da esfera é (b) t1 = e θ1 = ;
(a) aF/3; α α
(a) Icm α ω2
(b) aF/2; (c) t1 = e θ1 = ;
(b) (1/3)M R2 ω α
(c) aF ; α ω2
(c) (5/3)M R2 (d) t1 = e θ1 = ;
(d) 2aF . ω 2α
(d) (7/10)M R2 2
(e) 3aF . ω ω
(e) 2Icm (e) t1 = e θ1 = .
α 2α

1 2
9. Duas partı́culas de mesma massa caem verticalmente e 10. Duas partı́culas inicialmente separadas sofrem uma co- Seção 2. Questões discursivas (2×2,5 = 5,0 pontos)
colidem com uma mesa horizontal lisa com a mesma ve- lisão totalmente inelástica na ausência de forças exter-
locidade. Uma das partı́culas colide elásticamente com nas. Pode-se afirmar, sobre o sistema constituı́do pelas Não serão consideradas respostas sem justificativa; expresse-as somente em função dos dados fornecidos.
a mesa sem sair da vertical e a outra colide de forma to- duas partı́culas, que
talmente inelástica com a mesa. Denotando por ∆P~el a 1. Três partı́culas A, B e C, deslizando sobre uma mesa horizontal sem atrito, se aproximam da origem de um sistema de
variação do momento linear da primeira partı́cula na co- (a) a energia cinética do centro de massa do sistema eixos OXY no plano da mesa. Todas as três partı́culas atingem a origem simultaneamente e permanecem unidas após
lisão elástica, por ∆P~in a variação do momento linear da é a mesma antes e depois da colisão; a colisão, sem perder contato com a mesa, deslocando-se ao longo do eixo OX no sentido positivo com velocidade de
segunda partı́cula em sua colisão totalmente inelástica, (b) o módulo da velocidade do centro de massa do módulo V desconhecida. As partı́culas A, B e C têm massas respectivas mA = m, mB = 3m e mc = 4m, onde m é uma
podemos afirmar que sistema é menor antes do que depois da colisão; constante conhecida. Antes da colisão as partı́culas têm velocidades de módulos respecticos vA = 3v, vB = v e vC = 2v,
(c) a energia cinética total do sistema é a mesma an- onde v é uma constante conhecida. A partı́cula C tem velocidade na direção e sentido do eixo OX e a partı́cula B tem
(a) ∆P~el = ∆P~in velocidade com direção e sentido indicados na figura por meio do ângulo conhecido β (0 < β < π/2). A velocidade da
tes e depois da colisão;
(b) |∆P~el | < |∆P~in | partı́cula A tem o sentido indicado na figura mas sua direção é desconhecida e indicada pelo ângulo α (0 < α < π/2) a
(d) o módulo da velocidade do centro de massa do ser determinado. Calcule
(c) ∆P~el =
6 ~0 e ∆P~in = ~0 sistema é maior antes do que depois da colisão;
a) o ângulo α;
(d) ∆P~el = 2∆P~in (e) a energia cinética total do sistema é menor antes b) O módulo V da velocidade após a colisão em função de v;
(e) ∆P~el = −2∆P~in do que depois da colisão. c) a variação da energia cinética do sistema formado pelas três partı́culas no processo de colisão.

2. Um fio inextensı́vel de massa desprezı́vel é enrolado diversas


vezes em torno da periferia de um cilindro maciço de raio
R2 , segue horizontalmente até a periferia de outro cilindro
de raio R1 , pela qual passa e segue verticalmente, até sua
extremidade, na qual está supenso um bloco de massa m.
Os cilı́ndros, cada um também de massa m, são homogêneos
e podem girar sem atrito, cada um em torno de seu eixo fixo.
O bloco desce verticalmente puxando o fio que fica tenso e
faz os cilindros girarem, sem deslizar sobre suas periferias.
(O momento de inércia de um cilindro homogêneo de massa
M e raio R relativo ao seu eixo é M R2 /2.)
a) Faça um diagrama das forças externas que agem sobre o
cilindro de raio R1 .
b) Calcule o módulo da aceleração com que desce o bloco.
c) Obtenha o módulo da aceleração angular de cada cilindro.
d) Calcule o módulo da tensão no trecho vertical do fio e o
módulo da tensão no trecho horizontal do fio.

3 4
Universidade Federal do Rio de Janeiro
Centro de Ciências Matemáticas e da Natureza
Instituto de Fı́sica
Segunda Prova de Fı́sica IA - 23/05/2014
Respostas para provas hı́bridas

Gabarito das Questões objetivas (valor=5,0 pontos)

Versão A Versão B
Questão (a) (b) (c) (d) (e) Questão (a) (b) (c) (d) (e)
1 1
2 2
3 3
4 4
5 5
6 6
7 7
8 8
9 9
10 10

Versão C Versão D
Questão (a) (b) (c) (d) (e) Questão (a) (b) (c) (d) (e)
1 1
2 2
3 3
4 4
5 5
6 6
7 7
8 8
9 9
10 10

1
Questão discursiva 1 (valor=2,5 pontos)
a) valor=0,8 pontos
Pela conservação do momento linear do sistema, P~i = P~f , segundo os eixos coordenados OX
e OY ,
pa (cos α ı̂ + sin α ̂) − pb (cos β ı̂ + sin β ̂) + pc ı̂ = MV ı̂
ou seja, 
 3mv cos α − 3mv cos β + 4m2v = 8mV (i)

3mvsenα − 3mvsenβ = 0 (ii)


Levando-se em conta os dados fornecidos e a relação (ii) obtemos

m 3v sin α = 3m v sin β =⇒ sin α = sin β ∴ α = β.

b) valor=1,2 pontos
Se agora levarmos em conta o último resultado onde α = β, e a relação obtida na equação
(i) do sistema de equações, obtemos
( 
m(
( 3v(cos
(( α −
3m
vcos β + 4m 2v = 8mV ∴ V = v.

c) valor=0,5 ponto
1 1 1 1
Nesse caso que Ki = Ka + Kb + Kc = mava2 + mbvb2 + mc vc2 e Kf = MV 2.
2 2 2 2
1
A partir dos dados fornecidos, encontramos Ki = m 9v 2 + 3m v 2 + 4m 4v 2 = 14mv 2, en-
2
1
quanto Kf = 8m v 2 = 4mv 2. Com isso,
2
∆Kif = Kf − Ki = −10mv 2.

2
Questão discursiva 2 (valor=2,5 pontos)
a) valor=0,4 pontos
As forças que agem sobre o cilindro de raio
R1 estão indicadas na figura. As setas in-
dicam o sentido e direção e acima ou ao lado
delas os seus módulos, como no livro texto.

b) valor=1,7 pontos
Considerando o sentido de cima para baixo para o movimento do bloco e aplicando a segunda
lei de Newton,

• o movimento do bloco será dado por


m~a = F~R = T~b + P~b =⇒ Mg − T1 = ma (i)
• para o movimento de rotação do primeiro cilindro, considerando como positivo o sentido
anti-horário de rotação e calculando os torques em relação ao seu centro encontramos a relação
I1 α
~ 1 = ~τR = ~τF1 + ~τP1 + ~τT1 + ~τT2 ou seja,
I1α1 = R1 (T1 − T2) (ii),
• analogamente para o movimento de rotação do segundo cilindro encontramos,
I2 α
~ 2 = ~τR = ~τF2 + ~τP2 + ~τT2 o que resulta em ter
I2α2 = R2 T2 (iii)
Neste ponto devemos observar que a presença do fio acarreta a condição de vı́nculo

a = α1 R1 = α2 R2
Ao utilizarmos as relações acima, (i), (ii) e (iii) e a condição de vı́nculo obtemos o sistema
de equações,
 

 mg − T1 = ma 
 mg − T1 = ma (iv)

 

 
R1 T1 − R1 T2 = I1α1 → T1 − T2 = a1I1 /R21 (v)

 


 

R2 T2 = I2α2 T2 = a2I2/R22 (vi)
 
g
A resolução do sistema de equações da direita nos dá a aceleração a= .
1+ I1 /(mR21 ) + I2/(mR22 )
Como I1 = 21 mR21 e I2 = 12 mR22 então,
g g
a= 1 1 =⇒ a= .
1+ 2
+ 2
2

3
c) valor=0,2 pontos
Utilizando o resultado obtido para a aceleração do bloco nas equações de vı́nculo, obtemos
g g
α1 = e α2 = .
2R1 2R2
d) valor=0,2 pontos
Levando em conta o resultado para a aceleração do bloco nas relações obtidas anteriormente,
das linhas (v) e (vi) do sistema de equações encontramos
   
I1 I2 g 1 1 g 1
T1 = 2
+ 2 =⇒ T1 = m+ m =⇒ T1 = mg .
R1 R2 2 2 2 2 2

Da mesma forma, encontramos


   
I2 g 1 g 1
T2 = =⇒ T2 = m =⇒ T2 = mg .
R22 2 2 2 4

4
Instituto de Fı́sica - UFRJ

Prova Final de Fı́sica IA - 2011/1

Obs: em todas as questões em que for necessário, considere que g é o módulo da aceleração da gravidade

Questão 1) Um atleta está em uma competição de salto em altura, onde há uma vara colocada
em uma trave que é o seu obstáculo a ser transposto. A vara está distante de d da posição
a partir da qual ele efetua o salto com velocidade ~v0 e ângulo θ0 desconhecidos. Ao pular ele
passa imediatamente acima da vara. A trajetória do salto, indicada na figura, tem como altura
máxima a altura h. Considere o atleta como uma partı́cula e que o sistema de coordenadas
ZOX está localizado no ponto O, de onde ele salta. Determine:

a) o ângulo θ0 , em função de d e h;
b) o módulo da sua velocidade inicial v0 .

Questão 2) Um carrinho de montanha russa de massa m encontra-se com seus freios ativados
em uma rampa, inclinada de um ângulo θ com a horizontal, na iminência de deslizar.

a) Os freios do carrinho são soltos, e ele começa a deslizar para baixo sem atrito. Determine
o módulo de sua aceleração nesse instante.
b) O carrinho desliza por uma distância d na superfı́cie da rampa, e atinge em seguida o inı́cio
de um loop vertical de raio R. Supondo que o carrinho consiga completar o loop, determine
o módulo de sua velocidade v no ponto mais alto do cı́rculo descrito por ele.
c) Qual a altura mı́nima hmin que a rampa deve ter para que o carrinho atinja esse ponto
mais alto sem perder o contato com o trilho?
Questão 3) Um braço articulado, composto por dois semibraços de comprimento d e massa m
cada, está firmememte preso numa das extremidades a um eixo vertical(rı́gido) que gira sem
atrito com velocidade ω constante quando os semibraços fazem um ângulo reto entre si. Veja a
figura abaixo i). Por meio de um controle remoto, enquanto o sistema gira, o ângulo entre os
semibraços é alterado para a posição em que eles ficam alinhados como mostra a figura ii).
a) Calcule os momentos de inércia do sistema, formado pelos semibraços, em relação ao eixo
de rotação nas situações i) e ii); denomine-os I e I 0 respectivamente. (Dado: O momento
de inércia de uma barra de comprimento L e massa M, em torno de um eixo perpendicular
a ela e passando por uma de suas extremidades é I = (1/3)ML2 ).
b) Após o alinhamento dos semibraços, figura ii), a velocidade angular altera o seu valor para
ω 0 constante, calcule esta nova velocidade. Justifique a sua dedução.

Questão 4) Uma polia presa ao teto está ligada a um bloco por um fio ideal enrolado em torno
do seu ressalto. O bloco que está pendurado verticalmente tem massa m, a massa da polia
M = 2m, raio R = 2r e o raio do ressalto r. A polia inicia o seu movimento a partir do
repouso e pode girar em torno do seu eixo livremente e sem que o fio deslize. O momento de
inércia da polia em relação a um eixo que passa pelo seu centro e perpendicularmente a ela é
I = (1/2)MR2 . Após o inı́cio de seu movimento:
a) isole o bloco e a polia e por meio de um diagrama
represente todas as forças que atuam em cada um
deles;
b) de acordo com as leis da dinâmica escreva as
equações para a translação do bloco e a rotação
da polia e determine o módulo da aceleração com
que o bloco se movimenta;
c) após a polia dar uma volta completa, qual a ener-
gia cinética que o sistema bloco-polia adquiriu?
d) calcule o trabalho exercido pela força externa re-
sultante sobre o sistema bloco-polia, após ela dar
uma volta completa, e compare-o com o resul-
tado obtido no item anterior. O que conclui-se
desta comparação? Justifique a sua resposta.
Questão 1

a) valor = (2,0 pontos)


Durante a trejetória do atleta no ar este sofre a ação apenas de uma única força, a força
peso, que está orientada no sentido negativo do eixo Z e produz uma aceleração constante
g. Portanto, o movimento do atleta pode ser decomposto nas direções Z e X, onde os
movimentos são uniformemente acelerado e uniforme respectivamente. Observa-se que o
atleta passa necessariamente pelo ponto de coordenda (d, h) que representa o ponto onde
a componente Z da velocidade se anula. As equações horárias do movimento são:


 Eixo X : x = v0xt = v0 cos θ0 t


 Eixo Z : vz2 = vz0
2
− 2gz = v02 sen2θ0 − 2gz
vz = vz0 − gt = v0 senθ0 − gt

No ponto mais alto da trajetória, vz = 0, t = t∗, x = d e z = h, substituindo estes resultados


nas equações anteriores:

d = v0 cos θ0 .t∗ i)
v02sen2 θ0 = 2g ii)
v0senθ0 = gt∗ iii)

v02
Substituindo iii) em i): d = senθ0.cosθ0, eliminando v0 , com a utilização da equação
g
ii), temos:
2gh 1 2h 2h
d= senθ 0 .cosθ 0 = ⇒ tanθ0 =
sen2θ0 g tanθ0 d

b) valor = (0,5 pontos)


4h 4h2
Do resultado anterior, tan(θ0)2 = , logo, sen2
θ 0 = .cos2 θ0. Substituindo este resul-
d2 d2
4h2 d
tado na eq. ii), obtemos: v02.( 2 ).cos2 θ0 = 2gh, da eq. i), cosθ0 = , portanto:
d v0 t∗
 2  d2
2 4h 2h 2h
v0 . 2 . 2 ∗ 2 = ⇒ (t∗)2 = iv)
d v0 (t ) g g
Das eqs i) e iii), podemos isolar senθ0 e cosθ0 . Usando a identidade trigonométrica
sen2θ0 + cos2 θ0 = 1, obtemos:
g 2 (t∗ )2 d2 2 2 ∗ 2 d2
+ 2 ∗ 2 = 1 → v0 = g (t ) + ∗ 2
v02 v0 (t ) (t )
Finalmente da eq. iv),
r
2h gd2 gd2 g(d2 + 4h2 )
v02 = g 2 . + = 2gh + ⇒ v0 =
g 2h 2h 2h

1
Questão 1(Outra solução)

a) valor = (2,0 pontos)


O movimento do atleta durante o salto tem como equações para o seu alcance A e
altura máxima h,
2senθ0.cosθ0
A = v02.
g
sen2θ0
h = v02 . ii)
2g
senθ0.cosθ0
Como A/2 = d = v02. , dividindo a equação ii) por d,
g
Obtemos,

h v 2.sen2θ0 /2g tanθ0 2h


= 2 0 = ⇒ tanθ0 =
d v0 .senθ0.cosθ0 /g 2 d

b) valor = (0,5 pontos)


Sabemos que:
1 v0 z 2 2gh
h= → v02 =
2 g sen2 θ0
d2
v02 = 2ghcsc2 θ0 = 2gh(1 + cot2 θ0 ) = 2gh(1 + )
4h2
Portanto, r
g(d2 + 4h2 )
v0 =
2h
ou geometricamente, vide figura! A direção do vetor velocidade ~v0 é dada pelo ângulo de
lançamento!!

2h 2 4h2
senθ0 = √ → sen θ0 = 2
d + 4h2
2 d + 4h2 )
2gh
Mas, v02sen2 θ0 = 2gh → v02 =
sen2 θ0
Logo, r
(d2 + 4h2 ) g(d2 + 4h2 )
v02 = 2gh. ⇒ v0 =
4h2 2h

2
Questão 2

a) valor = (0,7 pontos)


Considerando como eixo x a direção paralela à superfı́cie da rampa e como eixo y a
direção perpendicular à mesma, a segunda lei de Newton para o eixo x pode ser escrita
como:
mg sin(θ) = ma
Logo,
a = g sin(θ)

b) valor = (1,0 pontos)


Temos, K0 +U0 = Ksolo +Usolo . Como o carrinho parte do repouso, podemos reescrever
a equação acima como mgh = (1/2)mv 2 + mg(2R), onde h = d sin(θ).
Logo, gd sin(θ) = (1/2)v 2 + g(2R).
Por fim, p
v = 2g(d sin(θ) − 2R)

c)valor= (0,8 pontos)


Quando o carrinho encontra-se na iminência de cair, a força realizada por ele no trilho
é nula; consequentemente, a reação do trilho - a Normal - será também nula. Com isso,
podemos escrever N ~ + P~ = m~a simplesmente como:
2
mvmin
P = marad = mg =
R
p
vmin = gR
Aplicando a conservação de energia para o sistema, encontramos:
1 2
mghmin = mvmin + mg(2R)
2
gR
hmin = + 2R
2g
5R
hmin =
2

3
Questão 3

a) valor = (1,0 pontos)


Na situação i), o momento de inércia do semi-braço perpendicular ao eixo de rotação é
1
igual a md2 . Para o semi-braço paralelo, como todos os seus elementos de massa estão
3
a uma mesma distância d do eixo de rotação, então, o seu momento de inércia é md2 .
Assim,

1 4
I = md2 + md2 ⇒ I = md2
3 3

Na situação ii), temos, efetivamente, uma barra de comprimento 2d e massa 2m perpen-


dicular ao eixo de rotação e com uma extremidade presa a ele. Logo,

1 8
I 0 = (2m)(2d)2 ⇒ I 0 = md2
3 3

c) valor=(1,5 pontos)
Justificativa(0,5 pontos):
Não há torques na direção do eixo de rotação e, portanto, a componente do momento
angular naquela direção é constante.

Resultado(1,0 pontos)
Usando a a conservação do momento angular na direção do eixo de rotação, temos,

Iω = I 0ω 0

Logo,
I 4/3 ω
ω0 = ω = ω ⇒
I0 8/3 2

4
Questão 4

a) valor = (0,5 pontos)


Veja o diagrama ao lado.


sobre a polia 
 sobre o bloco


 o peso M~g
o peso m~g , e
a tração T~ exercida pela corda no ponto P, e
a tração T~ 0 exercida pela corda
 
a tração T~O sobre o eixo no suporte fixo no teto

b) valor= (1,3 pontos)

IO α ~ × T~
~ = OP
m~a = m~g + T~ 0

Escolhendo o eixo Z vertical de cima para baixo e o eixo X para dentro e perpendicular
ao plano da figura. Temos:

~g = g k̂; T~ = T k̂
α
~ = αı̂; ~ × T~ = rTı̂
OP
~a = ak̂; T~ 0 = −T 0k̂

Das equações acima resultam em:

IO α = rT ; (i)
ma = mg − T 0 (ii)

Para o bloco, polia e o desenrolar do fio de massa desprezı́vel e inextenı́vel temos adi-
cionalmente as condições:
a = αr e |T~ 0| = |T~ | (iii)
Das relações (i), (ii) e (iii), onde IO = 1/2MR2 = (1/2)2m(2r)2 = 4mr2 obtemos:
m g
a= g=
m + IO /r 2 5

c) valor= (0,3 pontos)


Uma volta completa corresponde ao fio desenrolar de 2πr, ou seja, o deslocamento vertical
do bloco de h = 2πr. Como a aceleração do bloco é constante e igual a a, podemos aplicar
a equação de Torricelli obtendo v 2 = 2ah, logo,
1
K= 2
(IO ω 2 ) + 12 (mv 2) = 1
2
(IO /r2 + m) v 2

K = 12 (IO /r2 + m) (2ah) = mgh ⇒ K = mg.2πr

5
d) valor= (0,4 pontos)
A força externa resultante atuando sobre o sistema bloco-polia é a força peso que atua no
bloco. O trabalho desta força, após o bloco deslocar-se de 2πr é:

WP = P~ ◦ ~h = mg.2πr

Este resultado coincide com o resultado anterior c), pois expressa o Teorema Trabalho-
Energia aplicado ao sistema bloco-polia.

6
Instituto de Fı́sica - UFRJ

Prova Final de Fı́sica IA - 2011/2

Obs: em todas as questões em que for necessário, considere que g é o módulo da aceleração da gravidade

Questão 1) Um bloco de massa m está em repouso na extremidade de uma prancha horizontal de


comprimento D e massa M. A prancha encontra-se sobre uma superfı́cie horizontal também em
repouso em relação ao referencial fixo na Terra. Num dado instante aplica-se sobre a prancha a
força horizontal F ~ constante, como mostra a figura. Os coeficientes de atrito estático e cinético
entre o bloco e a prancha são µe e µc respectivamente. As dimensões do bloco são muito
pequenas em relação ao comprimento da prancha.
a) Desenhe em um diagrama as forças que atuam no bloco e na prancha(na condição de que
o bloco não desliza sobre a prancha).
b) Que força máxima F~max pode ser aplicada na prancha tal que o bloco não deslize sobre ela?
c) Para uma força F~ de mesma direção e sentido de F ~max mas de intensidade maior, calcule
as acelerações do bloco e da prancha, em relação ao referencial fixo na Terra.
d) Quanto tempo o bloco leva para atingir a outra extremidade da prancha?

Questão 2) Uma partı́cula de massa m está sujeita a um potencial que depende da posição x,
cuja função U(x) é dada pela figura abaixo.
a) Qual é a intensidade da força derivada deste potencial que atua na partı́cula nas posições
xc e xe e o estado dinâmico da partı́cula nestas posições?
b) Se a partı́cula tem a energia mecânica E1 calcule a energia cinética e potencial da partı́cula
na posição xa .
c) Para a energia mecânica E2 calcule a velocidade da partı́cula nas posições xb , xc e xd .
Questão 3) Um pequeno bloco apoiado sobre uma mesa horizontal sem atrito possui massa m.
Ele está preso a uma corda sem massa que passa através de um buraco na superfı́cie (veja a
figura). O bloco está inicialmente executando um movimento circular uniforme de raio R e
velocidade angular ω.
a) Qual é o vetor momento angular do bloco em relação à posição do buraco? Use o sistema
de referência indicado na figura.
b) A seguir, a corda é puxada lentamente para baixo, fazendo com que o raio do cı́rculo seja
reduzido a R/2. O momento angular é conservado? Por quê?
c) Qual a nova velocidade angular?
d) Qual a variação da energia cinética do bloco?
e) Qual o trabalho realizado sobre o bloco ao puxar a corda para baixo?

Questão 4) Duas roldanas 1 e 2 de raios R1 = R e R2 = 2R, massas M1 = m e M2 = 4m estão


presas ao teto por cabos que as sustentam passando pelos respectivos eixos de rotação. Um cabo
de massa desprezı́vel e inextensı́vel é passado por suas periferias e em suas extremidades são
pendurados verticalmente dois blocos de massas m1 = 2m e m2 = m. Os blocos são liberados a
partir do repouso. Durante o movimento dos blocos o cabo ao passar pelas roldanas não desliza
e não há atrito entre as roldanas e os seus eixos de rotação. Determine:
a) a razão entre as acelerações angulares α1 /α2 das roldanas 1 e 2; justifique a sua resposta.
b) a aceleração com que os blocos se deslocam;
c) os módulos das trações nas regiões I, II e III.
Obs: considere cada roldana como se fosse um disco. O momento de inércia de um disco é
I = (1/2)MR2 , segundo um eixo que passa pelo seu centro e perpendicular a sua superfı́cie.
Questão 1

a) valor = 1,0 pontos


O diagrama de forças é dado na condição em que o bloco não desliza por:

Onde:
Pp e P~b ; são as forças peso da prancha e do bloco;
~
N~ 00 é a força que o bloco exerce sobre a prancha e N ~ 0 a sua reação;
N~ é a força que a superfı́cie exerce sobre a prancha;
f~at é a força de atrito que age sobre o bloco e f~ 0 a sua reação;
at
F~ é a força extra aplicada sobre a prancha.

b) valor = 0,5 pontos


A segunda Lei de Newton nos diz que F~res = m~a, aplicada à prancha e ao bloco temos,
de acordo com o diagrama de forças e na direção e sentido do movimento:
Prancha:

F~ + N
~ 00 + P~p + N
~ + f~ 0 = M~ap
at ⇒ 0
F − fat = Map (1)

Bloco:
P~b + N
~ 0 + f~at = m~ab ⇒ fat = mab (2)

Para que o bloco e a prancha andem juntos, ou seja, o bloco não desliza ap = ab = a.
Da equação (2) e sabendo que |f~at | = µe |N
~ 0 | = µe mg obtemos a aceleração a:

a = µe g (3)

Como o resultado anterior está ligado à condição de não deslizamento este valor de a é o
seu valor limite e também o valor limite de |F~ | igual a Fmax. Portanto de (1) e (2).

Fmax = (m + M)a ⇒ Fmax = (m + M)µe g (4)

1
c) valor = 0,5 pontos
Quando |F~ | > |F~max| então ap 6= ab e a equação (2) torna-se,

µc N 0 = mab ⇒ ab = µc g (5)

Substituindo este resultado na equação (1) temos,

F − µc mg
F − µc N = Map ⇒ F − µc mg = Map ∴ ap = (6)
M

d) valor = 0,5 pontos


Para calcular o tempo que o bloco leva deslizando sobre a prancha é necessário obter o
movimento do bloco em relação à prancha. Portanto temos, considerando um referencial
localizado na extremidade da prancha:
~ + 1~ab t2
Para o bloco: ~rb = D 2
1
Para a prancha: ~rp = ~a t2
2 p

O deslocamento do bloco em relação à prancha é:

~ + 1 (~ab − ~ap )t2


~rbp = ~rb − ~rp = D
2
Como o movimento é unidimensional, de (5) e (6),

1 F − µc mg 2 1 µc (m + M)g − F 2
rbp = D + (µc g − )t ⇒ rbp = D + t
2 M 2 M
Ao alcançar a extremidade da prancha rbp = 0 no tempo t∗, logo,
s
2DM
t∗ =
F − µc g(m + M)

2
Questão 2

a) valor = (1,0 pontos)


A força derivada do potencial(unidimensional) é F (x) = −dU(x)/dx. A derivada é
nula nas posições xc e xe , logo a intensidade de F nestas posições é nula, o que caracteriza
um estado de equilı́ibrio. Além disso temos para U(x), na posição xc um mı́nimo e para
xe um máximo. Portanto o estado dinâmico da partı́cula em xc é de equilı́brio estável e
em xe de equilı́brio instável.

b) valor = (0,5 pontos)


Pelo princı́pio da conservação da energia mecânica U(x) + K = E. Para a energia E1 ,

K = E1 − U(x)

Em x = xa,
U(xa ) = E1 ⇒ Ka = 0

c) valor = (1,0 pontos)


Para a energia mecânica E2 , K = E2 − U(x), e sendo m a massa da partı́cula,
r
2
v(x) = (E2 − U(x))
m
Portanto,
x = xb ; U(xb ) = E2 ⇒ vb = 0
r
2
x = xc ; U(xc ) = 0 ⇒ vc = E2
m

x = xd ; U(xd ) = E2 ⇒ vd = 0

As posições xb e xd são pontos de retorno e xc onde a energia cinética é máxima!

3
Questão 3

a) valor = (0,5 pontos)


De acordo com sistema de referência indicado, o momento angular é:
~l = ~r × p~ = mωR2 k̂.

b) valor = (0,5 pontos)


Sim, o momento angular é conservado pois a força resultante sobre o bloco é radial e
portanto não produz torque.

c) valor = (0,5 pontos)


Como momento angular é conservado, | ~li | = | ~lf |, onde | ~li | = mωR2 e | ~lf | = mω 0(R/2)2 ,
de modo que a nova velocidade angular é

ω 0 = 4ω.

d) valor = (0,5 pontos)


A energia cinética inicial do bloco é Ki = 21 m(ωR)2 e a energia cinética final é dada
por Kf = 12 m(ω 0 R/2)2 = 2m(ωR)2 , de modo que a variação de energia cinética é,

3
∆K = Kf − Ki = m(ωR)2 .
2

e) valor = (0,5 pontos)


Pelo Teorema Trabalho-Energia o trabalho realizado sobre o bloco pela força ex-
terna(ao puxar o bloco) é, de acordo com o resultado anterior:
3
W = ∆K = m(ωR)2 .
2

4
Questão 4

a) valor = (0.5 pontos)


Como os fios inextensı́veis não deslizam sobre as roldanas, devemos ter a1 = a2 = a.
Ou seja, o módulo da aceleração tangencial na borda de cada roldana é igual ao módulo
da aceleração dos blocos. A relação entre a aceleração tangencial e a aceleração angular
de cada roldana é dada por a = αR. Portanto, temos, com R1 = R e R2 = 2R:
α1 R2
α1 R1 = α2R2 ⇒ = = 2. (1)
α2 R1
b) valor = (1.5 pontos)
Definimos a orientação do sistema de coordenadas e desenhamos os diagramas de
forças que agem sobre cada bloco e sobre as duas polias:

As equações de movimento para os blocos são

F~1 = m1~a1 ⇒ 2ma = 2mg − TI ,


X
(2)
F~2 = m2~a2 ⇒ −ma = mg − TIII.
X
(3)

Como os fios são inextensı́veis e possuem massas desprezı́veis, segue que |T~I0| = |T~I| =
TI, |T~II0 | = |T~II| = TII, e |T~III
0
| = |T~III| = TIII. Logo, as equações de rotação para as duas
roldanas (as forças peso e normal não produzem torque) são:
X 1 a
~τ1 = I1α
~1 ⇒ mR2 = RTI − RTII,
2 R
X 1  a 
~τ2 = I2α
~2 ⇒ (4m)(2R)2 = −(2R)TIII + (2R)TII .
2 2R
Simplificando as equações acima
1
ma = TI − TII, (4)
2
2ma = −TIII + TII. (5)

5
Com as equações (2), (3), (4) e (5) obtemos o sistema:


 2ma = 2mg − TI i)




 ma = −mg + TIII ii)

1
ma = TI − TII iii)




 2



2ma = −TIII + TII iv)

A solução do sistema acima nos dá:


2
a= g. (6)
11

c) valor = (0,5 pontos)


De i) e ii) obtemos
18 13
TI = mg, TIII = mg. (7)
11 11

De iii) ou iv) segundo os resultados anteriores,


17
TII = mg. (8)
11

6
3. Uma esfera rı́gida e homogênea é lançada com velo- 5. Um bloco de massa M é comprimido aplicando-se uma
UNIVERSIDADE FEDERAL DO RIO DE JANEIRO cidade de translação V~ sobre um plano inclinado de força F~ , contra uma parede vertical, perpendicular-
INSTITUTO DE FÍSICA um ângulo θ. Imediatamente após entrar em contato mente a ela, como mostra figura. Os coeficientes de
com o plano ela passa a rolar sem deslizar velocidade atrito estático e cinético entre as superfı́cies do bloco
FÍSICA I – 2012/2 angular variável. Com relação ao seu movimento no e da parede são respectivamente µe e µc . A força
PROVA FINAL – 07/03/2013 – Turma EQN plano podemos concluir que: mı́nima para que o bloco não saia do repouso é:

VERSÃO: A (a) O momento angular total desta esfera é con-


servado.
(b) Não há força de atrito atuando sobre a esfera
Nas questões em que for necessário, considere que g é o módulo da aceleração da gravidade. (c) O centro de massa da esfera desloca-se com ve-
locidade de translação variável
(d) No centro da esfera a velocidade angular não é
Seção 1. Múltipla escolha (10×0,5 = 5,0 pontos)
nula
(e) Nenhuma das respostas anteriores. (a) 2M g/µe ;
1. Um carro desloca-se com velocidade constante ~vc = vı̂. 2. Um ioiô é pendurado ao teto por um fio ideal. Ele é
Num dadao instante ele ultrapassa um ciclista com ve- liberado verticalmente a partir do repouso como mos- (b) M g/µc ;
locidade ~vci = −(v/5)ı̂. Após o carro percorrer uma tra a figura. O seu centro de massa cai de uma altura (c) µe M g;
distância d, a distância entre eles é: h em relação a sua altura incial e, durante a queda,
(d) µc M g;
o seu centro de massa move-se verticalmente. O ioiô
(a) (6/5)d. durante a queda não gira lateralmente. A afirmativa (e) M g/µe .
(b) (4/5)d. correta sobre a dinâmica do ioiô durante a queda é:
(c) (5/6)d.
(d) 10d.
(e) d/10. 6. Uma partı́cula move-se ao longo do eixo x numa região
em que a energia potencial U (x) varia conforme a fi-
gura abaixo. A afirmativa correta é:

A B
4. Duas rodas A e B estão em rotação conectadas por
uma correia passando por suas periferias e que não
desliza. Seja RA e RB os raios das rodas A e B res-
(a) a tração do fio varia linearmente com o tempo pectivamente. Qual a razão entre os seus momentos
durante a sua queda; de inércia IA /IB , se ambas têm o mesmo momento
(b) em relação ao centro de massa do ioiô o torque angular?
resultante é nulo;
(c) em relação ao centro de massa do ioiô o torque
da tração é nulo;
(a) Na região A a força que atua na partı́cula é
(d) o torque resultante devido às forças peso e
sempre atrativa;
tração sobre o ioiô é constante;
(b) Na região B a força que atua na partı́cula é
(e) a tração do fio realiza trabalho sobre o ioiô.
sempre repulsiva.
(a) RA /RB
(c) Na posição x0 o equiı́brio é instável;
(b) RB /RA
(d) A menor energia mecânica possı́vel da
(c) 1 partı́cula corresponde a energia potencial
2 2
(d) RA /RB U (x0 );
(e) Nenhuma das resposta anteriores (e) Nenhuma das respostas anteriores.

1 2
7. Considere um canhão de massa M apoiado sobre uma 9. Dois blocos de massas m1 e m2 são ligados por uma Seção 2. Questões discursivas (2×2,5 = 5,0 pontos)
superfı́cie horizontal e sem atrito. Ele dispara uma mola ideal. Eles são afastados esticando a mola sem
bala inclinado de um ângulo θ com a horizontal. A dobrá-la. A seguir os blocos são colocados sobre uma
1. Um bloco de massa m está preso a extremidade de uma mola ideal de constante elástica k, sobre um plano inclinado
velocidade com que a bala, de massa m, é disparada superfı́cie horizontal e sem atrito a partir do repouso.
de um ângulo θ em relação a horizontal como mostra a figura. Há atrito entre as superfı́cies do bloco e do plano,
tem módulo v medida em relação a referencial fixo na é correto afirmar que:
cujo coeficiente atrito estático desconhecido µe é e cinético µc . O bloco é solto a partir do repouso na posição em
Terra. O módulo da velocidade de recuo do canhão,
que a mola se encontra relaxada.
em relação ao referencial da Terra é:
a) Desenhe o em um diagram todas as forças que agem sobre o bloco, considerando que ele entra em movimento
(a) m/(m + M )v. imediatamente após ser solto.
(b) (M/m)v. (a) o centro de massa não permanecerá na posição b) Calcule a compressão máxima da mola.
(c) (m/M )vcos θ. em que foi colocado sobre a mesa; c) Após atingir a compressão máxima, o bloco permanece em repouso. Calcule o coeficiente de atrito estático µe .
(d) (M/m)vcos θ. (b) devido a ação da mola o momento linear não é
conservado.
(e) (m/(M − m)vcos θ
(c) o bloco de maior massa terá sempre maior ve-
8. Um aro A de raio R e de massa M rola sem deslizar locidade;
sobre uma superfı́cie horizontal com velocidade de seu
centro de massa V~ . Um outro aro B de raio 2R e de (d) a energia cinética do sistema permanecerá
mesma massa do aro A, também rola sem deslizar so- constante pois o momento linear é conservado;
bre a mesma superfı́cie com a velocidade do seu centro (e) em qualquer instante |v2 /v1 | = m1 /m2 .
de massa, igual a velocidade do centro de massa do
aro A. Sabe-se que o momento de inércia de um aro 10. Um pingo de chuva de massa m cai verticalmente com
de raio R e de massa M para um eixo de rotação pas- velocidade constante de uma altura h, próximo à su-
sando pelo seu centro de massa e perpendicular ao perfı́cie da Terra. O trabalho da força de atrito do ar,
plano no qual está contido é igual a M R2 . Qual das suposta constante, durante a queda é:
√ 2. Uma barra de madeira, de comprimento ℓ e de massa M , está em repouso sobre uma mesa horizontal sem atrito.
afirmativas abaixo está correta? (a) 2gh.
Uma extremidade da barra está presa a um pino no ponto O, podendo girar sem atrito em torno dele. Um projétil
(a) Os dois aros possuem o mesmo momento an- (b) mgh de massa m, deslocando-se com velocidade de módulo v0 e perpendicular à barra, atinge a outra extremidade. O
gular, em relação oa seu centro de massa; (c) (1/2)mv 2 . projétil atravessa a barra (sem perder massa), e sai com velocidade de módulo reduzido à metade de v0 , ou seja:
(b) Os dois aros possuem a mesma energia cinética (d) mgh + (1/2)mv 2 . v0 /2. O projétil ainda mantém a mesma direção e sentido de antes de atravessar a barra. Sabe-se que o momento
total; de inércia para uma haste de massa M e comprimento L para girar em torno de um eixo perpendicular a ela
(e) nenhuma das respostas anteriores. passando por seu centro de massa é ML2 /12. Determine:
(c) Os dois aros têm a mesma velocidade angular;
a) O vetor momento angular L ~ O do projétil, em relação ao ponto O antes e depois dele atravessar a barra; use o
(d) A energia cinética de rotação do aro A é menor
que a energia cinética de rotação do aro B; sistema de eixos orientados indicados na figura.
b) O módulo da velocidade angular da barra imediatamente após o projétil atravessá-la.
(e) Nenhuma das respostas anteriores. c) Calcule a variação da energia cinética do sistema barra-projétil, considerando antes e depois do projétil atravessar
a barra.

3 4
4. Um carro desloca-se com velocidade constante ~vc = vı̂. 6. Um bloco de massa M é comprimido aplicando-se uma
UNIVERSIDADE FEDERAL DO RIO DE JANEIRO Num dadao instante ele ultrapassa um ciclista com ve- força F~ , contra uma parede vertical, perpendicular-
INSTITUTO DE FÍSICA locidade ~vci = −(v/5)ı̂. Após o carro percorrer uma mente a ela, como mostra figura. Os coeficientes de
distância d, a distância entre eles é: atrito estático e cinético entre as superfı́cies do bloco
FÍSICA I – 2012/2 e da parede são respectivamente µe e µc . A força
(a) (6/5)d. mı́nima para que o bloco não saia do repouso é:
PROVA FINAL – 07/03/2013 – Turma EQN
(b) (4/5)d.
VERSÃO: B
(c) (5/6)d.
(d) 10d.
Nas questões em que for necessário, considere que g é o módulo da aceleração da gravidade. (e) d/10.

Seção 1. Múltipla escolha (10×0,5 = 5,0 pontos)

(a) 2M g/µe ;
1. Um ioiô é pendurado ao teto por um fio ideal. Ele é 2. Uma esfera rı́gida e homogênea é lançada com velo-
liberado verticalmente a partir do repouso como mos- cidade de translação V~ sobre um plano inclinado de (b) M g/µc ;
tra a figura. O seu centro de massa cai de uma altura um ângulo θ. Imediatamente após entrar em contato (c) µe M g;
h em relação a sua altura incial e, durante a queda, com o plano ela passa a rolar sem deslizar velocidade
(d) µc M g;
o seu centro de massa move-se verticalmente. O ioiô angular variável. Com relação ao seu movimento no
durante a queda não gira lateralmente. A afirmativa plano podemos concluir que: (e) M g/µe .
correta sobre a dinâmica do ioiô durante a queda é:
(a) O momento angular total desta esfera é con-
servado.
(b) Não há força de atrito atuando sobre a esfera
(c) O centro de massa da esfera desloca-se com ve- 7. Uma partı́cula move-se ao longo do eixo x numa região
locidade de translação variável em que a energia potencial U (x) varia conforme a fi-
(d) No centro da esfera a velocidade angular não é gura abaixo. A afirmativa correta é:
nula
(e) Nenhuma das respostas anteriores. B
A
5. Duas rodas A e B estão em rotação conectadas por
uma correia passando por suas periferias e que não
desliza. Seja RA e RB os raios das rodas A e B res-
(a) a tração do fio varia linearmente com o tempo pectivamente. Qual a razão entre os seus momentos
durante a sua queda; de inércia IA /IB , se ambas têm o mesmo momento
(b) em relação ao centro de massa do ioiô o torque angular?
resultante é nulo; 3. Considere um canhão de massa M apoiado sobre uma
superfı́cie horizontal e sem atrito. Ele dispara uma
(c) em relação ao centro de massa do ioiô o torque
bala inclinado de um ângulo θ com a horizontal. A
da tração é nulo;
velocidade com que a bala, de massa m, é disparada (a) Na região A a força que atua na partı́cula é
(d) o torque resultante devido às forças peso e tem módulo v medida em relação a referencial fixo na sempre atrativa;
tração sobre o ioiô é constante; Terra. O módulo da velocidade de recuo do canhão,
em relação ao referencial da Terra é: (b) Na região B a força que atua na partı́cula é
(e) a tração do fio realiza trabalho sobre o ioiô.
sempre repulsiva.
(a) m/(m + M )v. (a) RA /RB
(c) Na posição x0 o equiı́brio é instável;
(b) (M/m)v. (b) RB /RA
(d) A menor energia mecânica possı́vel da
(c) (m/M )vcos θ. (c) 1 partı́cula corresponde a energia potencial
2 2
(d) (M/m)vcos θ. (d) RA /RB U (x0 );
(e) (m/(M − m)vcos θ (e) Nenhuma das resposta anteriores (e) Nenhuma das respostas anteriores.

1 2
8. Um aro A de raio R e de massa M rola sem deslizar 9. Dois blocos de massas m1 e m2 são ligados por uma Seção 2. Questões discursivas (2×2,5 = 5,0 pontos)
sobre uma superfı́cie horizontal com velocidade de seu mola ideal. Eles são afastados esticando a mola sem
centro de massa V~ . Um outro aro B de raio 2R e de dobrá-la. A seguir os blocos são colocados sobre uma
1. Um bloco de massa m está preso a extremidade de uma mola ideal de constante elástica k, sobre um plano inclinado
mesma massa do aro A, também rola sem deslizar so- superfı́cie horizontal e sem atrito a partir do repouso.
de um ângulo θ em relação a horizontal como mostra a figura. Há atrito entre as superfı́cies do bloco e do plano,
bre a mesma superfı́cie com a velocidade do seu centro é correto afirmar que:
cujo coeficiente atrito estático desconhecido µe é e cinético µc . O bloco é solto a partir do repouso na posição em
de massa, igual a velocidade do centro de massa do
que a mola se encontra relaxada.
aro A. Sabe-se que o momento de inércia de um aro
de raio R e de massa M para um eixo de rotação pas- a) Desenhe o em um diagram todas as forças que agem sobre o bloco, considerando que ele entra em movimento
sando pelo seu centro de massa e perpendicular ao imediatamente após ser solto.
plano no qual está contido é igual a M R2 . Qual das (a) o centro de massa não permanecerá na posição b) Calcule a compressão máxima da mola.
afirmativas abaixo está correta? em que foi colocado sobre a mesa; c) Após atingir a compressão máxima, o bloco permanece em repouso. Calcule o coeficiente de atrito estático µe .
(a) Os dois aros possuem o mesmo momento an- (b) devido a ação da mola o momento linear não é
gular, em relação oa seu centro de massa; conservado.
(b) Os dois aros possuem a mesma energia cinética (c) o bloco de maior massa terá sempre maior ve-
total; locidade;
(c) Os dois aros têm a mesma velocidade angular; (d) a energia cinética do sistema permanecerá
constante pois o momento linear é conservado;
(d) A energia cinética de rotação do aro A é menor
que a energia cinética de rotação do aro B; (e) em qualquer instante |v2 /v1 | = m1 /m2 .
(e) Nenhuma das respostas anteriores. 10. Um pingo de chuva de massa m cai verticalmente com
velocidade constante de uma altura h, próximo à su-
perfı́cie da Terra. O trabalho da força de atrito do ar,
suposta constante, durante a queda é:
√ 2. Uma barra de madeira, de comprimento ℓ e de massa M , está em repouso sobre uma mesa horizontal sem atrito.
(a) 2gh.
Uma extremidade da barra está presa a um pino no ponto O, podendo girar sem atrito em torno dele. Um projétil
(b) mgh de massa m, deslocando-se com velocidade de módulo v0 e perpendicular à barra, atinge a outra extremidade. O
(c) (1/2)mv 2 . projétil atravessa a barra (sem perder massa), e sai com velocidade de módulo reduzido à metade de v0 , ou seja:
(d) mgh + (1/2)mv 2 . v0 /2. O projétil ainda mantém a mesma direção e sentido de antes de atravessar a barra. Sabe-se que o momento
de inércia para uma haste de massa M e comprimento L para girar em torno de um eixo perpendicular a ela
(e) nenhuma das respostas anteriores. passando por seu centro de massa é ML2 /12. Determine:
a) O vetor momento angular L ~ O do projétil, em relação ao ponto O antes e depois dele atravessar a barra; use o
sistema de eixos orientados indicados na figura.
b) O módulo da velocidade angular da barra imediatamente após o projétil atravessá-la.
c) Calcule a variação da energia cinética do sistema barra-projétil, considerando antes e depois do projétil atravessar
a barra.

3 4
3. Um bloco de massa M é comprimido aplicando-se uma 6. Considere um canhão de massa M apoiado sobre uma
UNIVERSIDADE FEDERAL DO RIO DE JANEIRO força F~ , contra uma parede vertical, perpendicular- superfı́cie horizontal e sem atrito. Ele dispara uma
INSTITUTO DE FÍSICA mente a ela, como mostra figura. Os coeficientes de bala inclinado de um ângulo θ com a horizontal. A
atrito estático e cinético entre as superfı́cies do bloco velocidade com que a bala, de massa m, é disparada
FÍSICA I – 2012/2 e da parede são respectivamente µe e µc . A força tem módulo v medida em relação a referencial fixo na
PROVA FINAL – 07/03/2013 – Turma EQN mı́nima para que o bloco não saia do repouso é: Terra. O módulo da velocidade de recuo do canhão,
em relação ao referencial da Terra é:
VERSÃO: C
(a) m/(m + M )v.
(b) (M/m)v.
Nas questões em que for necessário, considere que g é o módulo da aceleração da gravidade. (c) (m/M )vcos θ.
(d) (M/m)vcos θ.
Seção 1. Múltipla escolha (10×0,5 = 5,0 pontos) (e) (m/(M − m)vcos θ

(a) 2M g/µe ;
1. Um aro A de raio R e de massa M rola sem deslizar 2. Duas rodas A e B estão em rotação conectadas por
sobre uma superfı́cie horizontal com velocidade de seu uma correia passando por suas periferias e que não (b) M g/µc ;
centro de massa V~ . Um outro aro B de raio 2R e de desliza. Seja RA e RB os raios das rodas A e B res- (c) µe M g;
mesma massa do aro A, também rola sem deslizar so- pectivamente. Qual a razão entre os seus momentos
(d) µc M g;
bre a mesma superfı́cie com a velocidade do seu centro de inércia IA /IB , se ambas têm o mesmo momento
de massa, igual a velocidade do centro de massa do angular? (e) M g/µe .
aro A. Sabe-se que o momento de inércia de um aro
de raio R e de massa M para um eixo de rotação pas- 7. Um ioiô é pendurado ao teto por um fio ideal. Ele é
sando pelo seu centro de massa e perpendicular ao 4. Uma esfera rı́gida e homogênea é lançada com velo- liberado verticalmente a partir do repouso como mos-
plano no qual está contido é igual a M R2 . Qual das cidade de translação V~ sobre um plano inclinado de tra a figura. O seu centro de massa cai de uma altura
afirmativas abaixo está correta? um ângulo θ. Imediatamente após entrar em contato h em relação a sua altura incial e, durante a queda,
com o plano ela passa a rolar sem deslizar velocidade o seu centro de massa move-se verticalmente. O ioiô
(a) Os dois aros possuem o mesmo momento an- angular variável. Com relação ao seu movimento no durante a queda não gira lateralmente. A afirmativa
gular, em relação oa seu centro de massa; plano podemos concluir que: correta sobre a dinâmica do ioiô durante a queda é:
(b) Os dois aros possuem a mesma energia cinética (a) RA /RB
(a) O momento angular total desta esfera é con-
total; (b) RB /RA servado.
(c) Os dois aros têm a mesma velocidade angular; (c) 1 (b) Não há força de atrito atuando sobre a esfera
2 2
(d) A energia cinética de rotação do aro A é menor (d) RA /RB (c) O centro de massa da esfera desloca-se com ve-
que a energia cinética de rotação do aro B; (e) Nenhuma das resposta anteriores locidade de translação variável
(e) Nenhuma das respostas anteriores.
(d) No centro da esfera a velocidade angular não é
nula
(e) Nenhuma das respostas anteriores.

5. Um pingo de chuva de massa m cai verticalmente com (a) a tração do fio varia linearmente com o tempo
velocidade constante de uma altura h, próximo à su- durante a sua queda;
perfı́cie da Terra. O trabalho da força de atrito do ar,
suposta constante, durante a queda é: (b) em relação ao centro de massa do ioiô o torque
√ resultante é nulo;
(a) 2gh.
(c) em relação ao centro de massa do ioiô o torque
(b) mgh da tração é nulo;
(c) (1/2)mv 2 . (d) o torque resultante devido às forças peso e
(d) mgh + (1/2)mv 2 . tração sobre o ioiô é constante;
(e) nenhuma das respostas anteriores. (e) a tração do fio realiza trabalho sobre o ioiô.

1 2
8. Dois blocos de massas m1 e m2 são ligados por uma 10. Uma partı́cula move-se ao longo do eixo x numa região Seção 2. Questões discursivas (2×2,5 = 5,0 pontos)
mola ideal. Eles são afastados esticando a mola sem em que a energia potencial U (x) varia conforme a fi-
dobrá-la. A seguir os blocos são colocados sobre uma gura abaixo. A afirmativa correta é:
1. Um bloco de massa m está preso a extremidade de uma mola ideal de constante elástica k, sobre um plano inclinado
superfı́cie horizontal e sem atrito a partir do repouso.
de um ângulo θ em relação a horizontal como mostra a figura. Há atrito entre as superfı́cies do bloco e do plano,
é correto afirmar que:
cujo coeficiente atrito estático desconhecido µe é e cinético µc . O bloco é solto a partir do repouso na posição em
A B que a mola se encontra relaxada.
a) Desenhe o em um diagram todas as forças que agem sobre o bloco, considerando que ele entra em movimento
imediatamente após ser solto.
(a) o centro de massa não permanecerá na posição b) Calcule a compressão máxima da mola.
em que foi colocado sobre a mesa; c) Após atingir a compressão máxima, o bloco permanece em repouso. Calcule o coeficiente de atrito estático µe .
(b) devido a ação da mola o momento linear não é
conservado.
(c) o bloco de maior massa terá sempre maior ve-
locidade;
(d) a energia cinética do sistema permanecerá (a) Na região A a força que atua na partı́cula é
constante pois o momento linear é conservado; sempre atrativa;
(e) em qualquer instante |v2 /v1 | = m1 /m2 . (b) Na região B a força que atua na partı́cula é
sempre repulsiva.
9. Um carro desloca-se com velocidade constante ~vc = vı̂.
Num dadao instante ele ultrapassa um ciclista com ve- (c) Na posição x0 o equiı́brio é instável;
locidade ~vci = −(v/5)ı̂. Após o carro percorrer uma (d) A menor energia mecânica possı́vel da
distância d, a distância entre eles é: partı́cula corresponde a energia potencial
U (x0 ); 2. Uma barra de madeira, de comprimento ℓ e de massa M , está em repouso sobre uma mesa horizontal sem atrito.
(a) (6/5)d.
(e) Nenhuma das respostas anteriores. Uma extremidade da barra está presa a um pino no ponto O, podendo girar sem atrito em torno dele. Um projétil
(b) (4/5)d. de massa m, deslocando-se com velocidade de módulo v0 e perpendicular à barra, atinge a outra extremidade. O
(c) (5/6)d. projétil atravessa a barra (sem perder massa), e sai com velocidade de módulo reduzido à metade de v0 , ou seja:
v0 /2. O projétil ainda mantém a mesma direção e sentido de antes de atravessar a barra. Sabe-se que o momento
(d) 10d.
de inércia para uma haste de massa M e comprimento L para girar em torno de um eixo perpendicular a ela
(e) d/10. passando por seu centro de massa é ML2 /12. Determine:
a) O vetor momento angular L ~ O do projétil, em relação ao ponto O antes e depois dele atravessar a barra; use o
sistema de eixos orientados indicados na figura.
b) O módulo da velocidade angular da barra imediatamente após o projétil atravessá-la.
c) Calcule a variação da energia cinética do sistema barra-projétil, considerando antes e depois do projétil atravessar
a barra.

3 4
3. Uma partı́cula move-se ao longo do eixo x numa região 5. Uma esfera rı́gida e homogênea é lançada com velo-
UNIVERSIDADE FEDERAL DO RIO DE JANEIRO em que a energia potencial U (x) varia conforme a fi- cidade de translação V~ sobre um plano inclinado de
INSTITUTO DE FÍSICA gura abaixo. A afirmativa correta é: um ângulo θ. Imediatamente após entrar em contato
com o plano ela passa a rolar sem deslizar velocidade
FÍSICA I – 2012/2 angular variável. Com relação ao seu movimento no
PROVA FINAL – 07/03/2013 – Turma EQN A B plano podemos concluir que:

VERSÃO: D (a) O momento angular total desta esfera é con-


servado.
(b) Não há força de atrito atuando sobre a esfera
Nas questões em que for necessário, considere que g é o módulo da aceleração da gravidade. (c) O centro de massa da esfera desloca-se com ve-
locidade de translação variável
(d) No centro da esfera a velocidade angular não é
Seção 1. Múltipla escolha (10×0,5 = 5,0 pontos)
nula
(e) Nenhuma das respostas anteriores.
1. Duas rodas A e B estão em rotação conectadas por 2. Um bloco de massa M é comprimido aplicando-se uma
(a) Na região A a força que atua na partı́cula é
uma correia passando por suas periferias e que não força F~ , contra uma parede vertical, perpendicular-
sempre atrativa;
desliza. Seja RA e RB os raios das rodas A e B res- mente a ela, como mostra figura. Os coeficientes de
pectivamente. Qual a razão entre os seus momentos atrito estático e cinético entre as superfı́cies do bloco (b) Na região B a força que atua na partı́cula é
de inércia IA /IB , se ambas têm o mesmo momento e da parede são respectivamente µe e µc . A força sempre repulsiva.
angular? mı́nima para que o bloco não saia do repouso é: (c) Na posição x0 o equiı́brio é instável;
(d) A menor energia mecânica possı́vel da
partı́cula corresponde a energia potencial
U (x0 );
(e) Nenhuma das respostas anteriores.

(a) RA /RB
(b) RB /RA (a) 2M g/µe ;
(c) 1 (b) M g/µc ;
2 2 6. Dois blocos de massas m1 e m2 são ligados por uma
(d) RA /RB (c) µe M g; mola ideal. Eles são afastados esticando a mola sem
(e) Nenhuma das resposta anteriores (d) µc M g; dobrá-la. A seguir os blocos são colocados sobre uma
(e) M g/µe . superfı́cie horizontal e sem atrito a partir do repouso.
é correto afirmar que:

4. Um carro desloca-se com velocidade constante ~vc = vı̂.


Num dadao instante ele ultrapassa um ciclista com ve- (a) o centro de massa não permanecerá na posição
locidade ~vci = −(v/5)ı̂. Após o carro percorrer uma em que foi colocado sobre a mesa;
distância d, a distância entre eles é: (b) devido a ação da mola o momento linear não é
conservado.
(a) (6/5)d.
(c) o bloco de maior massa terá sempre maior ve-
(b) (4/5)d. locidade;
(c) (5/6)d. (d) a energia cinética do sistema permanecerá
(d) 10d. constante pois o momento linear é conservado;
(e) d/10. (e) em qualquer instante |v2 /v1 | = m1 /m2 .

1 2
7. Um aro A de raio R e de massa M rola sem deslizar 9. Considere um canhão de massa M apoiado sobre uma Seção 2. Questões discursivas (2×2,5 = 5,0 pontos)
sobre uma superfı́cie horizontal com velocidade de seu superfı́cie horizontal e sem atrito. Ele dispara uma
centro de massa V~ . Um outro aro B de raio 2R e de bala inclinado de um ângulo θ com a horizontal. A
1. Um bloco de massa m está preso a extremidade de uma mola ideal de constante elástica k, sobre um plano inclinado
mesma massa do aro A, também rola sem deslizar so- velocidade com que a bala, de massa m, é disparada
de um ângulo θ em relação a horizontal como mostra a figura. Há atrito entre as superfı́cies do bloco e do plano,
bre a mesma superfı́cie com a velocidade do seu centro tem módulo v medida em relação a referencial fixo na
cujo coeficiente atrito estático desconhecido µe é e cinético µc . O bloco é solto a partir do repouso na posição em
de massa, igual a velocidade do centro de massa do Terra. O módulo da velocidade de recuo do canhão,
que a mola se encontra relaxada.
aro A. Sabe-se que o momento de inércia de um aro em relação ao referencial da Terra é:
de raio R e de massa M para um eixo de rotação pas- a) Desenhe o em um diagram todas as forças que agem sobre o bloco, considerando que ele entra em movimento
(a) m/(m + M )v. imediatamente após ser solto.
sando pelo seu centro de massa e perpendicular ao
plano no qual está contido é igual a M R2 . Qual das (b) (M/m)v. b) Calcule a compressão máxima da mola.
afirmativas abaixo está correta? (c) (m/M )vcos θ. c) Após atingir a compressão máxima, o bloco permanece em repouso. Calcule o coeficiente de atrito estático µe .
(a) Os dois aros possuem o mesmo momento an- (d) (M/m)vcos θ.
gular, em relação oa seu centro de massa; (e) (m/(M − m)vcos θ
(b) Os dois aros possuem a mesma energia cinética
10. Um pingo de chuva de massa m cai verticalmente com
total;
velocidade constante de uma altura h, próximo à su-
(c) Os dois aros têm a mesma velocidade angular; perfı́cie da Terra. O trabalho da força de atrito do ar,
(d) A energia cinética de rotação do aro A é menor suposta constante, durante a queda é:
que a energia cinética de rotação do aro B; √
(a) 2gh.
(e) Nenhuma das respostas anteriores. (b) mgh
8. Um ioiô é pendurado ao teto por um fio ideal. Ele é (c) (1/2)mv 2 .
liberado verticalmente a partir do repouso como mos-
(d) mgh + (1/2)mv 2 .
tra a figura. O seu centro de massa cai de uma altura
h em relação a sua altura incial e, durante a queda, (e) nenhuma das respostas anteriores. 2. Uma barra de madeira, de comprimento ℓ e de massa M , está em repouso sobre uma mesa horizontal sem atrito.
o seu centro de massa move-se verticalmente. O ioiô Uma extremidade da barra está presa a um pino no ponto O, podendo girar sem atrito em torno dele. Um projétil
durante a queda não gira lateralmente. A afirmativa de massa m, deslocando-se com velocidade de módulo v0 e perpendicular à barra, atinge a outra extremidade. O
correta sobre a dinâmica do ioiô durante a queda é: projétil atravessa a barra (sem perder massa), e sai com velocidade de módulo reduzido à metade de v0 , ou seja:
v0 /2. O projétil ainda mantém a mesma direção e sentido de antes de atravessar a barra. Sabe-se que o momento
de inércia para uma haste de massa M e comprimento L para girar em torno de um eixo perpendicular a ela
passando por seu centro de massa é ML2 /12. Determine:
a) O vetor momento angular L ~ O do projétil, em relação ao ponto O antes e depois dele atravessar a barra; use o
sistema de eixos orientados indicados na figura.
b) O módulo da velocidade angular da barra imediatamente após o projétil atravessá-la.
c) Calcule a variação da energia cinética do sistema barra-projétil, considerando antes e depois do projétil atravessar
a barra.

(a) a tração do fio varia linearmente com o tempo


durante a sua queda;
(b) em relação ao centro de massa do ioiô o torque
resultante é nulo;
(c) em relação ao centro de massa do ioiô o torque
da tração é nulo;
(d) o torque resultante devido às forças peso e
tração sobre o ioiô é constante;
(e) a tração do fio realiza trabalho sobre o ioiô.

3 4
Universidade Federal do Rio de Janeiro
Centro de Ciências Matemáticas e da Natureza
Instituto de Fı́sica
Prova Final de Fı́sica IA - EQN - 07/03/2013
Respostas para provas hı́bridas

Gabarito das Questões objetivas - 0,5 ponto cada questão

Versão A Versão B
Questão (a) (b) (c) (d) (e) Questão (a) (b) (c) (d) (e)
1 1
2 2
3 3
4 4
5 5
6 6
7 7
8 8
9 9
10 10

Versão C Versão D
Questão (a) (b) (c) (d) (e) Questão (a) (b) (c) (d) (e)
1 1
2 2
3 3
4 4
5 5
6 6
7 7
8 8
9 9
10 10

1
Questão discursiva 1) - 2,5 pontos
a) valor=1,0 ponto

b) valor=1,0 ponto
A força de atrito é a única força não conservativa que age sobre o bloco, logo:

∆E = Wfat

Escolhendo de forma apropriada as posições onde as energias potenciais são nulas, temos
que no momento em que o bloco é solto, ele possui apenas energia potencial gravitacional:

Einicial = mgh = mg(∆x)senθ

Após atingir a compressão máxima, ele tem apenas energia potencial elástica:
1
Ef inal = k(∆x)2
2

O trabalho da força de atrito é negativo, pois seu sentido é contrário ao deslocamento do


bloco sobre o plano inclinado:

Wfat = f~at · ∆~x = −mgcosθµc (∆x)

Como ∆x é diferente de zero, e ∆E = Wfat , obtém-se:


2mg
∆x = (senθ − cosθµc )
k

c) valor=0,5 ponto
Para o bloco permanecer em repouso após atingir a compressão máxima da mola, as força
resultante precisa ser nula.

P~ + N
~ + F~at + Fmola
~ = ~0

A componente perpendicular ao plano inclinado se anula com: N = mgcosθ e a compo-


nente paralela ao plano inclinado se anula como: k∆x − mgsenθ − µe mgcosθ = 0.
Dessa forma,
−2µc cosθ + senθ
µe =
cosθ

2
Questão discursiva 2) - 2,5 pontos
a) valor=1,0 ponto
Pela definição de momento angular:
~ antes = ~r × ~p = lmv0k̂
L O

e
~ apos = ~r × ~p = lm v0 k̂
L O
2

b) valor=0,5 ponto
Como não há torques externos antes e depois do processo de colisão, o momento angular
do sistema barra-projétil se conserva. Antes da colisão:
~ antes
L ~ antes
O,barra−projetil = LO,projetil = lmv0k̂

Após a colisão:
~ apos ~ apos ~ apos v0
L O,barra−projetil = LO,projetil + LO,barra = lm k̂ + IO ωO k̂
2

Ml2
Pelo teorema dos eixos paralelos: IO = ICM + M(l/2)2 =
3
Logo:

Ml2 v0 v0 3mv0
ω = lmv0 − lm = lm → ω=
3 2 2 2Ml

c) valor=0,5 ponto
Apenas o projétil possuia energia cinética. Esta energia é antes dele atravessar a barra
dada por:
1
Kantes = mv02
2

Após o projétil atravessar a barra, temos:


1 1
Kapos = m(v0/2)2 + IO ω 2
2 2

1 3 m2 2
Kapos = mv02 + v
8 8M 0

Logo:
3 3 m 3 m
∆K = − mv02 + mv02 = mv02( − 1)
8 8 M 8 M

3
3. Um fio ideal passando por um pequeno orifı́cio O, so- 5. Uma barra é constituida de dois materiais ho-
UNIVERSIDADE FEDERAL DO RIO DE JANEIRO bre uma mesa horizontal e sem atrito, liga dois corpos mogêneos, aço (parte clara da barra na figura) e ma-
INSTITUTO DE FÍSICA de dimensões desprezı́veis. As massas destes corpos deira (parte escura da barra), distribuı́dos em cada
são m e M , onde o corpo de massa M está pendu- metade da barra. Ela repousa sobre uma mesa hori-
FÍSICA I – 2012/2 rado verticalmente. O corpo de massa m descreve zontal e sem atrito. Aplica-se a mesma forca F~ , per-
PROVA FINAL – 04/03/2013 uma trajetória circular com velocidade de módulo v pendicularmente ao extremo da barra nas situações
constante; como mostra a figura. O módulo ℓ0 do mo- (A) e (B) como indica a figura, fazendo-a girar em
VERSÃO: A mento angular do corpo de massa m, em relação a O torno de um pino de articulação O na situação (A) e
é: O′ na situação (B). Considerando a massa da parte
de aço maior do que a massa da parte de madeira, as
Nas questões em que for necessário, considere que g é o módulo da aceleração da gravidade. relações entre os módulos dos torques ~τ , em relação a
O e O′ , devido à força F~ e os módulos das acelarações
angulares resultantes α, nas situações (A) e (B), são
Seção 1. Múltipla escolha (10×0,5 = 5,0 pontos) dadas por:

1. A figura abaixo representa um observador inercial O 2. Um bloco move-se sobre uma superfı́cie horizontal
fixo na superfı́cie da Terra. Ele vê uma partı́cula de com atrito. As forças que agem sobre o bloco estão (a) ℓO = m2 v 3 /M g.
massa m caindo verticalmente em queda livre. O que indicadas no diagrama abaixo, onde a força F~ faz um (b) ℓO = M 2 v 3 /mg.
se pode afirmar sobre o módulo τ0 do torque, devido ângulo φ com a horizontal. Se o bloco move-se com
(c) ℓO = m3 v 3 /M 2 g.
à força peso, e o módulo ℓ0 do momento angular, refe- velocidade constante, a opção correta é dada por:
ridos ao observador em relação ao ponto O para esta (d) ℓO = mv 3 /g.
(a) τA = τB e α A = α B ;
partı́cula durante a sua queda? (e) Nenhuma das respostas anteriores
(b) τA = τB e α A < α B ;
(c) τA < τB e α A = α B ;
(d) τA = τB e α A > α B ;
(e) τA > τB e α A > α B .

(a) ~ = P~ .
N
(b) ~ = F~ senφ + P~ .
N
(c) ~ + F~ + f~at + P~ = ~0.
N
(a) τ0 e ℓ0 são constantes. (d) F~ cosφ + f~at = ~0.
(b) τ0 é constante e ℓ0 varia linearmente com o (e) Nenhuma das respostas anteriores.
tempo.
(c) τ0 é constante e ℓ0 varia quadraticamente com
4. A figura abaixo representa dois blocos idênticos e de 6. Uma esfera homogênea e rı́gida rola sem deslizar sobre
o tempo.
dimensões desprezı́veis que giram com velocidade an- uma superfı́cie plana e horizontal com velocidade an-
(d) τ0 e ℓ0 não são constantes. gular comum constante em torno de um eixo que passa gular constante. Sabendo-se que o momento de inércia
(e) τ0 não é constante e ℓ0 é constante. perpendicularmente por O. O fio A que liga um bloco de uma esfera de raio R e de massa M segundo um
ao outro tem o mesmo comprimento que o fio B que eixo de rotação que passa pelo centro de massa é igual
liga o bloco mais interno ao centro O da trajetória, (2/5)M R2 , a razão entre a energia cinética de rotação
como mostra a figura. A razão TA /TB entre as trações da esfera em torno do centro de massa e a sua energia
dos fios A e B é: cinética de translação é dada por:
(a) 1/2 (a) 2/5;
(b) 2/3 (b) 1/5.
(c) 1/4 (c) 1/2;
(d) 1 (d) 1;
(e) 1/3 (e) 4/5.

1 2
7. Considere o processo de colisão entre duas partı́culas 9. A figura representa o gráfico v × t do movimento de Seção 2. Questões discursivas (2×2,5 = 5,0 pontos)
na ausência de forças externas. Pode-se afirmar sem- um carro que se move em um trecho retilı́neo de uma
pre que: estrada. Entre as figuras I), II), III) qual ou quais po-
1. Um bloco de massa m desliza sem atrito por um plano inclinado de um ângulo θ com a horizontal, partindo do
(a) Na colisão elástica entre partı́culas de massas deriam representar a posição do carro em função do
repouso de uma altura H até atingir uma região plana, também sem atrito. Nesta região, ele colide elasticamente
diferentes o centro de massa permanece em re- tempo correspondente à figura (A)?
com outro bloco, de massa 2m, que se encontra em repouso, fazendo com que este suba por um segundo plano
pouso após a colisão. com a mesma inclinação do primeiro. Este segundo plano tem atrito, cujo coeficiente de atrito cinético entre a sua
(b) Em uma colisão inelástica o centro de massa superfı́cie e a do bloco de massa 2m é µc . Determine:
das partı́culas diminui de velocidade após a co- a) o módulo da velocidade com que o bloco de massa m atinge o de massa 2m;
lisão. b) as velocidades dos blocos após a colisão;
(c) A velocidade do centro de massa das partı́culas c) a altura H ′ , atingida pelo bloco de massa 2m no segundo plano;
permance constante para qualquer tipo de co-
lisão entre elas.
(d) Em uma colisão totalmente inelástica o centro
de massa aumenta de velocidade após a colisão.
(e) Em uma colisão em que as partı́culas têm mas-
sas iguais o centro de massa permanece em re-
pouso após a colisão.

(a) Somente I);


(b) Somente II);
(c) Somente III);
(d) I), II) e III); 2. Uma roda é formada por um anel de massa M e raio R, atravessado por uma haste homogênea de massa m, como
(e) I) e III). indicado na figura. Inicialmente, ela gira com velocidade angular ω0 em torno do eixo que passa pelo seu centro e
é perpendicular ao plano que a contém. Um freio é então acionado. Sabendo-se que a ação do freio produz uma
10. Um corpo preso a uma mola ideal descreve um movi- força de atrito de módulo F constante e tangencial a periferia do anel (ver a figura), pede-se:
mento circular e uniforme, onde a mola permanece
8. Um móvel descreve uma trajetória circular onde o distendida de ∆ℓ. Afirma-se que: I) a aceleração a) determine o módulo da acelaração angular α, produzida pelo freio (o resultado deste item e os resultados dos
módulo de sua velocidade aumenta linearmente com o tangencial do corpo é nula. II) A aceleração radial dois itens seguintes podem ser expressos em função do momento de inércia da roda I, em torno de seu eixo de
tempo. Dos diagramas abaixo, onde as setas indicam (centrı́peta) do corpo é nula. III) A energia potencial rotação que passa pelo seu centro;
os vetores velocidade e aceleração correspondentes a da mola é nula. IV) A energia mecânica do sistema b) calcule o deslocamento angular da roda desde o instante que o freio acionado até o instante que a roda para;
c) escreva o vetor momento angular da roda, L, ~ para um dado instante t, em termos dos vetores unitários indicados
este movimento num dado instante t, o que representa massa-mola é nula. Das afirmações I), II), III) e IV)
este movimento é: as corretas são: na figura, onde k̂ aponta para fora do papel.
d) sabendo que o momento de inércia para uma haste de massa M e comprimento L para girar em torno de um
eixo perpendicular a ela passando por seu centro de massa é ML2 /12, e que o momento de inércia de um anel de
massa M e de raio R, em relação a um eixo que passa pelo seu centro de massa e perpendiculat ao plano onde o
anel está contido é igual MR2 , calcule o momento de inércia da roda (anel-haste) I, com relação ao eixo de rotação
(em termos de M, m e R).

(a) I); (a) I) e II)


(b) IV) (b) I) e IV)
(c) III); (c) Somente III)
(d) II); (d) III) e IV)
(e) Nenhuma das respostas anteriores. (e) Somente I)

3 4
Universidade Federal do Rio de Janeiro
Centro de Ciências Matemáticas e da Natureza
Instituto de Fı́sica
Prova Final de Fı́sica IA - 04/03/2013
Respostas para provas hı́bridas

Gabarito das Questões objetivas - 0,5 ponto cada questão

Versão A Versão B
Questão (a) (b) (c) (d) (e) Questão (a) (b) (c) (d) (e)
1 1
2 2
3 3
4 4
5 5
6 6
7 7
8 8
9 9
10 10

Versão C Versão D
Questão (a) (b) (c) (d) (e) Questão (a) (b) (c) (d) (e)
1 1
2 2
3 3
4 4
5 5
6 6
7 7
8 8
9 9
10 10

1
Questão discursiva 1) - 2,5 pontos
a) valor=0,5 ponto
Até o momento da colisão o bloco de massa m percorre regiões onde somente atuam forças
conservativas, portanto a energia mecânica é conservada. Considerando o zero da Energia
Potencial gravitacional localizado no plano horizontal temos:

Ei = mgH (a energia mecânica inicial)


1 2
Ed = mvm (a energia mecânica imediatamente antes da colisão)
2
Como Ei = Ed , obtemos: p
vm = 2gH

b) valor=1,0 ponto
Considerando-se o processo de colisão, na ausência de forças externas na direção hori-
zontal, o momento linear total do sistema conserva-se nesta direção: P inicial = P final.
Designemos v1i e v1f as velocidades escalares do bloco de massa m antes e depois da
colisão. Para o bloco de massa 2m, v2f é a sua velocidade escalar após a colisão. A
colisão se dá somente na direção horizontal, neste caso podemos considerá-la como sendo
unidimensional. Como a colisão é elástica, a energia cinética é conservada. Assim temos:

Para a conservação do momento linear : mv1i =mv1f + 2mv2f (1)


1 1 1
Para a conservação da energia cinética : m(v1i )2 = m(v1f )2 + 2m(v2f )2 (2)
2 2 2
As equações (1) e (2) consituem um sistema de equações.

f f
 i
 v1 = v1 + 2v2

(v1i )2 = (v1f )2 + 2(v2f )2


Resolvendo-o, por exemplo para vf1 , encontramos dois valores para v1f : v1f = v1i e v1f = − 31 v1i .
O primeiro valor não tem significado fı́sico pois implicaria que o bloco de massa m
não alterou a sua velocidade, e o bloco de massa 2m permaneceu em repouso, ou seja, é
como se não houvesse ocorrido colisão. Assim,
1 1p
v1f = − v1i → v1f = − 2gH
3 3
e
2 2p
v2f = v1i → v2f = 2gH
3 3

após a subsituição de v1i = vm = 2gH; o sinal negativo de v1f indica que o bloco de
massa m inverte o seu movimento após a colisão.

2
c) valor=1,0 ponto
O bloco de massa 2m após a colisão adquire velocidade deslocando-se para o plano incli-
nado com atrito. No plano a energia mecânica não é conservada. Assim,

∆E = Wfat (i).

8
A energia mecânica inicial para este bloco é, Ei = Ki = mgH. Ao atingir a altura
0 0
9
máxima H no plano com atrito Ef = 2mgH , pois a sua velocidade se anula nesta altura.
O trabalho da força de atrito Wfat sobre o bloco é dado por,

Wfat = f~at ◦ ~`

Onde a variação de posição no plano é igual ~`; até parar momentaneamente na posição
H0
correspondente a altura H 0. Como ` = ,
sen θ
H0
Wfat = −µc 2mgcos θ → Wfat = −µc 2mgcot θH 0
sen θ
Da equação (i) e após substituirmos os valores de Ef e Ei , obtemos
8
2mgH 0 − mgH = −µc 2mgcot θH 0
9
Finalmente,
4 H
H0 =
9 (1 + µc cot θ)

3
Questão discursiva 2) - 2,5 pontos
a) valor=1,0 ponto
A aceleração angular α é dada pela relação,

α
~=
I
onde I é o momento de inércia da roda e ~τ é o torque resultante. Neste caso somente o
freio produz torque. Temos assim,
τ = (−Rı̂) × (F ̂) → ~τ = −RF k̂
RF
Deste último resultado α
~ =− k̂. Portanto a componente da aceleração angular é,
I
RF
α=− e o seu módulo | − α| = RF/I
I
De acordo com o sistema de referência e o sentido de rotação da roda, o valor negativo
da componente do vetor α ~ indica que o vetor aceleração angular se opõe ao sentido de
rotação da roda, como era de se esperar.

b) valor=0,5 ponto
Para calcular o deslocamento angular, utilizamos a equação análoga a de Torricelli apli-
cada a rotação da roda.
ωf2 − ωi2 = 2α∆θ
RF
como ωf = 0, ωi = ω0 e com o valor de α = − , obtemos:
I
1 Iω02
∆θ =
2 RF

c) valor=0,5 ponto
O momento angular inicial L~iO , em relação ao centro da roda, tem módulo Iω0 e está no
plano perpendicular à roda, ou seja, é paralelo ao eixo z. Como a rotação é no sentido
anti-horário, temos para um instante de tempo qualquer,
~ O = Iω(t)k̂
L
RF
como ω(t) = ω0 − t.
I
~ O = I(ω0 − RF t)k̂
L → ~ O = (Iω0 − RF t)k̂
L
I

d) valor=0,5 ponto
O momento de inércia da roda é dado por IR = Ihaste + Ianel . Segundo os dados fornecidos
Ihaste = mh (2R)2 /12 e Ianel = ma R2 , logo:
1
IR = (ma + mh )R2
3

4
3. Uma bola desloca-se com velocidade ~v constante so- 5. Dois discos idênticos A e B, giram com momentos an-
UNIVERSIDADE FEDERAL DO RIO DE JANEIRO bre o tampo de uma mesa horizontal e colide elastica- gulares de mesmo módulo em torno de eixos Z e Z ′ or-
INSTITUTO DE FÍSICA mente na lateral da mesa, segundo o ângulo α definido togonais às suas superfı́cies. O eixo Z passa pela peri-
entre a direção de seu movimento e a lateral da mesa feria do disco A e o eixo Z ′ passa pelo centro de massa
FÍSICA I – 2013/1 e é claro que |~v | = |~v ′ |; como mostra a figura. O pro- do disco B, como mostra a figura. A relação ωA /ωB ,
PROVA FINAL – 31/07/2013 cesso de colisão dura um tempo ∆t. Na figura estão entre as velocidades angulares dos discos é dada por:
indicadas 4 setas numeradas de 1 a 4. A opção cuja (o momento de inércia do disco B em relação eixo Z ′
VERSÃO: A seta representa a força média que atua na lateral da é (1/2)M R2 )
mesa durante a colisão é:

Nas questões em que for necessário, considere que g é o módulo da aceleração da gravidade.

Seção 1. Múltipla escolha (10×0,5 = 5,0 pontos)

1. Uma única força atua sobre um objeto em movimento 2. Um haltere é constituı́do de duas massas m e 2m liga-
retilı́neo. O gráfico da velocidade do objeto em função das por uma barra fina e rı́gida de massa desprezı́vel
do tempo é mostrado na figura. A opção que descreve e de comprimento a. Ele é colocado em um pino O (a) 4; (a) 1/5;
o sinal do trabalho realizado pela força sobre o ob- numa parede vertical de duas maneiras A e B. Na (b) 3; (b) 1/3;
jeto em cada um dos intervalos tAB , tBC , tCD e tDE , maneira A a massa m está no pino e na maneira B
(c) 2; (c) 1
respectivamente é: ele é invertido, veja a figura. Em ambas as situações o √
haltere é alinhado horizontalmente e liberado a partir (d) 1; (d) 3;
do repouso. Se τA , αA , τB , e αB são os módulos dos (e) nenhuma das respostas anteriores. (e) 2/3.
(a) +,0,-,+ ; torques e das acelerações angulares adquiridos imedia-
tamente após liberá-los, respectivamente nas situações
(b) -, 0, -, +; A e B, é correto afirmar:
(c) +, 0, -, -;
(d) +, +, -, +;
(e) +, +, +, -.
4. Dois blocos 1 e 2 de massas iguais a m1 = 2m e
m2 = m estão presos por um fio de massa desprezı́vel
e inextensı́vel, e repousam sobre um plano horizontal 6. Um corpo é solto de uma altura H em um plano in-
sem atrito. Entre os blocos há uma mola de cons- clinado de um ângulo θ em relação a horizontal. De-
tante elástica k comprimida de uma distância d, sem vido ao atrito cinético, ao chegar na base do plano
estar presa a eles. Num dado instante corta-se o fio a sua velocidade tem módulo v. O mesmo corpo
e os blocos movem-se em sentidos opostos e mesma quando solto nas mesmas condições em plano incli-
direção, sendo v1 o módulo da velocidade do bloco 1. nado idêntico mas sem atrito, ao chegar na base deste
A distância d era igual a: plano, o módulo da sua velocidade é o triplo do caso
(a) τA > τB e α A > α B ; r anterior. O coeficiente de atrito cinético do corpo com
(b) τA > τB e α A = α B ; 5m o plano, µc , é igual a:
(a) v1 ;
k 7
(c) τA < τB e α A < α B ; r (a) tanθ;
3m 9
(d) τA > τB e α A < α B ; (b) v1 ;
k 4
(e) τA < τB e α A > α B . r (b) tanθ ;
5m 9
(c) 2v1 ; 5
k (c) tanθ;
r 9
6m
(d) v1 ; 1
k (d) tanθ;
r 9
5m v1 8
(e) ; (e) tanθ.
k 2 9

1 2
7. Num dia chuvoso uma pessoa está parada numa 9. Um pequeno corpo é lançado a partir da origem com Seção 2. Questões discursivas (2×2,5 = 5,0 pontos)
estação de trem e observa a chuva caindo inclinada de velocidade ~v0 segundo um ângulo θ com a horizontal.
um ângulo θ em relação a direção vertical. Um pas- Outro corpo é lançado (não simultaneamente) hori-
1. Um cilindro homogêneo de massa M e raio R rola sem deslizar descendo um plano inclinado de um ângulo θ, como
sageiro sentado no interior do trem que se move hori- zontalmente de uma altura h com uma velocidade ~v1
mostra a figura. Sabendo que o momento de inércia de um cilindro relativo ao seu eixo de simetria longitudinal é
zontalmente com velocidade de módulo vT em relação de mesmo módulo de v0 , como mostra a figura. Qual
M R2 /2, responda aos itens:
a estação observa a chuva caindo verticalmente. O deve ser o valor de h tal que eles atinjam o mesmo
módulo da velocidade da chuva vC em relação a pes- ponto x no eixo OX? a) em um diagrama represente todas as forças que atuam no cilindro.
soa da estação é igual a: b) calcule o módulo da aceleração do centro de massa do cilindro;
c) calcule o módulo da força de atrito estático entre a superfı́cie do plano inclinado e o cilindro.
(a) vT senθ;
(b) vT /cosθ.
(c) vT /senθ ;
(d) vT tanθ;
(e) vT cotθ;
(a) (v0 senθ)2 /2g
(b) (v0 senθ)2 /g
(c) 2(v0 senθ)2 /g
(d) (v0 sen2θ)2 /2g
(e) (v02 senθ)/2g
10. Uma esfera rola sem deslizar sobre uma superfı́cie ho-
rizontal plana com velocidade vCM no sentido positivo
do eixo horizontal OX. Num dado instante aplica-se
no seu centro de massa uma força horizontal cons-
2. Uma esfera homegênea de massa M , raio R e momento de inércia I = (2/5)M R2 , em relação ao eixo de rotação
tante de módulo F e sentido contrário ao eixo OX. A
que passa pelo seu centro de massa, é liberada a partir do repouso de uma altura H do solo em uma rampa sinuosa
distância d percorrida pela esfera até que a sua velo-
como mostra a figura. O trecho CDE da rampa é um arco de cı́rculo de raio igual 10R. Ao deslocar-se sobre a
8. Uma força exerce um impulso J sobre um corpo de cidade angular se anule será igual a: (o momento de
rampa a esfera rola sem deslizar e sem perder contato com a superfı́cie da rampa. Considere que todas as alturas
massa m, variando o módulo de sua velocidade de v inércia de uma esfera em relação ao eixo que passa
são medidas em relação à posição do centro da esfera.
para v ′ . A força e o movimento do objeto estão na pelo seu centro de massa é igual a (I = 2M R2 /5)
2 a) Qual é o módulo da velocidade angular da esfera ao passar pelo ponto mais baixo da rampa, B, cuja altura do
mesma direção e sentido. O trabalho realizado pela 5 M vCM
(a) ; solo é h?
força sobre o corpo vale: 10 F b) Qual é a altura máxima Hmax , a partir da qual, a esfera (liberada a partir do repouso) passe pelo ponto D, o
2
(a) J(v + v ′ )/2; 2 M vCM ponto mais alto do trecho CDE sem perder contato com a superfı́cie da rampa?
(b) ;
(b) J(v + v ′ ); 5 F
2
5 M vCM
(c) 2J(v + v ′ ); (c) ;
7 F
′ 2
(d) J(v + v ) /v 2
2 M vCM
(d) .
(e) não é possı́vel calcular o trabalho pois a 7 F
distância percorrida pelo corpo não é conhe- 2
7 M vCM
cida durante a ação da força sobre o corpo. (e) ;
10 F

3 4
Universidade Federal do Rio de Janeiro
Centro de Ciências Matemáticas e da Natureza
Instituto de Fı́sica
Prova Final de Fı́sica IA - 31/07/2013
Respostas para provas hı́bridas

Gabarito das Questões objetivas (valor=5,0 pontos)

Versão A Versão B
Questão (a) (b) (c) (d) (e) Questão (a) (b) (c) (d) (e)
1 1
2 2
3 3
4 4
5 5
6 6
7 7
8 8
9 9
10 10

Versão C Versão D
Questão (a) (b) (c) (d) (e) Questão (a) (b) (c) (d) (e)
1 1
2 2
3 3
4 4
5 5
6 6
7 7
8 8
9 9
10 10

1
Questão discursiva 1 (valor=2,5 pontos)
a) valor=0,3 pontos

b) valor= 1,7 pontos


O cilindro desloca-se com o movimento dado pela dinâmica de rotação e de translação.

F~ ext = M~aCM
P
translação :

~τ ext = I α
P
rotação : ~

Considerando a direção e sentido do movimento de translação do centro de massa, a rotação


em torno do eixo longitudinal do cilindro no sentido anti-horário como positiva, os torques
calculados em relação centro de massa e que ele não desliza sobre a superfı́cie temos:


 Mgsen θ − fat = MaCM 
 Mgsen θ − fat = MaCM (i)



−fatR = −Iα →
fat = IaCM /R2 (ii)

 


aCM = αR

A solução do sistema de equações da direita permite obter a aceleração do centro de massa,


Mgsen θ 2
aCM = como I = (1/2)MR2 ∴ aCM = gsen θ
(I/R2 + M) 3

c) valor=0,5 ponto
Para obter o valor da for ca de atrito podemos usar a equação (ii) do sistema de equações,
substituindo o valor da aCM obtido do item anterior. Logo,
IaCM 1
fat = → fat = Mgsen θ
R2 3

2
Questão discursiva 2 (valor=2,5 pontos)
a) valor=1,0 ponto
Podemos aplicar o princı́pio de conservação da energia mecânica ao sistema rampa esfera, pois
a única força que realiza trabalho é a força peso. Para as posições inicial (i) e B, temos:

Ei = MgH

EB = Mgh + KR + KT
1 1
Sabendo que KR = Iω 2 e KT = MvCM 2
e que Ei = EB , temos:
2 2
1 1
MgH = Mgh + IωB2 + MvCM 2
2 2
Como a esfera rola sem deslizar vCM = ωB R e I = ICM = (2/5)MR2 substituindo estes valores
na igualdade anterior,
1 2 2
2 ωB 1 
2 2
MgH = Mgh + . MR 2 + .MωB R
2 5 R 2
r
1 10
ωB = g(H − h)
R 7
b) valor=1,5 pontos
Analogamente ao item anterior para aconservação de energia mecânica, com H = Hmax e que
hD é a altura na posição D, após aplicarmos a condição de que a esfera rola sem deslizar, temos:
Ei = MgHmax

7 2
ED = MghD + KR + KT = MghD + MvCM
10
7 2
Igualando Ei com ED obtemos MgHmax = MghD + MvCM .
10
Na altura Hmax temos a condição limite da esfera perder contato com a superfı́cie da rampa.
Na posição D pela segunda Lei de Newton N ~ + P~ = F~R , onde N~ é força de contato (normal),
P~ o peso da esfera e FR a força resultante (radial). No limite de perder o contato com a rampa
N~ → ~0. Logo P~ = F~R, em módulo, e note que o centro de massa da esfera percorre o arco de
raio R + RCDE .
2 2
Mg = FR = MvCM /R → vCM = (RCDE + R)g
2
Como o raio de curvatura no trecho CDE, RCDE = 10R, vCM = 11Rg Substituindo este
último resultado na igualdade da conservação de energia,
7
MgHmax = MghD + 11MRg
10

11
∴ Hmax = hD + 7 R
10
Observação: na versão anterior do gabarito foi usado que o centro de massa da esfera percorre o
arco de raio RCDE = 10R, de tal modo a obtermos o resultado Hmax = hD + 7R; este resultado
também será aceito como uma aproximação correta.

3
3. Um projétil puntiforme de massa m é lançado de um 5. Uma partı́cula de massa m está pendurada no teto por
UNIVERSIDADE FEDERAL DO RIO DE JANEIRO
ponto O de um plano horizontal com velocidade de uma mola de constante elástica k. Se a partı́cula é
INSTITUTO DE FÍSICA módulo v0 e ângulo θ0 de lançamento, onde 0 < θ0 < solta com velocidade nula na posição em que a mola se
FÍSICA I – 2013/2 π/2. O projétil atinge uma altura máxima H e um al- encontra relaxada e na vertical, podemos afirmar que a
cance A. Se LH o é o módulo do momento angular do mola estica de uma distância máxima h igual a
PROVA FINAL – 9/12/2013 projétil ao atingir a altura máxima H e LA
o , o módulo

VERSÃO: A do momento angular quando ele atinge o alacance A,


ambos calculados em relação a O, então (a) mg/k;
(b) mg/2k;
Nas questões em que for necessário, considere que g é o módulo da aceleração da gravidade. (c) 2mg/k;
r
mgh
(d) ;
Seção 1. Múltipla escolha (10×0,5 = 5,0 pontos) k
s
k
(e) .
1. Uma astronauta com sua caixa de ferramentas encontra- 2. Um bloco de massa m está em translação retilı́nea sobre mg
se no espaço sideral próxima à sua nave, todos muito uma superfı́cie horizontal lisa empurrado por uma força
(a) LH A
o = Hmv0 sen θ0 e Lo = A mv0 cosθ0 ;
afastados do restante do universo e com velocidades nu- constante de módulo F que faz um ângulo φ com a ho-
las relativamente a um referencial inercial. Em um dado rizontal, como mostra a figura. O módulo N da força A
(b) LH
o = mv0 cos θ0 e LA
o = A mv0 senθ0 ;
instante a astronauta arremessa sua caixa de ferramentas normal que a superfı́cie exerce sobre o bloco e o módulo 2 6. Um disco rola sem deslizar sobre um plano horizontal,
H A
com velocidade de módulo v, conforme indica a figura; a da aceleração do bloco são, respectivamente, (c) Lo = Hmv0 cos θ0 e Lo = A mv0 senθ0 ; mantendo-se sempre na vertical. Sejam em um certo
com isso a astronauta percorre uma distância D até che- A instante os pontos A, B e C localizados na perferia do
gar à nave. Sabendo-se que a massa da astronauta é M e (d) LH
o = mv0 sen θ0 e LAo = A mv0 cosθ0 ;
2 disco como mostra a figura. Para um referencial fixo no
a da caixa de ferramentas é m, conlcuı́mos que o tempo (e) H A
Lo = H mv0 cos θ0 e Lo = 2H mv0 senθ0 . plano, se v é a velocidade do centro de massa do disco, e
que a astronauta leva para percorrer a distância D é vA , vB e vC são os respectivos módulos das velocidades
dos pontos A, B e C, então

(a) vA = 2 v;
(a) mg e (F sen φ)/m; (b) vB = 2v;
(b) mg e (F cos φ)/m; (c) vC = v;

(c) mg + F sen φ e (F sen φ)/m; (d) vB = 2 v.
(d) mg − F sen φ e (F cos φ)/m (e) vA = v
(e) mg + F sen φ e (F cos φ)/m;
4. Uma barra de comprimento ℓ e massa M repousa, sem
estar fixa, sobre uma mesa horizontal lisa (sem atrito).
7. Uma partı́cula de massa m pendurada por um fio ideal
Aplicam-se à barra, simultaneamente, três forças F~1 ,
de comprimento ℓ, cuja extremidade é presa ao teto, é
F~2 e F~3 perpendiculares a ela, de mesma intensidade
abandonada em repouso com o fio esticado fazendo um
F e de sentidos como indicados na figura. As forças
ângulo θ0 com a vertical (0 < θ0 < π/2). Sejam, T~ a
F~1 e F~3 são aplicadas nas extremidades e a força F~2 no
força do fio sobre a partı́cula, P~ o seu peso e ~v a sua
centro de massa da barra. Os módulos da acelaração
(a) Dm/M v; velocidade, todos os três vetores no instante em que a
do centro de massa da barra e do torque resultante
(b) DM/mv. partı́cula passa pelo ponto mais baixo de sua trajetória.
sobre ela relativo ao seu centro de massa, imediata-
A a opção correta é
(c) D/v; mente após aplicação das forças, são, respectivamente,
(d) DM/[(m + M )v]; (a) F/M e zero; (a) T − P = mv 2 /ℓ
(e) Dm/[(M + m)v]; (b) 2F/M e zero; (b) T + P = mv 2 /ℓ
(c) 3F/M e F ℓ; (c) T = mv 2 /ℓ
(d) 2F/M e F ℓ/2; (d) T~ + P~ = ~0
(e) 2F/M e 2F ℓ. (e) T~ = P~ .

1 2
8. Dois projéteis 1 e 2 são lançados simultaneamente e ho- 10. A figura mostra um trilho perfeitamente liso contido Seção 2. Questões discursivas (2×2,5 = 5,0 pontos)
rizontalmente de alturas h1 e h2 (diferentes) em relação em um plano vertical. Uma partı́cula é abandonada em
ao solo. Verifica-se que ambos atingem o mesmo ponto repouso no ponto P1 do trilho e desliza sobre ele sem Não serão consideradas respostas sem justificativa; expresse-as somente em função dos dados fornecidos.
A no solo. A razão v1 /v2 entre os módulos v1 e v2 das nunca perder contato. A partı́cula passa pelos pontos
velocidades de lançamento dos respectivos projéteis 1 e P2 , P3 , P4 e atinge o ponto P5 , localizado em uma linha 1. Um bloco de massa m, pressionado sobre uma pa-
2 é horizontal passando por P1 . No percurso de P1 a P5 a rede vertical por uma força horizontal constante
energia cinética da partı́cula é F~ , como indica a figura, desce verticalmente com
p aceleração para baixo. Os coeficiente de atrito
(a) h1 /h2 ; cinético e estático entre o bloco e a parede são,
p respectivamente, µc e µe .
(b) h2 /h1 .
a) Faça um diagrama das forças sobre o bloco;
(c) h2 /h1 ;
b) calcule o módulo a da aceleração desse bloco;
(d) h1 /h2 ; c) determine o módulo da força total F~c exercida
p
(e) h2 /2h1 . pelo bloco sobre a parede.
d) Se, em vez de descendo, o bloco estivesse
9. Uma partı́cula desloca-se ao longo do eixo x sob a ação
em repouso, qual seria o valor mı́nimo Fmin do
de uma força conservativa F~ , correspondente a uma
módulo da força horizontal F~ que manteria o
energia potencial U (x), dada pelo gráfico da figura, na
(a) nula em P1 e P5 e máxima em P3; bloco em repouso.
qual o ponto C é um ponto de mı́nimo. Para este po-
tencial entre as opções abaixo a única incorreta é (b) nula em P1 e P5 e mı́nima em P2 e P4 ;
(c) nula em P1 , P3 e P5 ;
2. Um bloco de massa m está preso a um fio de massa
(d) nula em P1 e P5 ;
desprezı́vel e inextensı́vel, cuja outra extremidade
(e) nula em P1 , P2 , P3 , P4 e P5 ; está enrolada na periferia de um disco homogêneo
de massa M e raio R, que pode girar sem atrito
em torno de um eixo fixo horizontal. O bloco é
abandonado a partir do repouso e desce vertical-
mente fazendo o disco girar sem que haja desliza-
mento do fio sobre o disco. Dado que o momento
de inércia do disco relativo ao seu eixo de rotação
é (1/2)M R2 e que a massa do disco é o dobro da
massa do bloco, M = 2m, calcule
(a) na posição xC , tem-se a condição de equilı́brio
a) o módulo a da aceleração do bloco em movi-
estável;
mento;
(b) no deslocamento do corpo de xB para xC o tra- b) o módulo T da tração do cabo e o módulo Fe da
balho realizado pela força F~ é positivo; força que o suporte exerce sobre o disco em movi-
(c) o sentido da força F~ na posição xB é positivo; mento;
c) a energia cinética do bloco após o disco dar uma
(d) na posição xC a força F~ é nula. volta completa a partir do repouso.
(e) na posição xB a força sobre a partı́cula é nula;

3 4
Universidade Federal do Rio de Janeiro
Centro de Ciências Matemáticas e da Natureza
Instituto de Fı́sica
Prova Final de Fı́sica IA - 9/12/2013
Respostas para provas hı́bridas

Gabarito das Questões objetivas (valor=5,0 pontos)

Versão A Versão B
Questão (a) (b) (c) (d) (e) Questão (a) (b) (c) (d) (e)
1 1
2 2
3 3
4 4
5 5
6 6
7 7
8 8
9 9
10 10

Versão C Versão D
Questão (a) (b) (c) (d) (e) Questão (a) (b) (c) (d) (e)
1 1
2 2
3 3
4 4
5 5
6 6
7 7
8 8
9 9
10 10

1
Questão discursiva 1 (valor=2,5 pontos)
a) valor=0,4 ponto
No diagrama estão representadas as forças que agem sobre o
bloco, onde P~ é o peso do bloco, f~c a força de atrito cinética
eN~ a força que a parede exerce sobre o bloco.

b) valor=1,2 pontos
Pela segunda Lei de Newton F~ + f~c + N
~ + P~ = m~a e decompondo as forças segundo as direções
horizontal e vertical temos:

na direção horizontal: N − F = 0 (i)

na direção vertical: P − fc = ma (ii)

Como |f~c | = µc N = µc F e P = mg, de (ii)


P − fc µc
a= → a=g− F (iii)
m m

c) valor=0,4 ponto

A força F~C0 que o bloco exerce sobre a parede é obtida pela


soma das reações ~ 0 da normal e f~ 0 da força de atrito. Por-
p N c
~ 0 0 2 0 2
tanto |FC | = N + fc , como N = N e fc0 = fc = µc F ,
0

|F~C0 | = F 2 + (µc F )2
p

|F~C0 | =
p
1 + µ2c F

d) valor=0,5 ponto
A força mı́nima que permite manter o bloco em repouso corresponde fisicamente a a = 0. Neste
caso µc → µe e F~ = F~min . Portanto do resultado da aceleração (iii),
µe mg
0=g− Fmin → Fmin =
m µe

2
Questão discursiva 2 (valor=2,5 pontos)

a) valor=1,6 pontos
A dinâmica para o disco e o bloco corresponde a:
X
disco: ~τiext = I α
~
i

F~iext = m~a,
X
bloco:
i

além da condição de vı́nculo a = αR.


Assim temos, considerando o sentido de rotação positivo como
anti-horário e que |T~ | = |T~ 0 |:


 RT = Iα 
 T = (1/2)2mR2 /R2 = ma (i)



mg − T = ma ⇒
mg − T = ma

 


a = αR

1
∴ a= g
2

b) valor=0,5 ponto
A tração do fio pode ser obtida de (i), após substituirmos o valor de a encontrado no item
anterior,
mg
T = ma → T = .
2
Para a força F~e que o eixo exerce sobre o disco temos, T~ + F~e + M~g = ~0. Assim, como M = 2m,
1 5
Fe = Mg + T = 2mg + mg → Fe = mg
2 2

c) valor=0,4 ponto
Quando o disco gira de 2π o bloco cai da distância d = 2πR. Logo ∆K = Wtotal = Kf , pois
Ki = 0.
O trabalho total é dado pelo trabalho das forças T~ e m~g que agem sobre o bloco.
1
Kf = (mg − T )2πR = mg2πR
2

∴ Kf = πmgR

Obs: uma maneira simples de calcular Wtotal sobre o bloco é fazer Wtotal = Fres .d = ma.2πR
g
= m 2πR = πmgR !!
2

3
3. Duas partı́culas A e B de massas iguais colidem uni- 5. Dois discos A e B de mesmo raio R, constituı́dos de
UNIVERSIDADE FEDERAL DO RIO DE JANEIRO
dimensionalmente ao longo do eixo OX. Os gráficos a materiais homogêneos, repousam sobre uma mesa hori-
INSTITUTO DE FÍSICA seguir representam as velocidades em função do tempo zontal lisa. Aplica-se a mesma forca F~ a cada disco, tan-
FÍSICA I – 2014/1 para cada partı́cula. Se P~ é o momento linear do sistema gencialmente às suas respectivas periferias, como indica
formado pelas duas partı́culas e K sua energia cinética, a figura, fazendo-os girar em torno de um pino central
PROVA FINAL – 4/06/2014 podemos afirmar que nessa colisão O no disco A e O′ no disco B. Considerando a massa
VERSÃO: A do disco A maior do que a massa do disco B, as relações
entre os módulos dos torques τA e τB , em relação a O e
O′ , devido à força F~ e os módulos das respectivas ace-
Nas questões em que for necessário, considere que g é o módulo da aceleração da gravidade. larações angulares resultantes αA e αB são dadas por

Seção 1. Múltipla escolha (10×0,5 = 5,0 pontos)

1. Duas escadas rolantes estão dispostas como mostra a fi- 2. Três partı́culas de massas iguais a m movem-se na
gura. O ângulo de elevação θ de cada escada, em relação ausência de forças externas com velocidades ~v1 = −vı̂,
(a) P~ se conserva e K não se conserva;
a horizontal é o mesmo. As escadas movimentam os de- ~v2 = −vı̂, e ~v3 = −vı̂ +v̂. Num dado instante elas ocu-
graus com velocidades constantes de mesmo módulo v, pam as posições indicadas na figura abaixo. O vetor (b) P~ se conserva e K se conserva;
medidas em relação a um referencial fixo na Terra. Em momento angular total L ~ O deste sistema de partı́culas
(c) P~ não se conserva e K não se conserva;
um dado instante, duas pessoas A e B entram simulta- em relação a origem O do eixo de coordenadas da figura
neamente nas escadas, como mostra a figura, e ao en- é (d) P~ não se conserva e K se conserva; (a) τA = τB e α A = α B ;
trarem permanecem paradas em relação aos respectivos (e) P~ 6= ~0 e K = 0. (b) τA = τB e α A < α B ;
degraus. Seja um referencial fixo em B e em translação (c) τA < τB e α A = α B ;
relativa ao referencial da Terra. O módulo da velocidade
de A em relação B enquanto A e B estão nas escadas é (a) ~0. (d) τA = τB e α A > α B ;

(b) −2avmk̂. (e) τA > τB e α A > α B ;

(c) avmk̂. 6. Uma esfera é lançada rolando sem deslizar subindo um


plano inclinado com velocidade inicial ~v do seu centro de
(d) −avmk̂ massa. Um bloco é lançado subindo em translação um
(e) −3avmk̂. plano inclinado com a mesma inclinação e com a mesma
velocidade inicial ~v do seu centro de massa. No plano
onde se encontra a esfera há atrito suficiente para que
(a) |~vA/B | = 0; 4. Dois discos de massas 2m e 3m estão ligados por uma ela permaneça rolando sem deslizar e no plano onde está
(b) |~vA/B | = vsen θ; barra rı́gida de massa 5m sobre uma mesa horizontal o bloco não há atrito. A relação entre a altura máxima
lisa. Uma força F~ paralela ao plano da mesa está apli- Hb que o bloco atinge e a altura máxima He que a esfera
(c) |~vA/B | não é constante; cada sobre o primeiro disco em um certo instante, como atinge nos respectivos planos é dada por
(d) |~vA/B | = 2vcos θ mostra a figura (da mesa vista de cima). Nesse instante,
(e) |~vA/B | = vcos 2θ a aceleração do centro de massa do sistema constituı́do
pelos discos e pela barra é

F~
(a) ;
5m
F~ (a) Hb = He , pois tanto a energia cinética do bloco
(b) ; como a da esfera se conserva.
10m
F~ (b) Hb = He , pois tanto a energia mecânica do bloco
(c) ; como a da esfera se conserva.
3m
F~ (c) Hb > He , pois o bloco desliza sem atrito.
(d) ;
2m (d) He > Hb , pois a esfera rola sem deslizar.
F~ (e) He > Hb , pois tanto a energia mecânica do bloco
(e) .
8m como a da esfera se conserva.

1 2
7. Uma partı́cula percorre uma trajetória circular de raio 9. Uma partı́cula desloca-se ao longo do eixo x sob a ação Seção 2. Questões discursivas (2×2,5 = 5,0 pontos)
R onde, no trecho do arco AB, o módulo da sua velo- de uma força conservativa F~ , correspondente a uma
cidade é reduzido uniformemente, isto é, com derivada energia potencial U (x), dada pelo gráfico da figura, na Não serão consideradas respostas sem justificativa; expresse-as somente em função dos dados fornecidos.
constante. No ponto A a sua velocidade é ~vA , con- qual o ponto B é um ponto de máximo e C é um ponto
forme ilustrado na figura. Considere a sua aceleração ~a de mı́nimo. Para esse potencial, entre as opções abaixo 1. Um bloco A de massa m está sobre uma mesa horizontal sendo em-
decomposta segundo duas direções, um componente na a única incorreta é purrado em movimento retilı́neo por uma força constante F~ dada,
direção radial (aceleração radial) e o outro componente com velocidade e aceleração de mesmo sentido que a força; o coe-
na direção tangente à trajetória (aceleração tangencial). ficiente de atrito cinético entre o bloco e a mesa é µ. Sobre esse
No seu movimento de A para B a afirmativa correta é primeiro bloco há um segundo bloco B de mesma massa m e rigi-
damente colado ao primeiro, como indica a figura.
a) Faça um diagrama de forças externas sobre o sistema constituı́do
pelos dois blocos;
b) calcule o módulo a da aceleração com a qual os blocos se
deslocam;
c) Calcule os módulos da força horizontal e da força vertical que o
bloco A exerce sobre o bloco B.
(a) a aceleração ~a tem módulo constante;
(b) a aceleração radial tem módulo constante; (a) na posição xB a força sobre a partı́cula é nula;
(c) a aceleração tangencial tem módulo constante; (b) na posição xC , tem-se a condição de equilı́brio
estável; 2. Dois blocos de massas m e 3m estão ligados por um fio ideal que
(d) a aceleração tangencial é sempre nula; passa por uma roldana homogênea, também de massa m, e de raio
(c) o sentido da força F~ na posição xA é positivo; R. A roldana é sustentada por um eixo horizontal coincidente com
(e) nenhuma das respostas anteriores é correta.
(d) no deslocamento do corpo de xB para xC o tra- seu eixo geométrico de simetria, como ilustra a figura. Não há atrito
balho realizado pela força F~ é positivo; entre o eixo e a roldana e o fio não desliza sobre ela. Considere o
intervalo de tempo com um instante inicial em que a roldana e os
(e) na posição xC a força F~ é nula.
blocos estão em repouso e um instante final em que o bloco mais
10. Um bloco encontra-se em repouso sobre um plano incli- pesado já desceu uma distância vertical h. Considere como dados
nado como indicado na figura. Dentre os diagramas (I), m, R, h e o módulo g da aceleração da gravidade. O momento de
(II), (III) e (IV), o que melhor representa (ilustrada por inércia da roldana relativo ao seu eixo de simetria é mR2 /2. Calcule
uma seta) a força total que o bloco exerce sobre o plano a) a variação da energia cinética do sistema constituı́do pela roldana
inclinado é e os blocos no intervalo de tempo considerado;
b) o módulo da velocidade do bloco mais pesado no instante final;
c) a razão entre a energia cinética de rotação da roldana e a energia
cinética total de translação dos dois blocos no instante final.
8. Dois projéteis 1 e 2 são lançados simultaneamente e ho-
rizontalmente de alturas h1 e h2 (diferentes) em relação
ao solo. Verifica-se que ambos atingem o mesmo ponto
C no solo e têm o mesmo alcance A1 = A2 = A. A
razão v1 /v2 entre os módulos v1 e v2 das velocidades de
lançamento dos respectivos projéteis 1 e 2 é

(a) h1 /h2 ;
(a) nenhum dos diagramas;
(b) h2 /h1 ;
p (b) (I);
(c) h1 /h2 ;
(d)
p
h2 /h1 .
(c) (III);
(e)
p
h2 /2h1 . (d) (II);
(e) (IV);

3 4
Universidade Federal do Rio de Janeiro
Centro de Ciências Matemáticas e da Natureza
Instituto de Fı́sica
Prova Final de Fı́sica IA - 04/06/2014
Respostas para provas hı́bridas

Gabarito das Questões objetivas (valor=5,0 pontos)

Versão A Versão B
Questão (a) (b) (c) (d) (e) Questão (a) (b) (c) (d) (e)
1 1
2 2
3 3
4 4
5 5
6 6
7 7
8 8
9 9
10 10

Versão C Versão D
Questão (a) (b) (c) (d) (e) Questão (a) (b) (c) (d) (e)
1 1
2 2
3 3
4 4
5 5
6 6
7 7
8 8
9 9
10 10

1
Questão discursiva 1 (valor=2,5 pontos)
a) valor=0,4 pontos
As forças que agem sobre o sistema formado pelos blocos A e B estão representadas na figura.
As setas indicam o sentido e direção e ao lado delas os seus módulos, como no livro texto. No
diagrama alternativo R corresponde à força resultante de contato, P o peso total dos blocos e
F a força aplicada.

b) valor=1,6 pontos
Pela segunda Lei de Newton, F~res = R ~ + F~ + f~at + P~ = m~a. Obtemos para os componentes
das forças na direção vertical e horizontal as equações,


 F − fat = 2ma (i)

N − 2mg = 0 (ii)


fat = µN (iii)

Das equações (ii) em (iii), obtemos fat = 2µmg. Este resultado ao ser substituido em (i) nos
fornece o valor de a,
F
a= − µg
2m
c) valor=0,5 ponto
Para o bloco B temos de acordo com a figura e aplicando a segunda Lei
de Newton, nas direções vertical e horizontal,
fat(BA) = ma
(

NBA = mg

Com as equações acima e com o valor de a obtido no item anterior, o


módulo das forças fH horizontal e fV vertical que o bloco A exerce sobre
o bloco B são,
F
fH = − µmg
2
fV = mg

2
Questão discursiva 2 (valor=2,5 pontos)
a) valor=1,0 ponto
Como não há forças dissipativas atuando sobre o sistema, a energia mecânica do sistema formado
pela roldana e os blocos é conservada, ∆E = ∆K + ∆U = 0, logo,

∆K = −∆U = −(∆U1 + ∆U2)) = −(−3mgh + mgh)

∆K = 2mgh

b) valor=1,1 pontos
O procedimento para obter a velocidade do bloco de maior massa consiste em:

• obtermos as variações de energia cinética de cada constituinte do sistema formado pelo


blocos e pela roldana.
∆K = ∆Kb1 + ∆Kb2 + ∆KRold

Temos assim: ∆Kb1 = (1/2)3mvf2 , ∆Kb2 = (1/2)mvf2 e ∆KRold = (1/2)Iω 2

• uma vez que os blocos adquirem a mesma velocidade e a roldana gira com velocidade
angular ω = v/R vinculada ao movimento dos blocos, podemos expressar a variação de energia
cinética total em função da velocidade final dos blocos vf ; lembrando que I = (1/2)mR2 .

1 1 vf2  9
∆K = 3mvf2 + mvf2 + mR2 2 = mvf2
2 2 R 4

• comparar a variação de energia cinética com o resultado obtido no item a) ∆K = 2mgh.

Logo
r
9 2 8
mv = 2mgh ∴ vf = gh
4 f 9

c) valor=0,4 pontos
No cálculo da razão entre a energia cinética de rotação da roldana KRot e a energia cinética
total de translação dos blocos KT temos,

1 1 2 vf2
KRot ( mR )
=
2 2
R2 = 1
1 2
KT 2
(4mvf ) 8

3
Instituto de Fı́sica - UFRJ

Prova de Segunda Chamada de Fı́sica IA - 2011/1

Obs: em todas as questões em que for necessário, considere que g é o módulo da aceleração da gravidade

Questão 1) Uma partı́cula está sobre uma região plana horizontal sob a ação de uma força
constante. No instante inicial t0 as condições iniciais são ~v = v0 ̂ e ~r0 = −dı̂ e a sua aceleração
~a = aı̂, como mostra a figura abaixo.

a) Escreva as expressões horárias vetoriais para a posição, ~r(t), e velocidade, ~v (t), para um
instante qualquer.
b) A partı́cula intercepta o eixo OY ? Em caso afirmativo qual é a sua ordenada?
c) Determine a trajetória da partı́cula e faça o seu gráfico tendo por base a figura apresentada.

Questão 2) Uma roldana sustenta dois blocos de massas diferentes, mA > mB , ligados por um
~ , constante e vertical, como
fio de massa desprezı́vel e inextensı́vel, por meio de uma força F
mostra a figura abaixo. Os blocos estão apoiados sobre uma superfı́cie horizontal, a roldana tem
massa desprezı́vel e o fio não desliza sobre a roldana.

a) Qual é a condição para que ambos os blocos permaneçam na superfı́cie?


b) Quando |P ~B | < | F
~ /2| < |P
~A |, determine os módulos das acelerações dos blocos A e B.
c) Quando |F ~ /2| > |P~A |, determine os módulos das acelerações dos blocos A e B.
Questão 3) Um disco encontra-se, em repouso sobre uma mesa horizontal, encaixado em um
pino em torno do qual pode girar livremente. O disco tem um sulco na sua periferia onde está
uma pequena esfera de massa m, e todo o sistema encontra-se em repouso. Num dado instante
a esfera recebe um impulso perpendicular ao raio do disco aplicado no seu centro. A esfera
adquire uma velocidade ~v0 , em relação ao disco, e passa a rolar sem deslizar pelo sulco do disco.
O raio da esfera é desprezı́vel em comparacção com o raio do disco R. O disco tem massa M e
o seu momento de inércia em relação ao pino(eixo de rotação), é (1/2)MR2 .
a) Calcule o momento de inércia do sistema disco-esfera, em relação ao eixo de rotação.
b) Que velocidade angular adquire o disco após o impulso recebido pela esfera? Justifique a
sua resposta.

Questão 4) Uma esfera homogênea de raio r e massa m, é solta, a partir do repouso, dentro de
uma tigela hemisférica de raio R, a um dado ângulo θ, como mostrado na figura abaixo.
A superfı́cie da metade esquerda da tigela possui atrito suficiente para a esfera rolar sem deslizar,
enquanto a da metade direita está coberta com um óleo eliminando qualquer atrito. O momento
de inércia de uma esfera em relação a um eixo passando pelo centro de massa é I = (2/5) m r2 .
Em função dos dados acima e de g, determine:
a) a velocidade angular ωF da esfera ao atingir o fundo da tigela;
b) a fração translacional KT /Ktot e rotacional KR /Ktot da energia cinética total Ktot desde
o inı́cio da descida até o fundo da tigela;
c) o ângulo máximo θmax a partir da vertical que ela atinge no lado sem atrito.
Questão 1

a) valor = (0,5 pontos)


As equações horárias vetoriais são, de acordo com as condições iniciais:
1 2

~r (t) = ~
a t − d ı̂ + v0t̂


 2

 ~v(t) = d~r (t) = ~atı̂ + ~v ̂



0
dt

b) valor = (1,0 pontos)


Podemos escrever a equação horária da posição como: ~r(t) = rx (t)ı̂+ry (t)̂. Se a partı́cula
cortar o eixo OY deveremos ter ~rx (t∗) = 0, com uma solução real para ry (t∗ ), em t = t∗.
Aplicando esta condição temos:

rx (t∗) = 0 = 21 a(t∗)2 − d i)

ry (t∗) = v0t∗ ii)
r
2d
Da equação i) temos que t∗ = , uma solução possı́vel. Após substituir este resul-
a
tado em ii) obtemos a ordenada que a a partı́cula corta o eixo OY ,
r
2d
ry (t∗ ) = v0
a

c) valor = (1,0 pontos) Para obtermos a trajetória devemos eliminar o parâmetro


tempo e escrever a ordenada como função da abscissa ou vice-versa. Temos:

rx (t) = rx = 12 at2 − d

ry (t) = ry = v0t → t = ry /v0
Portanto,
1 ry  2 1a 2
rx = −d + a ⇒ rx = r −d
2 v0 2 v02 y
A equação
q acima corresponde a uma parábola com vértice em -d e cortando o eixo
OY em v0 2d a
; vide a figura abaixo.

1
Questão 2

a) valor = (0,8 pontos)


Sendo a roldana ideal a intensidade da força F~ distribui-se igualmente no fio pelos
dois lados da roldana. Para que ambos os blocos permaneçam em contato com o piso no
mı́nimo terı́amos a condição da iminência de se movimentarem dada por:

 F/2 = PA = mA g

F/2 = PB = mB g

Como mA > mB basta que F/2 seja menor que PB , ou,

|F~ | < |2P~B |

b) valor = (0,8 pontos)


|P~B | < |F~ /2| < |P~A |
Neste caso o bloco de massa mB passa a se movimentar e o bloco de massa mA
permanece parado. Logo,

F/2 − PB
FB = F/2 − PB = mb aB ⇒ aB =
mB

c) valor= (0,9 pontos)


|F~ /2| > |P~A |
Como mA > mB ambos os blocos movimentam-se. Analogamente ao item anterior,
obtemos:

F/2 − PA F/2 − PB
aA = e aB =
mA mB

2
Questão 3

a) valor = (1,0 pontos)


O momento de inércia do sistema disco-esfera, em relação a um eixo vertical que passa
pelo pino e perpendicular ao disco, é dado por I0 = Idisco + Iesf era . Como o raio da esfera
r << R,
1 1
I0 = MR2 + mR2 ⇒ I0 = M + m R2

2 2

b) valor=(1,5 pontos)
Justificativa(0,5 pontos):
Não há torques na direção do eixo de rotação e também no ponto de contato os
torques internos não interferem no estado rotacional do sistema, portanto, a componente
do momento angular na direção correspondente ao eixo de rotação é constante. Como o
sistema estava inicialmente em repouso Liz = 0.
Imediatamente após receber o impulso a esfera adquire a velocidade ~v0. Em relação
ao centro do disco onde está o pino o momento angular da esfera é; para r << R,

Lz = RmvCM = Rmv0

Para o sistema disco-esfera L0z = I0 ω = 21 M + m R2 ω.
O momento angular total final é conservado na direção Z, logo, Lz + L0z = 0,

1 Rmv0
Lfz = Rmv0 + M + m R2 ω = 0

⇒ ω=− 1

2 2
M + m R2
mv0
ω=− 1

2
M + m R
Portanto o disco adquire um movimento de rotação contrário ao movimento da esfera,
girando no sentido horário de acordo com a figura.

3
Questão 4

a) valor = (1,0 pontos)


Durante todo o movimento na metade esquerda(com atrito), a esfera está sujeita às
forças: Normal(perpendicular ao movimento, i.e., não realiza trabalho), força de atrito
estática(não realiza trabalho já que não há deslizamento) e a força peso que é conservativa.
Portanto, a Energia mecânica é conservada. Utilizando a conservação entre o ponto de
onde é solta e o ponto onde ela passa pelo fundo da tigela, obtemos:(tomando o referencial
da energia potencial zero como o fundo da tigela e como a esfera estava inicialmente em
repouso)
1 1
mg(R − (R − r)cosθ) = mgr + mv 2 + ICM w2
2 2
2
Usando que ICM = mr2 e a condição de rolamento puro, v = ωr, obtemos que:
5

r
10 (R − r)(1 − cosθ)g
ω=
7 r2
b) valor= (0,5 pontos)
A energia cinética total é dada pela soma da energia cinética de rotação com a energia
cinética de translação, Ktot = KR + KT . Logo,

 KR = 21 ICM ω 2 = 51 mr2 ω 2 7
⇒ KR + KT = mr2 ω 2
10
KT = 12 mv 2 = 21 mr2ω 2

Portanto, 
 KR /Ktot = 15 mr2ω 2 / 10
7
mr2 ω 2 = 2/7

KT /Ktot = 12 mr2 ω 2 / 10
7
mr2ω 2 = 5/7

c) valor= (1,0 pontos)


Ao longo da parte sem atrito, a energia cinética rotacional da esfera não varia, já que não
há torques externos, i.e., a esfera continua girando em torno do seu centro de massa com
velocidade angular constante e igual ao resultado obtido no item a). Logo a conservação
da energia mecânica entre o ponto mais baixo da tigela e o ponto mais lato que ela atinge
na região sem atrito, onde o a velocidade de translação momentaneamente se anula é:
1 2 1 1
mv + mgr + ICM ω 2 = 0 + mg(R − (R − r)cosθ0 ) + ICM ω 2
2 2 2
e usando que, v = ωr , obtemos que:
5
cosθ0 = 1 − (1 − cosθ)
7

4
UNIVERSIDADE FEDERAL DO RIO DE JANEIRO

INSTITUTO DE FÍSICA

FÍSICA I – 2012/1

PROVA DE SEGUNDA CHAMADA – 13/07/2012

VERSÃO: A

INSTRUÇÕES: LEIA COM CUIDADO!

1. Preencha CORRETA, LEGÍVEL E TOTALMENTE os campos em branco do cabeçalho do caderno de resolução,


fornecido em separado.

2. A prova constitui-se de duas partes:

• uma parte objetiva, perfazendo um total de 5,0 pontos, constituı́da por dez (10) questões objetivas (de múltipla
escolha), cada uma das quais valendo 0,5 ponto, sem penalização por questão errada.
• uma parte discursiva, perfazendo um total de 5,0 pontos, constituı́da por duas (2) questões discursivas (ou argu-
mentativas ou dissertativas), cada uma das quais valendo 2,5 pontos.

3. Acima da tabela de respostas das questões objetivas, na primeira página do caderno de resolução, INDIQUE CLA-
RAMENTE A VERSÃO DA PROVA (A, B,. . . ).

4. O item considerado correto, em cada uma das questões objetivas, deve ser assinalado, A CANETA (de tinta azul ou
preta), na tabela de respostas correspondente do caderno de resolução

5. É vedado o uso de qualquer instrumento eletro-eletrônico (calculadora, celular, iPod, etc)

6. Seja organizado e claro.

Formulário

sen2 θ + cos2 θ = 1, sen2θ = 2senθcosθ


sen(α ± θ) = senαcosθ ± cosαsenθ, cos(α ± θ) = cosαcosθ ∓ senαsenθ
Z
d n xn+1
x = nxn−1 xn dx = (n 6= −1)
dx n+1
d d
senax = acosax, cosax = −asenax
dx dx

a b c
Lei dos senos: = =
senα senβ senγ
Lei dos cossenos: a2 = b2 + c2 − 2bc cosα

1
Seção 1. Múltipla escolha (10×0,5 = 5,0 pontos)

1. Dentro de um trem em movimento uma pessoa está pa- 4. Uma partı́cula move-se sobre uma circunferência de raio
rada e segurando uma maçã. Em um dado instante, no R, com velocidade de módulo constante v no plano hori-
momento que o trem percorre um trecho retilı́neo da zontal XY . O módulo da taxa de variação instantanea
linha férrea, ele atira a maçã verticalmente para cima. com o tempo do momento linear da partı́cula é direta-
I) Se o trem está com velocidade escalar constante, a mente proporcional a:
maçã cai ........ (atrás da, exatamente na, na frente) da
(a) R3
sua mão. II) Se o trem está acelerando para a frente, √
a maçã cai........(atrás da, extamente na, na frente da) (b) R
sua mão. As palavras que completam as frases I) e II) (c) R2
são respectivamente:
(d) v2
(a) atrás da/na frente (e) v
(b) exatamente na/exatamente na
(c) atrá da/atrás da
(d) na frente da/exatamente na
5. Duas massas m1 e m2 , diferentes, estão conectadas por
(e) exatamente na/atrás da uma mola de constante elástica k e de massa desprezı́vel.
Elas estão em repouso sobre uma superfı́cie horizontal
com a mola no seu estado relaxado. Num dado instante
sobre a massa m2 aplica-se uma força horizontal F~ de
2. Qual das afirmações para um corpo que realiza um mo- módulo constante, cuja direção passa pelos centros das
vimento circular uniforme no plano horizontal é cor- massas; como mostra a figura. Para a aceleração do
reta? centro de massa das massas, a opção correspondente a
(a) A força resultante F~ sobre o corpo e a ace- esta situação é:
leração ~a tem o mesmo sentido e direção.
(b) A força F~ resultante sobre o corpo é constante
no tempo.
(c) A força resultante F~ sobre o corpo está na
direção radial e o seu sentido é para fora da (a) ~aCM = F~ /m1 − F~ /m2
trajetória circular.
(b) ~aCM = F~ /m2
(d) A força resultante F~ é tangente a trajetória do
corpo. (c) ~aCM = F~ /(m1 + m2 )
(e) O módulo da força F~ resultante sobre o corpo (d) ~aCM = F~ /m1 + F~ /m2
não é constante. (e) nenhuma das respostas anteriores

3. Você é o projetista de uma montanha russa onde os car-


ros passam no trecho indicado na figura. No ponto A 6. Considere uma esfera que rola sem deslizar em linha reta
eles partem do repouso a uma altura h. Os carros pas- sobre um plano inclinado. Inicialmente (situação I), a
sam por um vale e depois sobem um novo monte, que esfera desloca-se para cima no plano. Algum tempo de-
tem uma curvatura de raio R e altura h′ . A altura h′ pois ela começa a rolar para baixo sem deslizar (situação
para que os passageiros estejam na iminência de perder II). Considere que as únicas forças que atuam sobre a
contato com os assentos ao passar pelo topo do monte esfera sejam o seu peso, a reação normal do plano sobre
é dada por(despreze o atrito): a esfera e a força de atrito. O diagrama que melhor
representa o sentido da força de atrito que age sobre a
esfera (f~at ) nas duas situações descritas acima é:

(a) h (a) 1
(b) h − R/2 (b) 2
(c) h−R (c) 3
(d) h − R/3 (d) 4
(e) (h + R)/2 (e) nenhum diagrama

2
7. No fim de seu ciclo de vida uma estrela exótica, em 9. Um bloco e um cilindro são lançados com a mesma ve-
rotação em torno de um eixo que passa pelo seu centro, locidade escalar v, ao longo de planos que tem a mesma
expele metade de sua massa e contrai-se numa esfera inclinação (diferente de zero), em relação à horizontal.
com apenas 10% do raio R da esfera original. Supondo No plano onde está o bloco não há atrito e no plano
que a massa e momento de inércia iniciais da estrela onde se encontra o cilindro há atrito suficiente para que
sejam dados respectivamente por M e I = (2/5)M R2 ele role sem deslizar. A relação entre a altura máxima
e que seu perı́odo inicial seja Ti , qual é o seu perı́odo que obloco atinge Hb e a altura máxima que o cilindro
final (isto é, depois da contração) Tf ? atinge HC nos respectivos planos é dada por:

(a) Tf = 0,005 Ti
(b) Tf = 0,01 Ti
(c) Tf = 0,02 Ti
(d) Tf = 0,1 Ti
(e) Tf = Ti (a) Hb = Hc , pois tanto a energia cinética do bloco
como a do cilindro se conserva.
(b) Hb = Hc , pois tanto a energia mecânica do
bloco como a do cilindro se conserva.
(c) Hb > Hc , pois o bloco desliza sem atrito.
(d) Hc > Hb , pois o momento angular do cilindro
se conserva.
(e) Hc > Hb , pois tanto a energia mecânica do
bloco como a do cilindro se conserva.
10. A figura mostra três partı́culas lançadas do mesmo nı́vel
e com a mesma velocidade. A primeira é lançada ver-
ticalmente, a segunda fazendo um ângulo θ com a hori-
8. Dois cabos de massas desprezı́veis AC e BC estão presos zontal e a terceira é lançada sobre um plano inclinado
ao teto e sustentam no ponto C um corpo de peso P sem atrito. Em ordem crescente de velocidade na altura
mantendo-o equilibrado. Os ângulos α e β correspon- correspondente à linha tracejada tem-se:
dem àqueles que AC e BC fazem com a horizontal. Se
T1 e T2 são os módulos das trações nos cabos AC e BC
respetivamente a opção correta é:

(a) T1 /P = cos(α + β) (a) 1, 2 e 3


(b) T1 /P = cos β/sen α (b) 3, 2 e 1
(c) T1 /P = cos β/cos(α + β) (c) todas chegam com mesma velocidade
(d) T1 /P = cos β/sen(α + β) (d) 3, 1 e 2
(e) T1 /P = cos(β)/cos(α) (e) 2, 3 e 1

3
Seção 2. Questões discursivas (2×2,5 = 5,0 pontos)

1. Um bloco de massa m1 , desliza sobre uma superfı́cie rugosa, existindo um coeficiente de atrito cinético µc entre ambas
as superfı́cies. Uma bola de massa m2 está conectada ao bloco através de uma corda ideal que passa por uma polia
também ideal. Uma força de módulo F , age sobre o bloco, com uma direção que faz um ângulo θ em relação a
horizontal, arrasta o sistema bloco-bola para a direita. Em relação aos dados do problema e considerando que g é
módulo da aceleração local da gravidade:
a) faça um diagrama de corpo livre para ambas as massas, representando todas as forças que atuam em cada uma
delas;
b) calcule a força normal que a superfı́cie horizontal exerce sobre o bloco m1 ;
c) calcule a aceleração das massas
d) calcule a força de tração na corda que liga as duas massas

2.
Em um cilindro de massa M , raio R são colocados dois aros finos de raio r e de massas desprezı́veis sobre o seu topo e
base de tal forma que eles fiquem concêntricos e rigidamente fixados(não deslizam). O sistema aros-cilindro é colocado
sobre um trilho horizontal, mantendo-se contato com os aros. A seguir o sistema é puxado por um cabo ideal, passando
pelo seu centro, com uma tracão T~ de módulo constante e horizontal; como mostra a figura. O sistema parte do repouso
e verifica-se que ele rola sem deslizar. Considere que o momento de inércia formado pela aros e pelo cilindro é igual
ICM = (1/2)M R2 , segundo um eixo que passa longitudinalmente pelo seu centro de massa e perpendicular a sua seção
reta. Justificando as respostas, e de acordo com os vetores unitários indicados na figura; e considerando que g é módulo
da aceleração local da gravidade:
a) faça um diagrama de forças representando todas as forças que atuam sobre o sistema aros-cilindro idenficando-as;
b) escreva as equações do movimento para translação e rotação do sistema;
c) calcule o módulo da aceleração angular do sistema;
d) após o centro de massa percorrer uma distância d, sob a ação da força T~ , calcule a energia cinética do sistema.

4
Universidade Federal do Rio de Janeiro
Centro de Ciências Matemáticas e da Natureza
Instituto de Fı́sica
Segunda Chamada de Fı́sica IA - 04/07/2012
Respostas para provas hı́bridas

Gabarito das Questões objetivas

Versão A Versão B
Questão (a) (b) (c) (d) (e) Questão (a) (b) (c) (d) (e)
1 1
2 2
3 3
4 4
5 5
6 6
7 7
8 8
9 9
10 10

Versão C Versão D
Questão (a) (b) (c) (d) (e) Questão (a) (b) (c) (d) (e)
1 1
2 2
3 3
4 4
5 5
6 6
7 7
8 8
9 9
10 10

1
Questão discursiva 1
a) valor=0.5 ponto

As forças que agem na bola são: o peso P~2 e a tração T~ 0. No bloco temos: o peso P~1 , a
tração T~ , a força de atrito f~at , a reação da superfı́cie sobre o bloco N
~ e a força aplicada
F~ . Como o cabo que liga as duas massas é ideal |T~ | = |T~ |. 0

b) valor=0.5 ponto
De acordo com o sistema de eixos e pela Segunda Lei de Newton, na direção Z não há
aceleração, logo
N − m1g + F sen θ = 0 → N = m1 g − F sen θ

c) valor=1.0 ponto
O bloco(m1 ) e a bola(m2 ) tem movimento na direção X e na direção Z respectiva-
mente, e lembrando que |T~ | = |T~ 0| = T , logo:
 
 bola :T 0 − m2g = m2a  T − m2 g = m2 a

bloco : F cosθ − T − fat = m1 a F cosθ − T − fat = m1 a
 

A partir do resultado do item b) onde N = m1g − F sen θ e na condição de que


~ ~ a solução do sistema de equações nos fornece:
|fat| = µc |N|,
1
a= [F (cos θ + µc sin θ) − g (µc m1 + m2 )]
m1 + m2

d) valor=0.5 ponto
Para obter a tração T , podemos usar do sistema de equações a primeira linha, onde
T − m2 g = m2 a. Substituindo o valor de a obtido anteriormente, temos:
m2
T = m2 g + [F (cos θ + µc sin θ) − g (µc m1 + m2)]
m1 + m2
m2
T = [F (cos θ + µc sin θ) + m1g (1 − µc )]
m1 + m2

2
Questão discursiva 2

a) valor=0.5 ponto

De acordo com os vetores unitários indicados na figura temos: A força T~ = Tı̂ cor-
respondente à tração na corda, o peso P~ = −Mg̂, ambas agindo no centro de massa, a
força de atrito age nos dois pontos de contato com as aplicações que representamos por
um único ponto A e é dada por f~at = −fat ı̂ e, finalmente, agindo no ponto de contato A,
~ = N ̂.
a reação da superfı́cie aplicada no cilindro N

b) valor=0.7 ponto
A equação do movimento para a translação do centro de massa do sistema e rotação
em torno do centro de massa são dadas por:

translação: M ~aCM = T~ + f~at + M ~g + N


~

rotação: Iα
~ = ~τN + ~τP + ~τfat + ~τT
Para a translação do centro de massa segundo as direções de ı̂ e ̂:

MaX = T − fat (1)

MaY = N − Mg (2)
Para a rotação segundo os torques calculados em relação a o e considerando o sentido
anti-horário de rotação como o sentido positivo:

I(−α) = −rfat

Além das equações acima temos equações adicionais que correspondem aos vı́nculos
que dizem:
i) O contato tem que ser matido de modo que aY = 0. A equação (2) fornece esta
condição, ou seja, N = Mg
−→
ii) A velocidade no ponto de contato tem que ser nula: ~vA = ~vT −O + ~ω × OA; ~vT −O é a
velocidade de translação do centro de massa.
Este último resultado está de acordo com a condição de que o sistema rola sem deslizar e
nos dá: |~vT −O | = ωr. O que implica em aCM = aX = α r

3
c) valor=0.8 ponto
Com as condições descritas anteriormente obtemos o sistema de equações:

  
 T − fat = MaX  T − fat = Mαr  T − fat = Mαr
→ →
rfat = Iα rfat = Iα rfat = 12 MR2 α
  

A resolução deste sistema de equações nos fornece para a aceleração angular:


T r
α=
M (r + R2 /2)
2

d) valor=0.5 ponto
Podemos usar o Teorema Trabalho-Energia, para calcular a energia cinética após o
centro de massa percorrer a distância d.

∆K = Wtotal
~ e P~ são perpen-
Como a força de atrito estático não fornece trabalho, e as forças N
diculares à trajetória do centro de massa, e Ki = 0,

K = WT ⇒ K = Td

OBSERVAÇÃO : seria preciso calcular também fat e verificar que |f~at| ≤ µE N !! .


Qual seria então a condição para a tração T para que o sistema role sem deslizar?

4
4. Um disco de raio R e de massa M uniformemente 6. Em um sistema três partı́culas de massas iguais a
UNIVERSIDADE FEDERAL DO RIO DE JANEIRO distribuı́da, possui um pequeno furo na sua periferia. m movem-se na ausência de forças externas com ve-
INSTITUTO DE FÍSICA Por este furo o disco é colocado no pino O, preso a locidades constantes ~v1 = vı̂, ~v2 = −vı̂ e ~v3 = vı̂.
uma parede vertical. No posicionamento inicial do Num dado instante elas ocupam as posições indi-
FÍSICA I – 2012/2 disco o ponto A de sua periferia localizado diametral- cadas na figura abaixo. O vetor momento angu-
PROVA de Segunda Chamada - 13/03/2013 mente oposto a O está sobre o eixo vertical, acima de ~ O deste sistema de partı́culas em relação
lar total L
O; conforme mostra a figura abaixo. Como o disco a origem O do eixo de coordenadas da figura é:
VERSÃO: A está sob a condição de equilı́brio instável, ele cai gi-
rando em torno do pino O. Sabe-se que o momento (a) ~0.
de inércia ICM deste disco em torno de um eixo que
passa pelo centro de massa e perpendicular e ele é (b) −2avmk̂.
Nas questões em que for necessário, considere que g é o módulo da aceleração da gravidade.
igual (1/2)M R2 . Desprezando-se o atrito entre o pino (c) avmk̂.
e o disco, quando o ponto A passa pela linha vertical, (d) −avmk̂
Seção 1. Múltipla escolha (10×0,5 = 5,0 pontos) abaixo de O, a velocidade angular ω do disco, é igual
(e) −3avmk̂.

1. Um pêndulo balı́stico, é constituı́do de um bloco de 2. Uma esfera maciça tem massa M , raio R e momento
massa M e de cabos ideais presos ao teto, como mos- de inércia ICM segundo um eixo que passa pelo seu
tra a figura. Uma bala de massa m deslocando-se centro. Ela está girando, com velocidade angular ω0 ,
p
(a) 2 g/R
horizontalmente, colide com o pêndulo. Após a co- e é colocada suavemente, com velocidade do seu cen- p
lisão ela penetra no bloco e permanece presa a ele. O tro de massa nula, sobre uma superfı́cie plana e ho- (b) g/2R
p
centro de massa do sistema bala-pêndulo desloca-se rizontal, com o seu eixo de rotação posicionado para- (c) 2g/R
atingindo a altura máxima h em relação a sua altura lelamente ao plano horizontal. Inicialmente ela rola p 7. Duas máquinas de Atwood A e B são contituı́das de
(d) 3g/2R uma polia e dois blocos de massas m1 e m2 , m1 6= m2 ,
inicial. Desprezando-se o efeito do ar, o módulo da ve- e desliza sobre o plano e após o intervalo de tempo p ligados por um fio ideal e que passa pela sua periferia.
locidade da bala, imediatamente antes da colisão com ∆t, reduz a sua velocidade angular, passando a rolar a:(e) 2 2g/3R
o pêndulo é igual a: sem deslizar com velocidade angular ωf constante . O A polia da máquina A é ideal, enquanto que a polia
módulo do torque resultante, considerado constante, da máquina B tem massa M diferente de zero. Os
que atuou sobre a esfera, no intervalo de tempo ∆t é fios que ligam os blocos não deslizam sobre as polias.
igual a: Quando os blocos estão em movimento, as polias gi-
ω0 −ωf
ram em torno do eixo que passa pelos seus centros sem
(a) ICM ∆t
; atrito. No diagrama abaixo, estão representadas so-
ω0 mente as trações que atuam sobre as massas em cada
(b) ICM ∆t ;
máquina. A relação correta entre as trações de cada
(c) 0;
máquina corresponde a:
(d) RM g
(M +m) √ (e) Nenhuma das respostas anteriores.
(a) m
2gh
M

(b) m
gh
√ 3. Duas pequenas esferas de massas m e M , M 6= m, 5. A figura mostra uma placa metálica quadrada de lado
m
(c) 2gh colidem frontalmente na ausência de forças externas.
(M +m) ℓ e de massa M uniformemente distribuı́da, onde foi
(M +m) √ Após a colisão elas movimenta-se juntas. Durante a removido um pedaço correspondente a um quarto
(d) gh
m
√ colisão, atuam em cada esfera as forças médias F~M de sua área. A relação entre os valores dos mo-
m
(e) 2gh e F~m nas esferas de massa M e m respectivamente.
M mentos de inércia Ia , Ib e Ic , desta placa, calcula-
Qual é a relação entre as forças médias que atuaram dos segundo um eixo perpendicular a ela passando
nas esferas durante a colisão. pelos pontos a, b e c, respectivamente é dada por:
(a) |F~M |/|F~m | = m/M .
(a) Ia > Ib > Ic ; (a) |T~1 | = |T~2 | e |T~3 | = |T~4 |;
(b) |F~M |/|F~m | = M/m;
(b) Ia = Ic > Ib ; (b) |T~1 | = |T~2 | e |T~3 | =
6 |T~4 |;
(c) F~M + F~m = ~0;
(c) Ia < Ib < Ic ; (c) |T~1 | =
6 |T~2 | e |T~3 | = |T~4 |;
(d) F~M − F~m = ~0;
(e) Como o tempo de colisão não é conhecido, não (d) Ib < Ia < Ic ; (d) |T~1 | =
6 |T~2 | e |T~3 | =
6 |T~4 |;
é possı́vel obter uma relação entre as forças. (e) Ia = Ic < Ib . (e) Nenhuma das respostas anteriores.

1 2
8. Em uma loja duas escadas rolantes estão dispostas 9. Uma única força atua sobre uma partı́cula em mo- Seção 2. Questões discursivas (2×2,5 = 5,0 pontos)
como mostra a figura abaixo. O ângulo de elevação θ vimento retilı́neo. O gráfico da velocidade desta
de cada escada, em relação a horizontal é o mesmo. As partı́cula em função do tempo é mostrado na figura
1. Uma pedra de massa m está apoiada, sem estar presa, sobre uma mola ideal alinhada verticalmente, como mostra
escadas movimentam os degraus com velocidades de abaixo. Considerando os quatro intervalos de tempo
a figura abaixo. A pedra está em repouso e a mola encontra-se comprimida de ∆ℓ1 . A constante elástica da mola
mesmo módulo v, medidas em relação a Terra. Num tA→B , tB→C , tC→D e tD→E , respectivamente nesta or-
não é conhecida. Nas respostas das questões abaixo, expresse os seus resultados em função dos dados fornecidos.
dado instante duas pessoas A e B entram simultane- dem, a opção que diz se o trabalho realizado pela força
amente nas escadas e ao entrarem permanecem para- sobre a partı́cula é positivo (+), negativo (-) ou nulo a) Qual é a constante elástica da mola?
das em relação aos degraus das respectivas escadas. (0) é: b) A pedra é empurrada de ∆ℓ2 a mais para baixo e liberada em seguida (com velocidade nula). Qual é a energia
A pessoa A entra na escada que sobe e a pessoa B na potencial elástica armazenada pela mola imediatamente antes da pedra ser liberada?
escada que desce; vide a figura abaixo. Considerando c) Qual é o trabalho da força peso, desde a posição em que a pedra foi liberada, situação do item (b), até a altura
que a Terra é um referencial inercial, o módulo da ve- máxima que ela atinge?
locidade da pessoa A em relação a pessoa B, enquanto d) Qual é a altura máxima h, medida em relação a posição mais baixa correspondente a situação do item (b), que
elas estão nas escadas é: a pedra atinge?

(a) +, 0, +, -
(b) +, 0, +, +
(a) |~vA/B | = 0;
(c) +, +, +, -
(b) |~vA/B | = vcos θ;
(d) +, +, -, -
(c) |~vA/B | não é contante;
(e) +, 0, -, +
(d) |~vA/B | = 2vsen θ
2. Um cilindro maciço de raio R, de massa M homogêneo e de comprimento L encontra-se em repouso sobre um
(e) |~vA/B | = vsen 2θ 10. Dois elos de correntes de massas iguais a m1 e m2 plano horizontal onde há atrito. Num dado instante aplica-se uma força F~ horizontal no cilindro, constante e
e entrelaçados, são puxados verticalmente para cima paralela ao plano horizontal. A direção desta força passa pelo centro do cilindro; a figura mostra um corte lateral
por uma força F~ constante. Verifica-se que eles so- correspondente à situação da força em questão. Imediatamente após a aplicação da força F~ o cilindro rola sem
bem com velocidade constante. A expressão correta deslizar. Dados: o momento de inércia ICM , de um cilindro maciço e homogêneo de massa M, de raio R, para um
da dinâmica do movimento dos elos é: eixo que passa pelo seu centro de massa longitudinalmente ao cilindro é igual a (1/2)MR2 e o coeficiente µe de
(a) F~ − (m1 + m2 )~g = ~0; atrito estático entre as superfı́cies do plano e do cilindro.
(b) F~ + (m1 − m2 )~g = ~0; a) Em um diagrama represente todas as forças que atuam no cilindro após a aplicação da força F~ . Use como
orientação os vetores unitários indicados na figura.
(c) F~ + (m1 + m2 )~g = ~0;
b) Determine o vetor força de atrito que atua sobre o cilindro (módulo, direção e sentido), de acordo com o sistema
(d) F~ + (m2 − m1 )~g = ~0; de eixos orientados da figura.
(e) nenhuma das respostas anteriores. c) Qual é o valor máximo do módulo da força F~ , para que o cilindro ainda role sem deslizar?

3 4
Universidade Federal do Rio de Janeiro
Centro de Ciências Matemáticas e da Natureza
Instituto de Fı́sica
Prova de Segunda Chamada de Fı́sica IA - 13/03/2013
Respostas para provas hı́bridas

Gabarito das Questões objetivas - 0,5 ponto cada questão

Versão A Versão B
Questão (a) (b) (c) (d) (e) Questão (a) (b) (c) (d) (e)
1 1
2 2
3 3
4 4
5 5
6 6
7 7
8 8
9 9
10 10

Versão C Versão D
Questão (a) (b) (c) (d) (e) Questão (a) (b) (c) (d) (e)
1 1
2 2
3 3
4 4
5 5
6 6
7 7
8 8
9 9
10 10

1
Questão discursiva 1) - 2,5 pontos
a) valor=0,5 ponto
A pedra está em repouso, satisfazendo a condição de equilı́brio, P~ + F~m = 0. Na direção
vertical temos, P~ = mg k̂ e Fm = −k∆`1 k̂, logo:
mg
k=
∆`1

b) valor=0,5 ponto
A pedra ao ser empurrada para baixo passa a comprimir a mola de ∆`1 + ∆`2 . A energia
1
armazenada pela mola será ∆Um = k∆`2 . Como ∆` = ∆`1 + ∆`2 e com o valor de k
2
obtido no item anterior,
1 mg
Um = (∆`1 + ∆`2)2
2 ∆`1

c) valor=0,5 ponto
Para calcular o trabalho da força peso podemos usar o Teorema Trabalho-Energia, ∆K =
W C + W N C , onde W C é trabalho total de todas a s forças conservativas presentes e W N C
o trabalho total de todas as forças não conservativas. Como as forças que atuam sobre
a pedra são conservativas temos que a variação da energia cinética é nula, pois na altura
máxima a pedra tem momentaneamente velocidade nula, obtemos:

0 = W C = WP + Wmola
f i i
O trabalho da mola é igual a -∆Um = −(Um − Um ), como Um corresponde a energia
f
armazenada pela mola, item (b), e considerando Um = 0 a energia potencial da mola
quando ela está relaxada, logo:

i 1 mg
WP = −Um =− (∆`1 + ∆`2 )2
2 ∆`1
d) valor=1,0 ponto
Para obter a altura máxima basta calcular o trabalho da força peso, WP = P~ · ∆S;~ ∆S
~ é
o vetor deslocamento da pedra desde o inı́cio até atingir a altura máxima. Logo,
1 mg
−P ∆S = − (∆`1 + ∆`2 )2
2 ∆`1
Como h = ∆S,

1 (∆`1 + ∆`2 )2
h=
2 ∆`1

2
Questão discursiva 2) - 2,5 pontos
a) valor=0,5 ponto

b) valor=1,0 ponto
Para obter o vetor força de atrito primeiramente calcularemos o seu valor. A dinâmica
do movimento do cilindro é dada de acordo com os modos de translação e rotação:

F~iext = M~aCM
 P

~τ ext = I α
 P
~
Assim segundo a direção do movimento do cilindro, para a translação e rotação e os
eixos orientados temos:

 F − fat = MaCM translação

~τ = −Iαk̂ rotação

O cilindro rola sem deslizar, cuja condição é dada pelo vı́nculo aCM = αR e o torque
resultante em relação ao centro do cilindro é dado por ~τ = −R̂ × fatı̂. Portanto obtemos
~τ = −fat Rk̂. Substituindo estes resultados no sistema anterior, onde I = ICM :

 F − fat = MaCM

Rfat = ICM aCM /R


ICM F
A resolução do sistema nos dá fat = . Como ICM = (1/2)MR2 ,
(M + ICM /R )R
2 2
temos para o valor de fat,
F
fat =
3
O sinal deste valor é positivo, indicando que a orientação da força de atrito no diagrama
de forças está correta. Assim o vetor força de atrito é:
F
f~at = − ı̂
3

c) valor=1,0 ponto
Para obtermos o valor de máximo da força F , o valor da força de atrito deverá ser máximo.
Neste caso fmax = µe Mg. Portanto, com o resultado do item anterior:
Fmax = 3µe Mg

3
4. Uma partı́cula está sob a influência de um poten- 6. Livre de forças externas, dois blocos colidem frontal-
UNIVERSIDADE FEDERAL DO RIO DE JANEIRO cial unidimensional mostrado na figura. A afirmação mente. O primeiro bloco tem massa m1 = m e veloci-
INSTITUTO DE FÍSICA incorreta que corresponde à partı́cula com energia dade ~v e o segundo bloco massa m2 = 2m e velocidade
mecânica E e na porção do gráfico exibida é: −~v . Após a colisão eles ficam unidos. Se P~ é o mo-
FÍSICA I – 2013/1 mento linear total dos blocos e ~vCM é a velocidade do
PROVA DE SEGUNDA CHAMADA – 07/08/2013 centro de massa dos blocos, a opção correta é:

VERSÃO: A (a) O momento linear P~ não é conservado pois a


colisão é inelástica.
(b) |~vCM | = 0 antes da colisão
Nas questões em que for necessário, considere que g é o módulo da aceleração da gravidade. (c) |~vCM | = 0 depois da colisão, pois eles andam
unidos depois da colisão.
(d) ~vCM = −(1/3)~v , antes e depois da colisão.
Seção 1. Múltipla escolha (10×0,5 = 5,0 pontos)
(e) Nenhuma das respostas anteriores.
(a) a posição xa é um ponto de retorno.
1. Um arame homogêneo e uniforme, de espessura des- 3. Um disco que gira com velocidade angular ω0 é freado
prezı́vel, é moldado na forma de um quadrado de uniformemente até atingir o repouso. Durante a frena- (b) a energia cinética em xb é máxima.
aresta a. Retira-se então um segmento de compri- gem ele executa uma revolução. Outro disco, idêntico (c) a energia cinética em xc é nula.
mento a, reduzindo duas arestas à metade, como mos- ao primeiro, tem velocidade angular ω1 = 10ω0 é (d) a força na posição xb originada deste potencial
tra a figura. No novo sistema a distância do centro de freado com mesma desacelerção angular que o pri- é nula.
massa à origem é igual a, meiro. Quantas revoluções ele executa até atingir o
repouso? (e) ela pode movimentar-se entre xa e xd .

3 √ (a) 250
(a) a 2 5. Um fio de massa desprezı́vel e de comprimento d é
7 (b) 150
4 √ preso ao teto num ponto fixo. Na sua extremidade,
(b) a 2 (c) 100 uma partı́cula de massa m gira em torno do eixo Z
9
3 √ (d) 50 vertical com velocidade de módulo v constante des-
(c) a 2 (e) 200 crevendo um cı́rculo horizontal de modo que o fio des-
8
creva um cone, onde α é o ângulo constante que o fio
1 √
(d) a 2 faz com o eixo Z; veja a figura. A componente ℓZ do
4
momento angular dessa partı́cula, referido ao ponto
1 √
(e) a 2 O, na direção OZ é: 7. Duas rodas A e B estão em rotação conectadas por
3 uma correia que passa por suas periferias e não des-
liza sobre elas. Sejam RA e RB os raios das rodas A
e B respectivamente. Qual é a razão entre os seus
momentos de inércia IA /IB , se ambas têm a mesma
energia cinética?

2. Uma partı́cula move-se no plano horizontal sem atrito


com movimento circular e uniforme. O produto esca-
lar entre o vetor velocidade ~v (t) da partı́cula e o seu
vetor posição ~r(t), relativo ao centro da trajetória,
para um instante qualquer t é:
(a) nulo; (a) ℓZ = mvd sen α (a) RA /RB
(b) π/4; (b) ℓZ = mvdcos α (b) RB /RA
(c) π/2; (c) ℓZ = mvd (c) 1
2 2
(d) π; (d) ℓZ = mvd tan α (d) RA /RB
2 2
(e) nenhuma das respostas anteriores. (e) ℓZ = 2πdmv sen α (e) RB /RA

1 2
8. Duas partı́culas A e B movem-se no plano XOY , res- 10. Um disco A de raio R e de massa M rola sem deslizar Seção 2. Questões discursivas (2×2,5 = 5,0 pontos)
pectivamente com velocidades ~vA = −vA ı̂ e ~vB = vB ̂ sobre uma superfı́cie horizontal com velocidade de seu
constantes. Sendo ~vAB a velocidade de A com relação centro de massa igual a V~ . Um outro disco B de raio
1. Dois blocos 1 e 2 e de massas m1 e m2 , descem um plano inclinado de um ângulo θ em relação a horizontal. Os
a B a opção correta é: 2R e de mesma massa do disco A, também rola sem
blocos estão ligados por um cabo de massa desprezı́vel e inextensı́vel, como mostra a figura (o cabo permanece
deslizar sobre a mesma superfı́cie com a velocidade do
paralelo ao declive do plano inclinado). O coeficiente de atrito cinético entre as superfı́cies dos blocos 1 e 2 e do
seu centro de massa, igual a velocidade do centro de
plano inclinado são µ1 e µ2 respectivamente. Considere que as massas dos blocos, o ângulo θ e os coeficientes de
massa do disco A. Sabendo que o momento de inércia
atrito são tais que o cabo permanece tenso e que os blocos têm a mesma aceleração, no sentido de descida do plano
de um disco de massa M e de raio R segundo o eixo
inclinado.
que passa pelo seu centro de massa e perpendicular
ao plano que o contém é IC M = M R2 /2. A opção a) Faça um diagrama para cada bloco, representando todas as forças que atuam em cada um deles;
correta é: b) Calcule o módulo da aceleração com que os blocos se movimentam;
c) Calcule o módulo da força de tração que atua no cabo.
(a) Os dois discos possuem o mesmo momento an-
gular total;
(a) ~vAB = ~0.
p (b) Os dois discos possuem a mesma energia
(b) |~vAB | = vA2 + vB2 . cinética total;
(c) ~vAB = vA ı̂ − vB ̂. (c) Os dois discos têm a mesma velocidade angu-
p
(d) |~vAB | = vA2 − vB2 . lar;
(e) todas as opções acima estão erradas. (d) A energia cinética de rotação do disco A é me-
nor que a energia cinética de rotação do disco
9. Uma bola de borracha de massa m liberada a par-
B;
tir do repouso e cai verticalmente de uma altura 8h.
Ela rebate verticalmente sobre uma superfı́cie plana (e) Nenhuma das respostas anteriores.
e atinge, após ser rebatida, uma altura máxima igual
a 1/4 da altura inicial. Durante a colisão com a su-
perfı́cie decorre um intervalo de tempo ∆t. O módulo
força média Fm , que atuou sobre a bola durante este
intervalo de tempo é:
4m p 2. Um carretel é formado por um cilindro homogêneo de raio r e dois discos homogêneos idênticos de raio R, de
(a) gh ; modo que o eixo do cilindro seja o eixo de simentria do carretel. O carretel encontra-se sobre um trilho horizontal
∆t
m p e é puxado por um cabo de massa desprezı́vel com uma força horizontal de módulo constante F . O cabo está
(b) 6gh enrolado na periferia do carretel como mostra a figura. O carretel ao ser puxado rola sem deslizar sobre o trilho.
∆t
m p Considerando que o momento de inércia do carretel em relação a um eixo que passa longitudinalmente pelo seu
(c) 4gh
∆t centro de massa é ICM = M R2 , calcule:
8m p a) o módulo da aceleração do centro de massa do carretel;
(d) gh
∆t b) o módulo da força de atrito que atua sobre o carretel.
6m p c) Sabendo-se que o coeficiente de atrito estático entre a superfı́cie do trilho e do carretel é igual a µ, determine a
(e) gh
∆t intensidade máxima da força F~ horizontal com que o carretel pode ser puxado sem deslizar.

3 4
Universidade Federal do Rio de Janeiro
Centro de Ciências Matemáticas e da Natureza
Instituto de Fı́sica
Prova de Segunda Chamada de Fı́sica IA - 07/08/2013
Respostas para provas hı́bridas

Gabarito das Questões objetivas (valor=5,0 pontos)

Versão A Versão B
Questão (a) (b) (c) (d) (e) Questão (a) (b) (c) (d) (e)
1 1
2 2
3 3
4 4
5 5
6 6
7 7
8 8
9 9
10 10

Versão C Versão D
Questão (a) (b) (c) (d) (e) Questão (a) (b) (c) (d) (e)
1 1
2 2
3 3
4 4
5 5
6 6
7 7
8 8
9 9
10 10

1
Questão discursiva 1 (valor=2,5 pontos)
a) valor=0,5 ponto
O diagrama de forças é dado pela figura:

b) valor=1,5 pontos
Aplicando a segunda Lei de Newton para cada bloco temos:
 
 OX : T − fat1 + P1 senθ = m1 a  OX : −T − fat2 + P2 senθ = m2 a
Bloco 1 Bloco 2
OY : N1 − P1 cos θ = 0 OY : N2 − P2 cos θ = 0
 

Como o movimento efetivo se dá ao longo do plano inclinado na direção OX um novo


sistema de eguações pode ser estruturado:

 
 T − fat1 + P1 senθ = m1 a  T − m1a = fat1 − P1 senθ

−T − fat2 + P2 senθ = m2a −T − m2a = fat2 − P2 senθ
 


 T − m1a = µ1 P1 cos θ − P1 senθ

−T − m2a = µ2 P2 cos θ − P2 senθ


A resolução do sistema acima tendo como incógnitas a tração e a aceleração nos dá:

(µ1 m1 + µ2 m2)
a = gsenθ − g cos θ,
(m1 + m2)
após a substituição de P1 = m1 g e P2 = m2g.

c) valor=0,5 ponto
Para calcular o valor da tração T basta resolver o sistema anterior para T .
m1 m2
T = (µ1 − µ2 ) g cos θ
(m1 + m2)

2
Questão discursiva 1 (valor=2,5 pontos)
a) valor=1,0 ponto
O carretel desloca-se com o movimento dado pela dinâmica de rotação e de translação.

F~ ext = M~aCM
P
translação :

~τ ext = I α
P
rotação : ~

Adotando a translação para a direita como o sentido positivo do movimento do centro de


massa, a rotação em torno do eixo longitudinal do carretel no sentido anti-horário como positiva,
os torques calculados em relação centro de massa e que o carretel não desliza sobre o trilho
temos:

 F − fat = MaCM 
 F − fat = MaCM (i)



 
−F R − fatr = −Iα →

  f + R F = Ia /r2

(ii)
 at CM


aCM = αr r

A solução do sistema de equações da direita permite obter a aceleração do centro de massa,

F (1 + R/r) 2 F  r+R 
aCM = como I = MR ∴ aCM = r
(I/R2 + M) M r2 + R2

b) valor=0,5 ponto
O valor da força de atrito é obtido usando, por exemplo, a equação (i) do sistema de equações,
substituindo o valor da aCM obtido do item anterior. Logo,
   
F r+R r+R
F − fat = M =F −F
M r2 + R2 r2 + R2

R(R − r)
∴ fat = F > 0 ! R > r (o sentido da força de atrito usado na figura está correto)
(r2 + R2 )

c) valor=1,0 ponto
Para Fmax , na condição de limite de rolamento sem deslizamento, o atrito será máximo, ou
fmax = µe N. Como |N|~ = Mg e fat obtida do item anterior,
 R(R − r)   r2 + R2 
µe Mg = Fmax → Fmax = µe Mg
r2 + R2 R(R − r)

3
4. Dois cabos de massas desprezı́veis AC e BC estão pre- 7. Uma partı́cula move-se no plano horizontal com atrito
UNIVERSIDADE FEDERAL DO RIO DE JANEIRO
sos ao teto e a uma parede vertical respectivamente. No com movimento circular de raio R com velocidade de-
INSTITUTO DE FÍSICA ponto C um corpo de peso P é mantido equlibrado. O crescente em um intervalo de tempo ∆t, no sentido anti-
FÍSICA I – 2013/2 ângulo que o cabo BC faz com a parede vertical é igual horário. O ângulo φ entre o vetor aceleração radial da
a π/2 e o ângulo que AC faz com o teto θ 6= 0. Se T1 é partı́cula, ~ar (t) (aceleração centrı́peta), e o vetor ace-
PROVA de SEGUNDA CHAMADA – 18/12/2013 o módulo da tração no cabo BC a opção correta é leração tangencial da partı́cula, ~aT (t), neste intervalo de
VERSÃO: A tempo é
(a) nulo;
(a) T1 /P = cos θ;
Nas questões em que for necessário, considere que g é o módulo da aceleração da gravidade. (b) 0 < φ < π/2;
(b) T1 /P = sec θ;
(c) π/2 < φ < π;
(c) T1 /P = sen θ;
(d) φ > π.
Seção 1. Múltipla escolha (10×0,5 = 5,0 pontos) (d) T1 /P = tan θ;
(e) φ = π/2;
(e) T1 /P = cot θ.
1. Duas partı́culas 1 e 2 de mesma massa movem-se em 2. Uma partı́cula descreve um movimento circular, com ve-
cı́rculos concêntricos de raios r1 e r2 , contidos em um locidade de módulo constante e igual a V . Num inter-
plano A e na ausência de torques externos, como mostra valo de tempo em que percorre 3/4 da circunferência, o
a figura. A soma de seus momentos angulares em relação módulo de seu vetor velocidade média é igual a 5. Duas bolas A e B idênticas são liberadas da mesma
a origem O do eixo Z perpendicular ao plano A é nula. √
2 altura h e caem verticalmente sobre uma superfı́cie ho-
Se I1 e I2 são os momentos de inércia das partı́culas re- (a) V; rizontal. A bola A colide elasticamente com a superfı́cie
π
lativos ao eixo de rotação OZ e ω1 e ω2 são os módulos √ enquanto a bola B ao colidir permanece na superfı́cie.
das velocidades angulares em torno do eixo OZ é correta 2 2
(b) V; A duração do tempo de colisão de ambas as bolas com
a relação π a superfı́cie é o mesmo e FA e FB são as intensidades

2 2 das forças médias que atuaram sobre as bolas A e B
(c) V;
3π respectivamente. Podemos afirmar que

(d) V;
4
√ (a) FA = FB ;
2
(e) V. (b) FA < FB ;

(c) FA 6= 0 e FB =0;
3. Três projéteis são lançados de um ponto O de um plano (d) FA = 2FB ;
horizontal com velocidades iguais de módulo v0 . O
projétil A é lançado de um ângulo π/4 + δ, o projétil (e) FA = FB /2. 8. Um haltere é constituı́do de duas massas 1 e 2 iguais a m
(a) I1 /I2 = ω1 /ω2 ; B do ângulo π/4 − δ e o projétil C do ângulo π/4, onde ligadas por uma barra fina e rı́gida de massa desprezı́vel
(b) I1 /I2 = ω2 /ω1 . 0 < δ < π/4. As trajetórias destes projéteis estão in- e comprimento ℓ. Ele é colocado horizontalmente em um
dicadas na figura por A, B e C respectivamente. As apoio na posição O com as distâncias relativas às massas
(c) I1 /I2 = (ω1 /ω2 )2 ; dadas por d1 e d2 (d1 + d2 = ℓ). Imediatamente após ser
trajetórias ilustradas corretamente são 6. Três barras de massas m, 2m e 3m formam um arranjo
(d) I1 /I2 = (ω2 /ω1 )2 ; triangular. As barras de massa √ m e 2m têm compri- liberado a partir do repouso as massas 1 e 2 adquirem
(e)
p
I1 /I2 = ω1 /ω2 ; mento a e a outra comprimento a 2. Para o sistema de acelerações de módulos a1 e a2 e acelerações angulares
coordenadas, cuja origem coincide com um dos vértices α1 e α2 respectivamente. A opção correta corresponde
das barras, as coordenadas do centro de massa do a,
arranjo xcm e ycm são respectivamente

√ √
(a) A, B e C; (a) ( 2/3)a e ( 2/5)a; (a) a 1 = a2 e α 1 = α 2 ;
(b) somente A e B; (b) (2/3)a e (2/3)a; (b) a 1 > a2 e α 1 > α 2 ;
(c) somente A e C; (c) (1/3)a e (2/3)a; (c) a1 < a2 e α 1 = α 2 ;
(d) somente B e C; (d) a/3 e 5a/12. (d) a 1 < a2 e α 1 < α 2 ;

(e) somente B. (e) a/3 e ( 2/5)a (e) a1 > a2 e α 1 = α 2 .

1 2
9. Um dispositivo é constituido de dois arames (1) e 10. Duas partı́culas deslocando-se com momentos p~1 e Seção 2. Questões discursivas (2×2,5 = 5,0 pontos)
(2)ligando os pontos A e B. O arame (1) está esticado p~2 colidem entre si em uma região remota do espaço
formando um segmnento de reta e o (2) apresenta uma sideral, e sujeitas somente às suas interações mútuas F~12 Não serão consideradas respostas sem justificativa; expresse-as somente em função dos dados fornecidos.
forma sinuosa. Duas contas idênticas são soltas a partir (força que 1 exerce sobre 2) e F~21 (força que 2 exerce
do repouso no ponto A. Elas deslizam pelos arames sobre 1). Considerando P~ o momento linear total do 1. Um bloco de massa m comprime, em repouso, uma mola de
com atrito desprezı́vel e chegam em B, com velocidades sistema formado pelas duas partı́culas e as variações de constante elástica k na base de um plano inclinado do ângulo
de módulo v1 e v2 respectivamente, que correspondem momento linear ∆~p1 e ∆~p2 das partı́culas 1 e 2, antes e θ em relação à horizontal; a compressão é suficiente para fazer
às velocidades por onde deslizaram nos arames (1) e depois da colisão, é correta a opção o bloco se movimentar. A partir da posição inicial A, a mola
(2). A resposta correta abaixo é é liberada empurrando o bloco para cima ao longo do plano
(a) ∆~p1 = ∆~p2 ; inclinado. Verifica-se que na posição B, distanciada de D em
relação à posição A, o bloco está subindo o plano inclinado
(b) ∆~p1 = −∆~p2 ; com uma velocidade de módulo vB , e livre de contato com
(c) P~ não é conservado; a mola. Há atrito entre o bloco e o plano inclinado, cujo
coeficiente atrito cinético é µc . Determine:
(d) F~12 = F~21 ;
a) o trabalho realizado pela força de atrito entre as posições
(e) nenhuma das respostas anteriores pois não se co-
A e B;
nhece o tipo de colisão que ocorreu entre elas.
b) a energia potencial elástica da mola armazenada na
posiçõa A.

(a) v1 = v2 .
(b) v1 > v2 pois o arame (1) sempre está mais alto
que o arame (2). 2. Um dispositivo é contituı́do de dois discos ligados a um fio de
(c) v1 < v2 pois a distância percorrida pela conta massa desprezı́vel e inextensı́vel, como mostra figura. O disco
ao longo do caminho (2) é maior que a distância que está sobre o plano inclinado tem o fio ligado ao seu eixo
percorrida no caminho (1). de rotação e a outra extremidade do fio encontra-se enrolada
na periferia do disco cujo eixo de rotação está fixo. O disco
(d) O trabalho da força peso da conta que passa por do plano inclinado é liberado a partir do repouso e rola sem
(1) de A até B , é maior que o trabalho da força deslizar ao longo do plano mantendo-se sempre alinhado na
peso da conta que passa por (2) de A até B . vertical. Ele desce puxando o fio, desenrolando-o sem desliza-
(e) todas as opções acima estão erradas. lo sobre o disco fixo, que pode girar sem atrito em torno do seu
eixo de rotação. Os discos são homogêneos de massa M e raio
R e o ângulo de inclinação do plano é igual a θ em relação à
horizontal; 0 < θ < π/2. Dado que o momento de inércia dos
discos relativos aos seu eixos de rotação é (1/2)M R2 , calcule
a) a aceleração do centro de massa do disco que rola sobre o
plano inclinado;
b) o módulo T da tração do fio que liga os dois discos.

3 4
Universidade Federal do Rio de Janeiro
Centro de Ciências Matemáticas e da Natureza
Instituto de Fı́sica
Prova de Segunda Chamada de Fı́sica IA - 18/12/2013
Respostas para provas hı́bridas

Gabarito das Questões objetivas (valor=5,0 pontos)

Versão A Versão B
Questão (a) (b) (c) (d) (e) Questão (a) (b) (c) (d) (e)
1 1
2 2
3 3
4 4
5 5
6 6
7 7
8 8
9 9
10 10

Versão C Versão D
Questão (a) (b) (c) (d) (e) Questão (a) (b) (c) (d) (e)
1 1
2 2
3 3
4 4
5 5
6 6
7 7
8 8
9 9
10 10

1
Questão discursiva 1 (valor=2,5 pontos)
a) valor=1,0 ponto
O trabalho da força de atrito é dado por Wfat = f~at ·∆~rA−B , onde ∆~rA−B é o vetor deslocamento
de A → B. Como |f~at| = µc |N| ~ = µc mgcos θ e que |∆~rA−B | = D , obtemos

Wfat = −µc mgDcos θ

b) valor=1,5 pontos
Pelo Princı́pio da Conservação da Energia Mecânica para o Sistema bloco-mola, ∆E = Wfat .
Como E = 12 mv 2 + Uel + Ugrav , aplicando-o nas posições A e B,
1
EB − EA = [ mvB2 + 0 + mgDsen θ] − [0 + Uel(A) + 0] = −µc mgDcos θ
2

Onde vA = 0, e considerando Ugrav(A) = 0


1
∴ UA = mgD(sen θ + µc cos θ) + mvB2
2

2
Questão discursiva 2 (valor=2,5 pontos)

a) valor=2,0 pontos
Designando os discos como disco 1, o disco sobre o plano, e
disco 2 o disco fixo tem-se o diagrama de forças da figura. A
dinâmica para cada um deles é dada por:

disco 1

F~iext = m~acm ,
X X
~τi ext = I α
~1
i i

disco 2
X
~τi ext = I α
~ 2,
i

Assim temos, considerando o sentido de rotação positivo como anti-horário, os torques calcula-
dos em relação aos centros de cada disco respectivamente, |T~ | = |T~ 0| e que o disco (1) rola sem
deslizar acm = α1 R:


 Rfat = Iα1 


 
Disco 1 P sen θ − fat − T = Macm Disco 2 T = Iα2/R (i)

 


acm = α1 R

A condição de vı́nculo entre eles é que a aceleração tangencial do disco (2) tenha o mesmo
valor que a aceleração do centro de massa do disco (1), ou seja, aT2 = acm1 . Isto implica que
α1 R1 = α2 R2 . Como os raios dos discos são os mesmos,

α1 = α2 = α

Portanto ambos os discos giram com a mesma aceleração angular α!!


Logo, dos sistemas de equações Disco 1 e 2,

Mgsen θ = Iacm/R2 + Iacm /R2 + Macm ,

como I = (1/2)MR2 ,
1
∴ acm = gsen θ
2

b) valor=0,5 ponto
A tração do fio pode ser obtida de (i), após substituirmos o valor de acm encontrado no item
anterior,
Mgsen θ
T = Iacm /R2 → T = .
4

3
Instituto de Fı́sica - UFRJ

Processo Seletivo Monitoria de Fı́sica IA - 2013

Obs: em todas as questões em que for necessário, considere que g é o módulo da aceleração da gravidade

Questão 1) Dois blocos de massas m1 e m2 estão empilhados na base de um plano inclinado. Este
plano forma um ângulo θ com a horizontal. Os dois blocos estão em repouso na base do plano
como mostra figura abaixo. Os coeficientes de atrito estático e cinético entre todas as superfı́cies
dos blocos e do plano inclinado são os mesmos e valem µe e µc respectivamnete.
a) Indique em um diagrama todas as forças que atuam em cada bloco.
b) Determine a força de atrito que age sobre os blocos e o plano inclinado.
c) Aplica-se uma força F~ de módulo constante, paralela ao palno inclinado, no bloco de baixo.
Os dois acixotes começam a subir juntos. Qual omaior valor de F ~ para que isto ocorra?

Questão 2) Um disco de raio R e massa M está preso rigidamente a uma barra de comprimento
2R. A barra de massa M, por sua vez tem a sua extremidade livre presa a um pino por um eixo
perpendicular a uma parede vertical em torno do qual pode girar livremente. A seguir alinha-se a
barra horizontalmente e o sistema barra-disco é liberado a patrtir do repouso; vide a figura abaixo.
a) Qual é o momento de inércia do sistema barra-disco em relação ao eixo de rotação (extremidade
da barra)?
b) Qual a velocidade angular do sistema quando a barra faz um ângulo θ com a horizontal?
c) Qual a velocidade linear de um ponto localizado no centro do disco quando a barra estiver
alinhada verticalmente?

Dados: o momento de inércia de um disco de raio R e massa M para um eixo passando pelo
seu centro é (1/2)MR2 e para uma barra de comprimento L e massa M para um eixo passando
pelo seu centro é (1/12)ML2 .
5. Um arame quadrado de lado a, fino, rı́gido e de massa 7. Um carrinho percorre um trilho em forma de laço
UNIVERSIDADE FEDERAL DO RIO DE JANEIRO
desprezı́vel, tem nos seus vértices quatro massas iguais a vertical, como mostra a figura. Ele passa no ponto
INSTITUTO DE FÍSICA m. Os momentos de inércia IA , IB e IC , relativos aos res- mais alto A pressionando o trilho. Se g é o módulo da
FÍSICA I – 2014/1 pectivos eixos A, B e C mostrados na figura relacionam- aceleração da gravidade e a o módulo da aceleração
se como centrı́peta do carrinho no ponto A, o módulo N da força
PROVA DE SEGUNDA CHAMADA –11/06/2014 que o trilho exerce sobre o carrinho em A é igual a
VERSÃO: A (a) IA > IB > IC ;
(b) IA > IB = IC ;
Nas questões em que for necessário, considere que g é o módulo da aceleração da gravidade. (c) IA = IB = IC ;
(d) IA < IB < IC ;
Seção 1. Múltipla escolha (10×0,5 = 5,0 pontos) (e) IA < IB = IC .
(a) zero
1. Uma partı́cula que está sob a ação de exatamente duas 3. Duas partı́culas de massas ma e mb colidem frontal-
(b) m(a − g)
forças encontra-se acelerada. Podemos concluir que mente de forma totalmente inelástica na ausência de
forças externas. Antes da colisão a partı́cula de massa (c) m(a + g)
(a) a partı́cula não pode se mover com velocidade de mb encontrava-se em repouso. A razão entre a energia (d) mg
módulo constante; cinética inicial e final, Ki /Kf , do processo de colisão é
(e) Nenhuma das respostas anteriores.
(b) a sua velocidade nunca será nula; dada por
(c) a partı́cula não pode se mover com velocidade de (a) (ma + mb )/ma
direção constante; 6. Um ioiô é pendurado ao teto por um fio ideal. Ele é
(b) (ma − mb )/ma
(d) a soma das duas forças não pode ser nula; liberado verticalmente a partir do repouso como mostra
(c) (ma − mb )/mb a figura. O seu centro de massa cai de uma altura h
(e) nenhuma das respostas anteriores está correta.
(d) (ma + mb )/mb em relação a sua altura incial e, durante a queda, o seu
(e) ma /mb centro de massa move-se verticalmente. O ioiô durante
a queda não gira lateralmente. Despreze a resistência do
2. Três partı́culas têm momentos lineares constantes p~1 , p~2 ar e considere que o fio não desliza sobre o ioiô. Durante
e p~3 que desenhados consecutivamente formam um dia- a queda do ioiô é correto afirmar que
grama vetorial com a forma de um triângulo, como in-
dicado na figura. É necessariamente verdade que

4. Uma haste fina de massa desprezı́vel tem em sua extre-


midade uma pequena bola de dimensões desprezı́veis. A
bola inicialmente alinhada horizontalmente é empurrada
(a) a força externa resultante sobre o sistema não é e adquire uma velocidade de módulo v tal que a bola
nula; chega ao ponto mais alto com velocidade nula. Para que (a) em relação ao centro de massa o torque resultante
(b) as partı́culas estão em movimento circular em isto ocorra o módulo da velocidade v é igual a devido às forças peso e tração sobre o ioiô não é
sentido horário em torno de um mesmo centro; nulo;

(a) gL; (b) a força externa resultante sobre o ioiô é nula;
(c) as partı́culas estão em movimento circular em √
sentido anti-horário em torno de um mesmo cen- (b) 2gL; (c) em relação ao centro de massa do ioiô o torque

tro; (c) 2 gL; resultante é nulo;

(d) a velocidade do centro de massa das partı́culas é (d) 2 2gL; (d) em relação ao centro de massa do ioiô o torque
nula; √ da tração é nulo;
gL
(e) a energia cinética total das partı́culas é nula. (e) . (e) a energia cinética se conserva.
2

1 2
8. Uma partı́cula se desloca ao longo do eixo OX, da ori- 9. Uma partı́cula move-se sobre uma circunferência de raio Seção 2. Questões discursivas (2×2,5 = 5,0 pontos)
gem até a posição x3 = 3d, onde d é uma distância R, com velocidade de módulo constante v no plano hori-
positiva. A única componente da força resultante sobre zontal OXY . O módulo da taxa de variação instantânea Não serão consideradas respostas sem justificativa; expresse-as somente em função dos dados fornecidos.
a partı́cula, Fx , varia com a posição x conforme o gráfico com o tempo do momento linear da partı́cula é direta-
da figura, linearmente da origem a x2 = 2d, cortando o mente proporcional a 1. Um carro de massa m viajando de oeste para leste com velocidade
eixo OX em x1 = d, e também linearmente de x2 = 2d (a) R3 constante de módulo v desconhecido colide com um caminhão de
até x3 = 3d; no gráfico também está indicado o valor √ massa 2m viajando de sul para norte com velocidade constante de
máximo F0 da força e o valor mı́nimo −F0 . Denotando (b) R módulo V , também desconhecido. Na figura estão indicadas as
por K0 , K1 , K2 e K3 as energias cinéticas nas posições (c) R2 trajetórias do carro e do caminhão em um sistema de eixos OXY
x = 0, x1 = d, x2 = 2d e x3 = 3d, respectivamente, com origem O no ponto de colisão. Após a colisão os veı́culos
(d) v
podemos afirmar sobre as variações ∆K1 = K1 − K0 , permanecem juntos e se arrastam até parar por uma distância d em
∆K2 = K2 − K0 e ∆K3 = K3 − K0 que (e) v2 uma trajetória retilı́nea que faz um ângulo θ com a direção original
10. Um bloco encontra-se em repouso sobre um plano hori- do carro (eixo OX na figura). O coeficiente de atrito cinético entre
zontal ligado a uma mola (distendida horizontalmente) os veı́culos e o chão é µ e todos os movimentos são de translação
como indicado na figura. Considerando as forças f~at em um plano horizontal. Considerando como dados m, d, θ, µ e
~ (força normal), P~ (força peso) e
(força de atrito), N módulo g da aceleração da gravidade, determine;
F~m (força da mola) que agem sobre o bloco é correta a a) o trabalho realizado pela força de atrito desde o instante em
opção que ocorre a colisão até o instante em que o sistema constituı́do
pelo carro e caminhão para;
b) o módulo da velocidade do centro de massa sistema imediata-
mente após a colisão;
c) o módulo v da velocidade do carro e o módulo V da velocidade
do caminhão antes da colisão.

(a) N~ = F~m ;
(a) ∆K1 > ∆K2 > ∆K3 ;
(b) ∆K1 = ∆K3 < ∆K2 ; (b) f~at − F~m = 0;
2. Uma polia constituida por dois cilindros homogêneos rigidamente
(c) ∆K1 < ∆K2 < ∆K3 ; (c) F~m + f~at = 0; ligados e com o mesmo eixo de simetria, em torno do qual gira
(d) N~ + F~m = P~ + f~at ; sem atrito; os raios do cilindros são R e 2R e amassa do conjunto
(d) ∆K1 = ∆K3 > ∆K2 ;
formando a polia é M . Um fio inextensı́vel de massa desprezı́vel
(e) ∆K1 > ∆K2 e ∆K2 < ∆K3 . (e) ~ + f~at ;
P~ + F~m = N enrolado na periferia do cilindro de raio 2R está sendo puzado por
uma força constante F~ . Outro fio inextensı́vel de massa desprezı́vel
enrolado na periferia do cilindro de raio R está puxando um bloco,
também de massa M , em movimento vertical, como indica a figura.
Sabe-se que os fios se enrolam e desenrolam sem deslizar nas perife-
rias dos cilindros e que a força F~ é tal que a aceleração do bloco, tal
como sua velocidade, tem sentido para cima. Dado que o momento
de inércia da polia relativo ao eixo de simetria formada pelos dois
cilindros é M R2 , calcule
a) o módulo da aceleração do bloco;
b) a tensão no fio que puxa o bloco;
c) qual seria o valor do de F para que o bloco subisse com
velocidade constante.

3 4
Universidade Federal do Rio de Janeiro
Centro de Ciências Matemáticas e da Natureza
Instituto de Fı́sica
Prova de Segunda Chamada de Fı́sica IA - 11/06/2014
Respostas para provas hı́bridas

Gabarito das Questões objetivas (valor=5,0 pontos)

Versão A Versão B
Questão (a) (b) (c) (d) (e) Questão (a) (b) (c) (d) (e)
1 1
2 2
3 3
4 4
5 5
6 6
7 7
8 8
9 9
10 10

Versão C Versão D
Questão (a) (b) (c) (d) (e) Questão (a) (b) (c) (d) (e)
1 1
2 2
3 3
4 4
5 5
6 6
7 7
8 8
9 9
10 10

1
Questão 1) [2,5 pontos]
(a) [0,8 pontos]
Como o peso 3m~g do sistema e a reação normal N ~ sobre ele se cancelam, N = 3mg, de modo
que a força de atrito estático tem módulo f = µN = µ3mg. Portanto, o trabalho realizado por
ela no intervalo considerado é Wat = −fd = −µ3mgd, ou seja Wat = −3µmgd.

(b) [0,7 pontos] Como o peso do sistema e a normal se cancelam, a força resultante é a de
atrito e, pelo teorema do trabalho e energia cinética, o seu trabalho é a variação da energia
2
cinética do sistema no intervalo considerado, 0 − (1/2)3mvcm = Wat = −3µmgd, onde vcm é o
módulo da velocidade do centro de massa
√ do sistema imediatamente após a colisão. Portanto,
2
(1/2)3mvcm = 3µmgd, donde vcm = 2µgd.

(c) [1,0 ponto] Durante a √ colisão há conservação


√ do momento linear, de modo √ que, mv~ı +
2mV√~ = 3m~vcm , onde ~vcm = 2µgd cos θ~ ı+ 2µgd sin θ~, isto é, mv~ı+2mV
√ √ ~ = 3m 2µgd cos θ~ı+
3m
√ 2µgd sin θ~
 . Portanto,
√ mv = 3m 2µgd cos θ e 2mV = 3m 2µgd sin θ, ou seja, v =
3 2µgd cos θ e V = (3/2) 2µgd sin θ.

Questão 2) [2,5 pontos]


(a) [1,0 ponto]
Aplicando a segunda lei de Newton ao bloco, obtemos a equação T − Mg = Ma, na qual T é
a tensão no fio preso ao bloco e a sua aceleração. Aplicando o teorema do torque e aceleração
angular à polia, obtemos a equação F 2R − T R = Iα, onde I é o momento de inércia da polia
e α sua aceleração angular. Mas I = MR2 e α = a/R; logo: F 2R − T R = MR2 (a/R), isto é,
2F − T = Ma. Assim, temos as equações

T − Mg = Ma e 2F − T = Ma . (1)

Somando membro a membro essas duas equações, obtemos 2F − Mg = 2Ma, donde


F g
a= − . (2)
M 2
(b) [1,0 ponto]
Substituindo essa expressão de a na primeiro equação em (1), obtemos T − Mg = F − (Mg/2),
donde T = F + (Mg/2).

(c) [0,5 pontos]


O bloco subiria com velocidade constante no limite em que sua aceleração fosse zero. Essa
condição levada na equação (2) fornece a condição 0 = (F/M) − (g/2), isto é, F = Mg/2.

Você também pode gostar